Test Bank

Download as pdf or txt
Download as pdf or txt
You are on page 1of 197
At a glance
Powered by AI
The document appears to contain solutions to various accounting problems, with references to topics like partnerships, revenue recognition, and exchange differences.

Accounting problems and their solutions, with references to topics like partnerships, revenue recognition, and exchange differences.

Partnership problems involving capital contributions and profit/loss allocation. References to IAS 27 and revenue recognition standards. Examples of exchange differences disclosure requirements.

Catalog

AFAR - random9.pdf ····························································································································· 1


AFAR - random10.pdf ························································································································· 12
AFAR - random11.pdf ························································································································· 17
AFAR - random12.pdf ························································································································· 48
AFAR - random13.pdf ························································································································· 61
Solution 18-110 ························································································································· 95
Solution 18-110 (cont.) ············································································································· 95
Instructions ································································································································ 96
Solution 18-111 ························································································································· 96
Solution 18-112 ························································································································· 96
Solution 18-117 ························································································································· 97
Solution 18-117 (cont.) ············································································································· 97
Solution 18-118 ························································································································· 98
Solution 18-118 (cont.) ············································································································· 99
AFAR - random14.pdf ······················································································································· 104
AFAR 2 [random topics].pdf ·············································································································· 119
EASY

1. Which of the following costs shall be considered as both prime costs and conversion costs?
a. Supervisory salaries for manufacturing plant
b. Property taxes on manufacturing plant
c. Costs of direct materials used un production
d. Employee benefits earned by machine operators in producing the firm’s product

2. When a contract outcome cannot be estimated reliably, which action should be taken relative to recognizing revenue?

A. Recognize contract revenue and costs by reference to the stage of completion of the contract at the balance sheet
date.
B. Recognize revenue only when it becomes possible to foresee the outcome of the contract.
C. Recognize revenue only to the extent of costs incurred that it is probable will be recoverable.
D. None of the above

3. When treating exchange differences, what is included in income for the period?
A. Gains or losses arising when monetary items are settled at amounts different from their carrying value
B. Differences arising when monetary items held at the year-end are retranslated at the closing rate
C. Exchange differences arising from the translation of a foreign operation previously classified in equity
D. A and B
E. B and C

4. Which of the following examples of disclosures are required under IAS 27 (Revised)?

A. The entity has elected not to prepare consolidated financial statements. The entity measures its investments in
subsidiaries at fair value. The fair value was determined in accordance with its quoted price in the London Stock
exchange at 31 December 20X1 of 400.
B. The entity has elected not to prepare consolidated financial statements. The entity measures its investments in
subsidiaries at cost. The summarized financial information for the joint venture is as follows: currents assets –
100; non-current assets – 400; current liabilities – 300, non-current liabilities – 200; revenue – 1,200.
C. The entity has elected not to prepare consolidated financial statements. The entity measures its investments in
subsidiaries at fair value. The amount of the transactions with its Subsidiary B is 1,300. Trade receivables from
Subsidiary B are 200. The borrowings from Subsidiary B amount to 1,100.
D. The entity has elected not to prepare consolidated financial statements. The entity measures its investments
in subsidiaries and joint ventures at cost. The entity has two subsidiaries (ABC and DEF) and one joint venture
(JHG). The subsidiaries are wholly owned whereas the joint venture is owned at 50%. The activities of the
subsidiaries and joint venture are real estate.

5. To which financial statements is IAS 29 applied?


A. The primary statements of any entity that reports in the currency of a hyperinflationary economy
B. The interim statements of any entity that reports in the currency of a hyperinflationary economy
C. The cash flow statements of any entity that reports in the currency of a hyperinflationary economy
D. The balance sheet of the parent entity that owns an entity located within a hyperinflationary economy

6. PFRS 3 – Business Combinations does not apply to which of the following?


I. Formation of a joint arrangement.
II. Combination of entities or businesses under common control.
III. Acquisition of an asset or a group of assets that constitute a business.
IV. Acquisition by an investment entity of an investment in a subsidiary
V. Not-for-profit organizations.
a. I, II and III only b. I, II and IV only c. I, II, III and V only d. I, II, III, IV and V

7. Franchise fees received upon contract signing shall be recognized as income by the franchisor when the following
conditions are met, EXCEPT:
A. Substantial performance required under the contract is done
B. Period of refund for any amount received under the contract has expired
C. Franchise operations have earned considerable income to defray franchising expenses
D. Collectability of any promissory note arising from the franchise agreement is reasonably assured

8. Under PFRS 10, what factor/s should an investor consider in assessing whether it has de facto control over an entity?
a. Voting patterns at future shareholders meetings
b. Size of the investor’s holding of voting rights relative to the size of dispersion of other vote holders
c. Non-voting rights held by the investor or other vote holder
d. All of the above.

9. When will the average process costing method produce the same cost of goods manufactured as the FIFO process
costing method?
a. When materials are added 100% at the end of the process.
b. When materials are added 100% at the beginning of the process.
c. When the beg. WIP inventory and ending WIP are equal.
d. When there is no beg. WIP inventory.
10. Group A has acquired the following. Which of the following acquisitions are business combinations under IFRS 3?
A. Land and a vacant building from Company B. No processes, other assets or employees are acquired. Group A does
not enter into any of the contracts of Company B.
B. An operating hotel, the hotel’s employees, the franchise agreement, inventory, reservations system and all “back
office” operations.
C. All of the outstanding shares in Biotech D, a development stage company that has a license for a product
candidate. Phase I clinical trials are currently being performed by Biotech D employees. Biotech D’s administrative
and accounting functions are performed by a contract employee.
a. All three acquisitions are business combinations under PFRS 3.
b. A and B acquisitions are business combinations under PFRS 3.
c. A and C acquisitions are business combinations under PFRS 3.
d. B and C acquisitions are business combinations under PFRS 3.

MODERATE

11. Amounts that have been billed by the contractor but are not paid by the customer until the satisfaction of conditions
specified in the contract for the payment of such amounts, or until defects have been rectified.
a. Advances b. Incentives c. Retentions d. Progress billings

12. HFR Ltd. has a 12% holding in the shares of ABC Ltd. In addition, HFR has, through one of its subsidiaries, an option to
buy 13% more shares in ABC. Although the exercise price is in the money, HFR does not have the intention and the
financial ability to exercise this option.
a. A subsidiary b. An associate c. A join arrangement d. None of these categories

13. In reporting a company that is to be liquidated, assets are shown at


a. Book value b. Historical cost c. NRV d. Present value using effective rate

14. Under PFRS 15 (effective January 1, 2018), revenue from contracts with customers
a. Is recognized when the customer receive the right to receive consideration
b. Is recognized even if the contract is wholly unperformed
c. Can be recognized even when a contract is still pending
d. Cannot be recognized until a contract exists

15. Entity A acquired Entity B. On the acquisition date, Entity B had an operating lease as a lessee with a remaining period
of two years out of the original four years. Due to significant changes in the market, Entity B is paying less than what
you would expect to currently pay for a similar lease. The value of the existing lease based on the current terms is
10,000 and that of a lease based on relative market terms is 13,000. How should Entity A account for this?
a. Entity A should disregard this, as this is an operating lease of Entity B and no asset or liability is recognized related
to operating leases.
b. Entity A determines whether the terms of each operating lease in which Entity B is the lessee are favorable or
unfavorable. Entity A should account for the difference between the value of the existing lease terms and the
market terms in profit or loss.
c. Entity A determines whether the terms of each operating lease in which Entity B is the lessee are favorable or
unfavorable. Entity A should recognize an intangible asset separate from goodwill for the favorable portion of
the operating lease relative to market terms.
d. None of the above.

16. A partner’s drawing account is, in substance,


a. A capital account
b. A contra-capital account
c. A salary expense account
d. A loan account (a loan from the partnership)

17. Build Company recorded the following costs relating to the project of constructing a factory for a client: project
manager costs of 1,000, costs of 1,500 to destroy an existing old factory building, costs of 500 to restore an old factory
building, attributable insurance costs of 200, non-reimbursable general administration costs of 200, selling costs of
150, and reimbursable development costs of 200. Which of the following cost elements should not be included in the
contract costs according to IAS 11 Construction Contracts?
a. Costs relating to the destruction of an existing old factory building of 1,500 and restoration of an old factory
building of 500
b. Attributable insurance costs of 200 and general administration costs of 200
c. Costs relating to the destruction of an existing old factory building of 1,500, restoration of an old factory building
of 500, and general administration costs of 200
d. General administration costs of 200 and selling costs of 150
e. General administration costs of 200, selling costs of 150, and reimbursable development costs of 200

18. Binfathi Group acquired an 80% interest in Entity B. The consideration for the 80% interest in Entity B was P36,000 in
shares in Binfathi and P12,000 in cash. To issue the shares, Binfathi incurred a cost of P2,000 and incurred costs of
P1,400 associated with legal fees and the valuation of Entity B. The fair value of the net assets of Entity B amounted
to P64,000. How should Binfathi account for this acquisition?
a. Binfathi shall book a gain (negative goodwill) through profit or loss of 3,200 related to the acquisition, recognize
expenses of 1,400 and deduct from equity 2,000 relative to the cost of issuing the shares.
b. Binfathi shall book goodwill as an asset of 200.
c. Binfathi shall book a gain (negative goodwill) through profit or loss of 1,200 and recognize the costs of legal fees of
1,400 as expenses in profit or loss.
d. Binfathi shall book a gain (negative goodwill) though profit or loss of 3,200 and recognize expenses of 3,400, relative
to the costs of issuing shares, paying legal fees and performing the valuation of Entity B, in profit or loss.

19. Under the cost recovery method of revenue recognition (assuming properly disclosed in the notes to FS),
a. Income is recognized immediately
b. Income is recognized on a proportionate basis as the cash is received on the sale of the product
c. Income is recognized when the cash received from sale of the product is lower than the cost of the product
d. Income is recognized when the cash received from sale of the product is higher than the cost of the product

20. With which of the following disclosure requirements should an entity comply, according to IAS 11, Construction
Contracts (Select the incorrect item)?
a. The amount of contract revenue recognized as revenue in the period
b. The methods used to determine the stage of completion of contracts in progress
c. Advances received in cash at the balance sheet date, for each material contract
d. The methods used to determine the contract revenue recognized in the period

DIFFICULT

21. The “Home Office” ledger account in the accounting records of a branch is best described as:
a. An equity account b. A revenue account c. A liability account d. A deferred income account

22. The consideration transferred in the business combination was P55,000. Transaction costs amount to P1,000. The fair
value of the acquiree’s net assets at the acquisition date was P63,000. The acquirer has not yet decided whether to
measure the 20% non-controlling interest (NCI) in the acquiree at the NCI’s proportionate share of the fair value of
the acquiree’s net assets, which is P12,600, or at the NCI’s fair value, which is P13,000. Does the choice of measuring
the NCI impact the determination of goodwill at the acquisition date?
a. No, the accounting policy choice for NCI does not impact goodwill at the acquisition date.
b. Yes, it does. If the acquirer values the NCI at its proportionate share of the fair value of the acquired business,
the goodwill amounts to P4,600; if the acquirer values the NCI at its fair value, then the goodwill amounts to
P5,000.
c. Yes, it does. If the acquirer values the NCI at its proportionate share of the fair value of the acquired business, the
goodwill amounts to P5,600; if the acquirer values the NCI at its fair value, then the goodwill amounts to P6,000.
d. No, it does not. However, the accounting policy choice for NCI impacts the fair value of the acquiree’s net assets.
If the acquirer values the NCI at its proportionate share of the fair value of the acquired business, the acquiree’s
net assets amount to P63,000; if the acquirer values the NCI at its fair value, then the acquiree’s net assets amount
to P63,400.

23. In partnership liquidation, the final cash distribution to the partners should be made in accordance with
a. Partners’ profit and loss ratio
b. Balances of the partners’ capital accounts
c. Ratio of capital contributions by the partners
d. Ratio of capital contributions less withdrawals by the partners

24. Entity A had several business acquisitions during the reporting period and after the reporting period. Entity A will
disclose, among other information, the following:

 The name and a description of the acquiree


 The acquisition date
 The percentage of voting equity interests acquired
 The primary reasons for the business combination and a description of how the acquirer obtained control of
the acquiree

a. These disclosures shall be done for each business combination that occurred in the reporting period only, but are
not required for business combinations that occurred after the end of the reporting period.
b. These disclosures shall be done for each material business combination that occurred both in the reporting period
and after the end of the reporting period, but before the financial statements are authorized for issue. The
information is disclosed in aggregate for individually immaterial business combinations.
c. These disclosures are optional for each business combination that occurred both in the reporting period and after
the end of the reporting period, but before the financial statements are authorized for issue.
d. These disclosures shall be done for each business combination that occurred both in the reporting period and
after the end of the reporting period, but before the financial statements are authorized for issue.

25. Under the installment method of revenue recognition, when interest is charged, each cash collection made after the
sale is composed of:
a. Cost and profit
b. Cost and interest
c. Interest and profit
d. Cost, interest and profit

26. Entity A acquired Entity B, which is a material business combination, during the reporting period. Among the assets
acquired, trade accounts receivable were provisionally accounted for at fair value of 1,736. Which of the following
information shall be provided additionally to the fair value amount of the trade accounts receivable? Select all that
apply.

I. Entity A does not need to disclose any further information.


II. Entity A must disclose that the fair value of the accounts receivable was determined provisionally.
III. Entity A must disclose the nominal value of the accounts receivable.
IV. Entity A must disclose the amount of the contractual cash flows that it does not expect to collect.

a. I, II, III and IV b. I, II and III only c. II, III and IV only d. I and II only

27. Which of the following is/are false?


I. When estimating the outcome of cost-plus contracts, it is necessary to be able to predict the total costs, past and
future, in order to assess the final profit, and also to make accurate assessments of the stage of completion that
has been reached at the balance sheet date.
II. In the case of a service provider, inventories (essentially their work in progress) should include profit margins and
non-attributable overheads.
a. I only b. II only c. I and II d. Both are True

28. In partnership liquidation, the final cash distribution to the partners should be made in accordance with
a. Partners’ profit and loss ratio
b. Balances of the partners’ capital accounts
c. Ratio of capital contributions by the partners
d. Ratio of capital contributions less withdrawals by the partners

29. In preparing the combined financial statements of the home office and its various branches:
a. Both reciprocal and nonreciprocal accounts are combined
b. Both reciprocal and nonreciprocal accounts are eliminated
c. Reciprocal accounts are eliminated but nonreciprocal accounts are combined
d. Reciprocal accounts are combined but nonreciprocal accounts are eliminated

30. The goodwill resulting from the acquisition of Entity C by Entity B amounts to 50,000. Which disclosures does Entity B
provide relating to the goodwill? Select all that apply.

I. Entity B shall describe the factors that make up the goodwill to be recognized.
II. Entity B shall disclose the total amount of goodwill deductible for tax purposes.
III. Entity B shall disclose the amortization period of goodwill for tax purposes.

a. I, II and III b. I and II only c. I and III only d. I only

CLINCHER

Vex, general manager of AB Corporation, provided the following information for transactions that occurred during August.
The corporation uses JIT costing system:

 Raw materials purchased and requisitioned for product were ₱84,000.


 Direct labor costs of ₱78,000 were incurred.
 Actual factory overhead costs amounted to ₱250,000.
 Applied conversion costs totaled ₱340,000. This included ₱78,000 of direct labor.
 All units were completed.

31. How much is the balance of Finished Goods account in August 31?
a. P412,000 debit b. P424,000 debit c. P412,000 credit d. P424,000 credit

PBC Company’s Job 004 manufactured 13,750 units that were completed in February at unit costs presented as follows:

Direct materials P300


Direct labor 270
Factory overhead (includes an allowance of P15
Spoiled work) 270

Final inspection of Job 004 disclosed 1,250 spoiled units, which were sold for P225,000.

32. What would be the unit cost of good units if the spoilage loss is attributable to exacting specifications of Job 004?
a. P 840 b. P 889.50 c. P 825 d. P 862.50
On January 1, 2016, Bruno Co. acquired all of the identifiable assets and assumed all liabilities of Mars, Inc. by paying
P1,000,000. On this date, identifiable assets and liabilities assumed have fair value of P1,600,000 and P900,000,
respectively. Terms of the agreement are as follows: (a) 30% of the price shall be paid on January 1, 2016; (b) the balance
on December 31, 2017 (the prevailing market rate on the same date is 12%). The acquirer shall also transfer its piece of
land with book and fair value of P500,000 and P300,000, respectively. Included in the liabilities assumed is an estimated
liability for deficiency taxes. The carrying amount and fair value of this provision amounted to P120,000 and P97,500,
respectively. The acquiree guarantees that this provision would only be settled for P90,000.

33. The amount of indemnification asset to be recognized, if any


a. P30,000 b. P22,500 c. P7,500 d. P0

CP, LK and TQ share profits in the ratio of 3:5:2. On April 30, LK opted to retire from the partnership. The capital balances
on this date follow:

CP P280,000
LK P350,000
TQ P320,000
34. Assuming LK sold her interest to TQ for P375,000, which of the following statements is false upon retirement of LK?
a. LK’s personal assets will increase by P375,000.
b. The capital account of CP will not change.
c. TQ’s capital account in the partnership will increase by P670,000.
d. The total capital of the partnership after the retirement of LK is P950,000.

BTS Company acquired all of the outstanding shares of BigBang Company by issuing its own P15 par value ordinary shares
totaling 46,667 shares at market price of P 15.70. BTS Company had the following expenditures incurred:

Finder’s fee paid P 50,000


Pre-acquisition audit fee/accounting due diligence, 30% was paid 40,000
General administrative costs 15,000
Legal fees for the combination paid 32,000
Audit and legal fees for SEC registration of share issue 46,000
Listing fees paid for the shares issued 10,000
Other share issuance costs paid (inclusive of any tax cost) 10,000
Other indirect costs paid 16,000
Documentary stamp tax (DST) paid on the original issuance 3,500

35. The total amount debited to expense should be


a. P 153,000 b. P 163,000 c. P 176,333 d. P 179,833

CV and LX are partners with profit and loss of 80:20 and capital balances of P700,000 and P350,000, respectively. TM is to
be admitted into the partnership by purchasing a 30% interest in the capital, profit and losses for P420,000. Assuming that
no asset revaluation is to be made,

36. Which of the following is true in the books of the partnership upon admission if TM?
a. Increase in assets in the amount of P420,000.
b. Credit capital accounts of the selling partners with total amount of P315,000
c. The entry upon admission will not affect the total capital of the partnership.
d. Decrease in capital account of the acquiring partner in the amount of P105,000.

XXX Inc. sells automatic weapons costing P700,000 at a price of P1,200,000. Division Corp. buys a dozen of automatic
weapons on installment and trade in six of its old weapons at a trade-in value of P300,000 each. XXX Inc. spends P25,000
to recondition the old guns and sells them for P315,000. XXX Inc. expects a 10% gross profit from the sale of used guns.

37. What is the under-allowance granted by XXX Inc, on the trade-in transaction?
a. P249,000 b. P234,000 c. P99,000 d. P0

On December 31, 2016, the following figures were taken from the trial balances of Blackpink Company and 2ne1 Co.:

Blackpink 2ne1
Current assets P 175,000 P 65,000
Noncurrent assets 725,000 425,000

Liabilities 65,000 35,000


Ordinary Share Capital, P20 par 550,000 300,000
Share Premium 35,000 25,000
Accumulated profits (losses) 250,000 130,000

On January 1, 2017, Blackpink issues 35,000 shares with a market value of P25/share for the net assets of 2ne1. The book
value reflects the fair value of the assets and liabilities, except that the noncurrent assets of 2ne1 have fair value of
P630,000 and Blackpink discovered an error on its books that resulted into an overstated noncurrent asset of P30,000.
Contingent consideration payable after 2 years if profit target will be achieved, which is determinable, have an expected
value of P18,151. Applicable discount rate on this type of agreement is 10%. Blackpink also paid for the share issuance
costs worth P34,000 and other acquisition related costs amounting to P19,000. Based on the foregoing, determine the
following:

38. The amount of gain on bargain purchase recognized


a. P215,000 b. P233,151 c. P230,000 d. P0

The company signed an P800,000 contract to build an environmentally friendly access trail to Morayta, Manila. The project
was expected to take approximately 3 years. The following information was collected for each year of the project – Year 1,
Year 2, and Year 3:

Cost Expended Expected Support Additional Trail feet Additional trail


during the Year additional cost to timbers laid support timbers constructed feet to be
completion during the year to be laid during the year constructed
Year 1 P100,000 P450,000 150 850 3,000 15,200
Year 2 150,000 280,000 300 520 7,500 8,200
Year 3 250,000 -0- 500 -0- 8,000 -0-

Compute the amount of revenue to be recognized in Year 3, assume that the company employs

39. the efforts-expended method of estimating percentage of completion, if the company measures its progress by the
number of support timbers laid in the trail
a. P428,864 c. P428,864
b. P422,640 d. P350,800

The Brooke Corporation has two branches, Branch P and Branch Q. The home office shipped P80,000 in merchandise to
Branch P and prepaid the freight charges of P500. A short time thereafter, Branch P was instructed to ship this merchandise
to Branch Q at a prepaid freight cost of P700. Freight charges for this merchandise normally cost P800 when shipped from
the home office directly to Branch Q.

40. Compute the excess freight on transfers of merchandise:

a. P700 b. P800 c. P500 d. P400

On May 31, 2016, TVD Company, a subsidiary of CW Philippines Corporation, through TO Inc., completed the purchase of
the net assets (including certain contracts) of Archie Company. The transaction between TVD Company and Archie
Company qualifies for recognition under PFRS 3 since it involves acquisition of group of assets qualifying as a business. The
total purchase price paid for said acquisition is P2 M. Based on the guidance provided for under IFRS 3, below are the fair
values of Archie Company’s assets and liabilities:

Real Property Leases [nil]


Personal Properties (BV is P600,000) 800,000
Business Contracts [nil]
Customer and Supplier List [nil]
Transportation and Warehouse
Management System [nil]
Unearned Revenues [nil]
Key employees [nil]
Entities involved are all subject to the 30% regular corporate income tax (RCIT). Based on the tax rules in the Philippines
for this type of acquisition, TVD Company can depreciate the acquisition cost of the assets other than land (if any) acquired
over the remaining life thereof and claim the same as a deduction for income tax purposes. TVD Company is expected to
be in net income/taxable income position in the future and is not expected to incur any losses for both accounting and tax
purposes. Based on the foregoing, determine the following:

41. Total amount of net assets (including DTA/DTL, if any) that will be considered in determining the goodwill or gain on
bargain purchase
a. P1,220,000 c. P800,000
b. P1,160,000 d. P0 since acquisition of assets only
On January 1, 2016 an entity purchased a tract of vacant land that is situated overseas for Baht90,000. The entity classified
the land as investment property. The fair value of the land at December 31, 2013 is Baht100,000. The entity’s functional
currency is Php (Peso).

Spot currency rates:


January 1, 2013 : 1 Baht= P2
Weighted average exchange rate in 2013 : 1 Baht = P2.04
December 31, 2013 : 1 Baht = P2.10

42. What is the carrying amount of the investment property at December 31, 2016 and what amount would be presented
in profit or loss for the year ended December 31, 2016?
a. Carrying amount of investment property = P210,000. Profit for the year includes P30,000 increase in the fair
value of the investment property.
b. Carrying amount of investment property = P210,000. Profit for the year includes P20,400 increase in the fair value
of the investment property and P9,600 foreign exchange (forex) gain.
c. Carrying amount of investment property = P180,000. Profit for the year includes no amount in respect of the
investment property.
d. Carrying amount of investment property = P189,000. Profit for the year includes P9,000 forex gain.

43. The ABC Chemical Company produces a product known as “minergy” from which by-product results.
• This by-product can be sold at ₱10 per pound.
• The manufacturing costs of the main product and by-product up to the point of separation for the three months
ended March 31, 2012 follows: Materials, ₱175,000; Labor, ₱100,000; Overhead, ₱100,000.
• The units processed were 35,000 pounds of the main product and 3,500 pounds of the by-product.
• During the period, 31,500 pounds of the “minergy” were sold at ₱48; while the company was able to sell 2,625
pounds of the by-product.
• Selling and administrative expenses related to the main product amounted to ₱210,000.
• Disposal cost per each unit of the by-product is ₱2.

Assume that the by-product is inventoried and recorded at net realizable value. The net realizable value of the by-product
reduces the manufacturing costs of “minergy”. What is the unit cost of “minergy”? Assume that the by-product is recorded
as realized. What is the cost of inventory of “minergy”?

a. P10.71; P37,500 b. P9.91; P37,500 c. P9.91; P34,700 d. P10.71; P34,700

The historical comprehensive income statement of Reese Company for 2016

Sales 2,500,000
Less: Cost of sales
Inventory, January 1 175,000
Add: Purchases 1,250,000
Less: Inventory, December 31 250,000 1,175,000
Gross profit 1,325,000
Less: Operating expenses, other than depreciation 1,000,000
Depreciation expense 1,000,000
Net loss 675,000
 Sales were earned, purchases other than ending inventory were made and operating expenses other than depreciation
expense were incurred evenly throughout the year.
 Ending inventory was acquired during the last week of December 2016
 Depreciable assets were acquired on January 1, 2013
 General price indices were:
January 1, 2013 125
January 1, 2016 140
December 31, 2016 360
44. If Reese Company was operating in a hyperinflationary economy, the amount to be reported as net income (loss) is

a. P2,720,000 b. P2,412,000 c. P972,000 d. P675,000

On December 31, 2016, Conti’s Inc. authorized Mary Grace Co. to operate as a franchisee for an initial franchise fee of
P3.40 million (M). Upon signing the contract, P0.90M was received and the balance is paid by a note, due in 5 equal annual
installments, beginning December 31, 2017. The prevailing market rate is 12%. The down payment is nonrefundable and
it represents a fair measure of the services already performed by Conti’s and substantial future services are still required.

45. How much is the deferred revenue to be recognized as of December 31, 2016?
a. P 1,518,677 b. P 1,802,390 c. P 2,500,000 d. P 2,702,390
BUZZER ROUND

Forrest Company uses standard cost system for its production process and applies overhead on direct labor hours. The
following information is available for August when Forrest made 4,500 units:

Standards:
DLH per unit 2.50
Variable overhead per DLH P1.75
Fixed overhead per DLH P3.10
Budgeted variable overhead P21,875
Budgeted fixed overhead P38,750
Actual:
Direct labor hours 10,000
Variable overhead P26,250
Fixed overhead P38,000

46. Using two-variance approach, what is the controllable variance?

a. P 5,812.50 U b. P 5,812.50 F c. P 4,375 U d. P 4,375 F

On January 2, 2016, GCC Corporation purchase 80% of VIP Company’s outstanding shares for P19,000,000. Included in the
price paid is control premium amounting to P500,000. The direct cost (acquisition related) amounted to P45,000 was
debited as part of the investment in subsidiary account since GCC opted to use the cost method of accounting its
investment in accordance with PAS 27. NCI is measured at the present ownership instruments' proportionate share in the
recognized amounts of the VIP's identifiable net assets. At that date, VIP had P16M of ordinary shares outstanding and
accumulated profits of P6.40M. GCC’s accumulated profits at the date of acquisition was P13.80M.

VIP’s equipment with remaining life of 5 years had a book value of P9.00M and a fair value of P10.52M. VIP’s remaining
assets had book value equal their fair values. All intangible assets except goodwill are expected to have remaining lives of
8 years. The income and dividend figures on the separate financial statements (SFS) for both GCC and VIP are as follows:
Net income of GCC in 2016 is P3.60M; 2017 is P4.40M. Net income of VIP in 2016 is P1.36M; 2017 is P2.04M. Dividends
declared by GCC in 2016 is P0.88M; 2017 is P1.56M. Dividends declared by VIP in 2016 is P0.28M; 2017 is P0.52M.

47. Non-controlling interest in net assets (NCINAS) in 2017


a. P 5,000,000 b. P 5,209,600 c. P 5,158,000 d. P 5,182,400

A taxpayer from the city of Las Pinas has the following information relating with his real property: FMV of Land, P500,000;
FMV of Res. House, P1,500,000. The one percent (1%) real property tax and 1% special education tax are both based on
the assessed value of the real property. The assessed value is 20% of the fair market value. Garbage fees amounted to
P500.

48. How much is the total amount collected from the taxpayer?
a. P 40,500 b. P 8,500 c. P 400,500 d. P 4,500

49. Manila Home Company ships and bills merchandise to its provincial branch at cost. The branch
carries its own accounts receivable and makes its own collections. The branch also pays its expenses.

The transactions for 20x3 are reflected in the branch trial balance that follows:

Debit Credit
Cash P11,900
Manila Home Co. Current P90,000
Shipments from Manila Home Co. 120,000
Accounts Receivable 62,500
Expenses 8,100
Sales 112,500
Total P202,500 P202,500

The net profit of the branch


A. P22,500
B. P21,300
C. P14,400
D. P12,400
50. BB Inc., DD Inc., and GG Inc. agree to consolidate. It was agreed that the new corporation will issue a single class of
stock at P100 par value. The new shares will be exchanged for net assets transferred taking into account the effect of
goodwill represented by annual earnings in excess of 6% on asset contributions, capitalized at 20%. Goodwill
calculations are made only for the purpose of making an equitable allotment of the new shares among the
constituent corporations. Their assets and estimated annual earnings follow:

Asset Contributions Earnings Contributions


BB P200,000 P30,000
DD 300,000 30,000
GG 500,000 40,000

If the new corporation is to be issued 1,000 shares, how will these be distributed among BB, DD,
and GG, respectively?
BB DD GG
A. 200 300 500
B. 300 300 400
C. 450 300 250
D. 242 300 458

51. On September 3, 20x3, Pia placed a noncancellable purchase order with a Japanese company for a custom-built
machine. The contract price was 1,000,000 yens. The machine was delivered on December 23, 20x3. The invoice was
dated November 13, 20x3, the shipping date (FOB shipping point). The vendor was paid on January 7, 20x4. The spot
direct exchange rates for the Japanese yens on the respective dates are as follows:

Sept. 3, 20x3 Nov.13, 20x3 Dec. 23, 20x3Dec. 31, 20x3 Jan. 7, 20x4
P.20 P.21 P.22 P.23 P.24

What amount is the capitalizable cost of the equipment?


A. P200,000 C. P220,000
B. P210,000 D. P230,000

52. On August 1, 20x3. Yellow Company paid P1,240,000 for all the issued and outstanding common shares of Green,
Inc. The basic financial information of Green Inc. as of said date registered asfollows:

Cash P120,000
Inventory 360,000
Property and Equipment (net of accumulated depreciation,P440,000) 640,000
Goodwill 200,000
Bonds Payable (240,000)

The following information is relevant:


- Green owns its factory which is included in the accounts at P100,000 and no adjustment had been
made to recognize the valuation of P220,000 put on the property when it was professionally revalued
on July 15, 20x3.
- The fair values of Green’s inventory on August 1, 20x3 is estimated to be P60,000 less than its book
value at that date.

The amount of goodwill generated therefrom:


A. Zero
B. P300,000
C. P240,000
D. P100,000

53. The following account balances were available for the Perry, Quincy, and Renquist partnership just before it entered
liquidation:

Cash $ 90,000 Liabilities $ 170,000


Noncash assets 300,000 Perry, capital 70,000
Quincy, capital 50,000
Renquist, capital 100,000
Total $ 390,000 Total $ 390,000

Perry, Quincy, and Renquist had shared profits and losses in a ratio of 2:4:4. Liquidation expenses were expected
to be P8,000. All partners were solvent. What would be the minimum amount for which the noncash assets must
have been sold for, in order for Quincy to receive some cash from the liquidation?
a. any amount in excess of P117,000.
b. any amount in excess of P183,000.
c. any amount in excess of P198,667.
d. any amount in excess of P168,333.
54. Soriente, Santos, and Salazar formed a joint venture. Soriente has been designated as manager of the venture, of
which he is to receive a bonus of 15% of the profit after deduction of the bonus as an expense. The net profit, after
bonus, has been agreed to be divided as follows: Soriente, 25%; Santos, 40%; and Salazar,35%. After five months, the
joint venture is terminated as of May 31, 20x3. On this date, the trial balance kept by Soriente contains the following
balances:
Debit Credit
Joint Venture 90,000
Santos 5,000
Salazar 20,000

The venture has still some undisposed merchandise which Soriente agreed to purchase at its cost of P25,000. The
bonus of Soriente has not yet been taken up. The bonus to Soriente is
a. P115,000 c. P11,500
b. P15,000 d. P100,000

55. Sharp Company's Subic branch submitted the following information for 20x3, Subic branch's first year of operations:
Sales 203,500
Expenses 18,755
Shipments from home office 186,120 debit
Home office– current 48,125 credit

Shipments to branches are billed at cost. Subic further reported an ending inventory (31 December) of P25,245. For
20x3, the branch reported to the home office net profit of
a. P25,245 c. P48,125
b. P42,625 d. P23,870

56. The National Home Company ships and bills its Provincial branch merchandise at cost. The branch carries its own
accounts receivable, makes its own collections and pays its expenses.

The transactions in 20x3 are reflected in the branch trial balance as follows:
Debit Credit
Cash 11900
National Home Co. Current 90000
Shipments from National Home Co. 120,000
Accounts Receivable 62,500
Expenses 8,100
Sales 112,500

Branch inventory on December 31, 20x4, P30,000. How much is the net profit of the branch?
a. P22,500
b. P14,400
c. P2l,900
d. P104,400

57. The branch manager of Blue Corporation in Cebu submitted a report as of May 31, 20x3 containing the following
information:
Petty cash fund 1,500
Sales 198,720
Sales returns 3,600
Accounts written off 1,920
Shipments from home office 136,000
Accounts receivable - May 31,20x2 43,800
Accounts receivable - May 31, 20x3 49,140
Inventory, May 31,20x2 37,170
Inventory, May 31,20x3 41,370
Expenses 57,930

Based on the above data, you can assume that on May 31, 20x2, the Branch current account had balance of
a. P82,470 c. P95,930
b. P136,600 d. P 1,500

58. Hannah Corporation repossessed goods previously sold and recorded the repossessed inventory at P15,250. The
business eventually resold the repossessed inventory for P30,000 after incurring remodeling costs of P2,750. The
buyer initially paid P8,000 and promised to pay the remaining amount in 3 equal monthly payments. Assuming the
installment method is used, how much gross profit was realized upon receipt of the P8,000 down payment?
A. 8,000 c. 3,920
B. 4,800 d. 3,200
59. Jennifer Company sold goods amounting to P150,000 on June 19, 2012. The company is a wholly owned subsidiary of
Jen Incorporated who presents its financial statements in dollars. Exchange rates for the dollar on specific dates are
as follows:
January 1, 2012 $1: P42
June 19, 2012 $1: P44
December 31, 2012 $1: P45
Average for 2012 $1: P43

When this transaction is translated in term of the presentation currency, how much will the sales be presented?
a. $3,571 c. $3,333
b. $3,409 d. $3,488

60. On March 1, Chow Corporation entered into a firm commitment to purchase specialized equipment from the Gifu
Trading Company for ¥80,000,000 on June 1. The exchange rate on March 1 is ¥100 = P1. To reduce the exchange
rate risk that could increase the cost of the equipment in U.S. pesos, Chow pays P20,000 for a call option contract.
This contract gives Chow the option to purchase ¥80,000,000 at an exchange rate of ¥100 = P1 on June 1. On June 1,
the exchange rate is ¥105 = P1. How much did Chow save by purchasing the call option (answers rounded to the
nearest peso)?
a. P20,000
b. P27,619
c. P47,619
d. Chow would have been better off not to have purchased the call option.
College of Business Administration

ACTG 105A - ADVANCED FINANCIAL ACCOUNTING AND REPORTING 1


FINAL EXAMINATION

NAME:_________________________________________________COURSE & YEAR: _____________________

Instructions: 1. AVOID CHEATING. It’s better to fail than to fool yourself. Answer the test the best way you
can. You will reap rewards for that.
2. BE STICK TO ONE. It would be better if you choose the answer by avoiding erasures.
3. LEARN TO LET GO. Do not dwell too much time on one item. Move to the next and go back for
it later.

Multiple Choice Questions: Encircle the letter of your best choice.

1. Which of the following is not a characteristic of a partnership?


A) The partnership itself pays no income taxes.
B) It is easy to form a partnership.
C) Any partner can be held personally liable for all debts of the business.
D) A partnership requires written Articles of Partnership.

2. Which of the following statements is correct regarding the admission of a new partner?
A) A new partner must purchase a partnership interest directly from the business.
B) The right of co-ownership in the business property can be transferred to a new partner
without the consent of other existing partners.
C) The right to participate in management of the business can be conveyed without the consent of other
existing partners.
D) The right to share in profits and losses can be sold to a new partner without the consent of other existing
partners.

3. Withdrawals from the partnership accounts are typically not used


A) to record compensation for work performed in the business.
B) to reduce the partners' capital account balances at the end of an accounting period.
C) to record interest earned on a partner’s capital balance.
D) to reduce the basic investment that has been made in the business.to record a reward for ownership in
the partnership.

4. The partnership contract for Hanes and Jones LLP provides that Hanes is to receive a bonus of 20% of net
income (after the bonus) and that the remaining net income is to be divided equally. If the partnership
income before the bonus for the year is P57,600, Hanes’ share of this pre-bonus income is:
A) 28,800.
B) 33,600.
C) 34,560.
D) 43,200.

5. When Danny withdrew from John, Daniel, Harry, and Danny LLP, he was paid $80,000, although his capital
account balance was only $60,000. The four partners shared net income and losses equally. The journal
entry of the partnership to record Danny's withdrawal preferably should include :
A) 6,667 debit to John, Capital.
B) 6,667 credit to John, Capital.
C) 6,667 debit to John, Drawing.
D) 5,000 debit to John, Capital.

6. When Mr. B is admitted to the partnership, the fair value of the assets he contributes exceeds his initial
capital balance. In this case, who gets the bonus?
A) Mr. B gets the bonus
B) The old partners get the bonus
C) No partner will get a bonus
D) None of the above

7. When Rivera retired from the partnership of Rivera, Nolasco and Andres, the final settlement of Rivera’s
interest exceeded Rivera’s capital balance. They share profits and losses equally. Under the bonus method,
the excess
A) Had no effect on the capital balances of Nolasco and Andres
B) Reduced the capital balances of Nolasco and Andres
C) Was recorded as goodwill
D) Was recorded as expense

8. The December 31, 2017 balance sheet of the ABC Partnership follows:
Cash P100,000 B, Loan P100,000
Non-cash Assets 500,000 A, Capital 100,000
B, Capital 200,000
C, Capital 200,000
Total P600,000 Total P600,000
The partners share profits and losses as follows: A, 20%; B, 30%; C, 50%. B is retiring from the
partnership and the partners have agreed that the non-cash assets should be adjusted to fair value of
P600,000 at December 31, 2017. They further agreed that B will receive P344,000 cash for his total
partnership interest.
After B’s retirement, the capital balances of A and C, respectively, will be:

Page 1
A) P116,000 and P240,000 C) P100,000 and P200,000
B) P101,714 and P254,286 D) P73,143 and P182,857

9. De Lima and Henares, partners sharing profits in the ratio of 60% and 40% wants to retire. The partners
agree that the fixed assets are undervalued by P35,000 and that De Lima’s share of this increase shall be
recorded and creditable to her capital account. Since the working capital is only P70,000, it is decided that
De Lima shall receive only one-third of her adjusted capital credit in cash. For the remainder, she accepts
securities, which have been carried as other assets at their book value and market value of P 12,000 and a
six-month note payable. The statement of financial statement, which then prepared, appears as follows:
Current assets P 53,000 Current liabilities P 52,000
Other assets 3,000 Henares, capital 50,000
Fixed assets 46,000
P 102,000 P 102,000

10. Current liabilities before De Lima’s retirement must be:


A) P52,000 C) P10,000
B) P80,000 D) P42,000

11. Other assets before De Lima’s retirement must be:


A) P 3,000 C) P15,000
B) P 12,000 D) -0-

12. PP, QQ and RR, partners to a firm, have a capital balances of P11,200, P13,000 and P5,800, respectively,
and share profits in the ratio of 4:2:1. Prepare a schedule showing how available cash will be given to the
partners as it becomes available. Who among the partners shall be paid first with an available cash of
P1,400?
A) QQ C) RR
B) No one D) PP

13. Gardo and Gordo formed a partnership on July 1, 2017 to operate two stores to be managed by each of
them. They invested P30,000 and P20,000 and agreed to share earnings 60% and 40%, respectively. All
their transactions were for cash, and all their subsequent transactions were handled through respective
bank accounts as summarized below:
Gardo Gordo
Cash Receipts P79,100 P65,245
Cash Disbursements 62,275 70,695

On October 31, 2017, all remaining noncash assets in the two stores were sold for cash of P60,000. The
partnership was dissolved and cash settlement was affected. In the distribution of the P60,000 cash, Gardo
received:
A) P24,000 C) P34,000
B) 26,000 D) 36,000

14. The assets and equities of the Queen, Reed and Stac Partnership at the end of its fiscal year on October 31,
2017 are as follows:

Assets Liabilities & Equity


Cash P15,000 Liabilities P50,000
Receivables - net 20,000 Loan from Stac 10,000
Inventory 40,000 Queen, capital - 30% 45,000
Plant assets - net 70,000 Reed, capital - 50% 30,000
Loan to Reed 5,000 Stac, capital - 20% 15,000
Total Assets P150,000 Total Liabilities & Equity P150,000

The partners decide to liquidate the partnership. They estimate that the noncash assets, other than the loan
to Reed, can be converted into P100,000 cash over the two-months period ending December 31, 2017.
Cash is to be distributed to the appropriate parties as it becomes available during the liquidation process.
The partner most vulnerable to partnership losses on liquidation is:
A) Queen C) Reed and Queen equally
B) Reed D) Stac

15. Using the same information in no. 14, and P56,000 is available for first distribution, it should be paid to:
Priority Creditors Queen Reed Stac
A) P60,000 P5,000 P 0 P 0
B) 60,000 1,500 2,500 1,000
C) 50,000 5,000 0 10,000
D) 50,000 12,000 0 3,000

Rebecca Miranda and Stephanie Calamba are partners with capital balances of P400,000 and P200,000, respectively.
They share profits in the ratio of 3:1. The partners agreed to admit Theodore Calaguas as a member of the firm.

16. If Theodore Calaguas invested P250,000 for a one-fourth interest in the business and the partners decided
not to revalue the assets of the partnership and that the total agreed capital is P850,000, how much will be
credited to Calaguas?
A) 250,000 C) 209,375
B) 212,500 D) 200,000

17. If Theodore Calaguas invested P400,000 in the business and P100,000 is considered bonus to existing
partners, how much will be credited to Calaguas?
A) 225,000 C) 200,000
B) 400,000 D) 300,000
18. If Theodore Calaguas invested P240,000 in the business for a one-third interest in the business, how much
will be credited to Calaguas?
A) 240,000 C) 280,000
B) 200,000 D) 190,000

Page 2
19. If Theodore Calaguas invested P300,000 for a 50% interest in the business, how much will be credited to
Calaguas?
A) 450,000 C) 300,000
B) 150,000 D) 200,000

20. Using no. 19 information, how much is the new capital of Miranda after the admission of Calaguas?
A) 162,500 C) 37,500
B) 287,500 D) 112,500

21. Salaries to partners of a partnership typically should be accounted for as:


A) Drawings by the partners from the partnership
B) A device for sharing net income
C) An operating expense of the partnership
D) Reductions of the partners’ capital account balances

22. In the cash distribution plan, how is the amount of a cash distribution determined?
A) By multiplying a partners’ profit and loss ratio by his pre-liquidation capital balance
B) By subtracting a partners’ loss absorption potential from the loss absorption potential of the next
strongest partner
C) By subtracting a partners’ loss absorption potential from the loss absorption potential of the next
strongest partner and dividing this difference by his profit and loss ratio
D) By multiplying a partners’ profit and loss ratio by the difference between his loss
absorption potential and the loss absorption potential of the next strongest partner.

23. The partnership has the following accounting amounts:


a) Sales = P70,000
b) Costs of Goods Sold = P40,000
c) Operating Expenses = P10,000
d) Salary allocations to partners = P13,000
e) Interest paid to banks = P2,000
f) Partners’ withdrawals = P8,000

The partnership net income (loss) is:


A) 20,000
B) 18,000
C) 5,000
D) (3,000)

24. Lancelot is trying to decide whether to accept salary of P40,000 or a salary of 25,000 plus a bonus of 10%
net income after salary and bonus as a means of allocating profit among the partners. Salaries traceable to
the other partners are estimated to be 100,000. What amount of income would be necessary so that
Lancelot would consider the choices to be equal?
A) 165,000
B) 290,000
C) 265,000
D) 305,000

25. On January 1, 2013, Casio Realty Company sold property carried in inventory at a cost of P840,000 for
P1,400,000 . A 10% down payment was made and the balance payable in 4 equal installments of P363,625,
inclusive of 12% annual interest, payable semi-annually every June 30 and December 31.
Under installment method, how much is the total realized gross profit in 2013?
A) 293,332.60
B) 346,900
C) 308,000
D) 237,332.60

26. SPAIN, Inc., agrees to transfer television sets to Gasol Bros. on a consignment basis. The consignee is to
sell a set at 40% above cost exclusive of freight and is to receive a 10% commission on sales price. The
consignor agrees to reimburse the consignee for all expenses related to the consignment. The agreement
also calls for an advance payment by the consignee of 30% per set based on selling price; the said advance
is to be deducted as settlement is made for each set sold. The consignee is to provide an account sales
quarterly and is to make cash remittance for the amount owed at that time. The following consignment
sales activities occurred during the October 1 to December 31 of current year:
Sets shipped – 100; Unit cost each set – P 10,000; Freight charges on the shipment paid by the consignor –
P75,000; The consignee made advance payments on the sets received; Advertising cost paid by the
consignee – P 50,000 The consignee sold 80 sets for cash; expenses of delivery and installation were P
25,000. After notifying the consignor with the total sets sold for the period, the consignee returned 10 sets
representing a model that could not be sold and paid freight charges of P 8,000 on the return. The net
income to be reported by the consignor as a result of the above is
A) 65,000
B) 73,000
C) 125,000
D) 57,500

On May 1, Rubina Products Company ships five (5) of its appliances to EZ Company on consignment. The cost of the
appliances shipped is P155 per unit. The consignor paid shipping costs totalling P50.
Each unit is to be sold at P250 payable P50 in the month of purchase and P10 per month thereafter. The consignee is
entitled to 20% of all amounts collected on consignment sales.
EZ Company was able to sell 3 appliances in May and 1 in June. Regular monthly collections by the consignee, and
appropriate cash remittances have been made to the consignor at the end of each month.

27. The total amount remitted to consignor as of June is:


A) -0- C) P184
B) P64 D) P200

Page 3
28. The profit from the consignment is:
A) P294 C) P150
B) P140 D) None of these

29. The cost of inventory on consignment is:


A) P155 C) P245
B) P165 D) None of these

Cooper Construction Company had a contract starting April 2017, to construct a P9,000,000 building that is expected
to be completed in September 2019 at an estimated cost of P8,250,000. At the end of 2017, the costs to date were
P3,795,000 and the estimated total costs to complete had not changed. The progress billings during 2017 were
P1,800,000 and the cash collected during 2017 was P1,200,000.

30. For the year ended December 31, 2017, Cooper would recognize gross profit on the building of:
A) 316,250 C) 405,000
B) 345,000 D) -0-

31. At December 31, 2017, Cooper would report Construction in Progress in the amount of:
A) 345,000 C) 4,140,000
B) 3,795,000 D) 3,540,000

Cheek-In Builders, Inc. employs the cost-to-cost method in determining the percentage-of-completion for revenue
recognition. The company’s records show the following information on a recently completed project for a contract
price of P5,000,000.
2017 2018 2019
Costs incurred to date 900,000 2,550,000 ?
Gross profit (loss) 100,000 350,000 (50,000)

32. The estimated cost to complete the project at December 31, 2018 is
A) 850,000 C) 2,300,000
B) 1,700,000 D) 2,550,000

33. The actual costs incurred during the year 2019 is


A) 2,550,000 C) 2,200,000
B) 2,300,000 D) 2,050,000

34. LakeSide Bank holds a P100,000 note secured by a building owned by Fly-By-Night Manufacturing, which
has filed for bankruptcy. If the property has a book value of P120,000 and a fair market value of P90,000,
what is the best way to describe the note held by Second Bank and Trust Company? The bank has a (an)
A) Secured claim of P100,000
B) Unsecured claim of P100,000
C) Secured claim of P90,000 and an unsecured claim of P10,000
D) Secured claim of P100,000 and an unsecured claim of P20,000

35. Equipment with a book value of P120,000 is sold in a liquidation process for cash of P110,000. This
equipment was security for a P150,000 bank loan. Any remainder is considered unsecured. How would this
transaction be reported on the Statement of Realization and Liquidation?
A) A reductions in non-cash assets of P120,000
B) A loss reported on owner’s equity of P10,000
C) A disbursement of cash to the bank of P110,000, a reduction in partially secured liability of
P150,000 and an increase in unsecured without priority liability of P40,000
D) All of the above would occur

36. In a bankruptcy, which of the following statements is true?


A) An order for relief results only from voluntary petition
B) Creditors entering an involuntary petition must have debts totalling at least P20,000
C) Secured notes payable are considered liabilities with priority on a statement of affairs
D) None

Trial balance for the home office and the branch of Terry Company show the following accounts before adjustments
on December 31, 2017. The home office policy of billing the branch for merchandise is 20% above cost.
Home Office Branch
Allowance for overvaluation 60,000
Shipments to branch 240,000
Purchases (outsiders) 75,000
Shipments from home office 270,000
Merchandise Inventory, 12/01/17 100,000

The branch Merchandise Inventory on December 31, 2012 of P 50,000 includes purchases from outsiders of P
20,000.

37. The working paper entry to eliminate profit in the beginning inventory includes debit to
A) Allowance for overvaluation, P48,000
B) Branch income, P5,000
C) Merchandise Inventory (12/1/2017), P12,000
D) Allowance for overvaluation, P12,000

38. The entry on the books of the home office to recognize mark-up includes credit to
A) Allowance for overvaluation, P5,000
B) Branch income summary, P55,000
C) Branch income summary, P5,000
D) Branch income summary, P52,000

39. After paying all their liabilities, S, T and P had the following balances:

Page 4
Partner Capital Loans P&L Ratio
S P10,296 P9,000 12/25
T 8,904 3,000 8/25
P 6,810 3,990 5/25
Total 26,010 15,990

Cash available for distribution amounts to P3,780. Remaining assets of P38,220 will be realized piecemeal in the next
month. How much of the P3,780 cash should P receive?
A) 3,060 C) 720
B) 756 D) -0-

40. When the investment of a new partner exceeds the new partners’ initial capital balance and goodwill is not
recorded, who will receive the bonus?
A) The new partner
B) The old partners in their old profit and loss ratio
C) The old partners in their new profit and loss ratio
D) The old and new partners in their new profit and loss ratio

-END-

“Be a warrior, not a worrier.” -amcpa

Prepared by: Approved by:

ANNA MAE MAGBANUA, CPA DEAN ROSEMARIE CRUZ-ESPAÑOL, CPA


Instructor Dean, College of Business Administration

Page 5
HOME OFFICE AND BRANCH ACCOUNTING

PROBLEMS:

1. Cebu branch submitted the following data to its home office in Manila for 2016, its first year of operation:

Sales P 2,300,000
Shipments from home office 1,850,000
Operating expenses 235,000
Home Office 480,000

Shipments to the branch are billed at cost. The December 31 inventory of the branch was P255,500.

What is the balance of the Investment in Branch account on December 15, 2016 on the home office books?
a. P 950,500
b. P 470,500
c. P 950,000
d. P 480,000

SOLUTION:

Answer: A

Since the balance of the reciprocal accounts “Home Office” account and “Investment in Branch”
account are equal, then the balance of the Home Office account after closing the branch profit is to be
computed. The computation is:

Home Office account balance before branch profit P 480,000


Add: Profit (loss)
Sales P 2,300,000
Cost of sales:
Shipments from HO P 1,850,000
Inventory, Dec. 31 255,500 1,954,500
Gross profit P 705,500
Operating expenses 235,000 470,500
Home Office account balance, December 31, 2014 P 950,500

2. The home office in Quezon City ships and bill merchandise to its provincial branch at cost. The branch
carries its own accounts receivable and makes its own collections. The branch also pays its expenses.

The transactions for 2016 are reflected in the branch trial balance that follows:

Cash P 20,000
Accounts Receivable 80,000
Home Office P 180,000
Shipments from Home Office 250,000
Sales 225,500
Expenses 55,500

Total P 405,500 P 405,500

December 31, inventory P 65,000


No. 2 – Continued

Assuming all the transactions are properly recorded, what is the balance of the Investment in Branch account in the
home office books?
a. P 180,000
b. P 195,000
c. P 165,000
d. P 175,000

SOLUTION:

Answer: C

Home Office account before branch profit (loss) P 180,000


Add: Profit (loss)
Sales P 225,500
Cost of Sales:
Shipments from HO P 250,000
Inventory, 12/31 65,000 185,000
Gross profit P 40,500
Expenses 55,500 ( 15,000)
Home Office account balance, 12/31 P 165,000

Therefore the balance of the Investment in Branch account is also P 165,000.

3. The following data pertains to the shipments of merchandise from Home Office to Branch during 2016:

Home office’s cost of merchandise P 350,000`


Inter-office billings 420,000
Sales by branch to outsiders 520,000
Merchandise inventory on December 31, 2016 50,000

In the combined statement of comprehensive income of the Home Office and the Branch for the year ended
December 31,2016, what amount of the above transactions should be included as sales?
a. P 570,000
b. P 520,000
c. P 470,000
d. P 350,000

SOLUTION:

Answer: B

In the preparation of combined statements of the home office and the branch, all inter-office transactions are
eliminated as if it had never occurred. Therefore, the only transactions that should be presented are transactions to
outsiders, which is in this problem, the P 520,000 sales by branch to outsiders.
4. Nike Corporation operates a number of branches in the provinces. On December 31, 2016, its Davao branch
showed a Home Office account balance of P 54,700 and the home office books showed an Investment in Davao
Branch account balance of P 51,100. The following information they help in reconciling both accounts:

1. A P 24,000 shipment, charged by Home Office to Davao Branch, was actually sent to and retained by Cebu
Branch.
2. A P 30,000 shipment, intended and charged to Aklan Branch was shipped to Davao Branch and retained by the
latter.
3. A P 4,000 emergency cash transfer from Cebu Branch was not taken up in the Home Office books.
4. Home Office collects a Davao Branch accounts receivable of p 7,200 and fails to notify the branch.
5. Home Office was charged for P 2,400 for merchandise returned by Davao Branch on December 30. The
merchandise is in transit.

Home Office erroneously recorded Davao Branch’s net income for 2016 at P 32,550.

What is the adjusted balances of the Home Office and Davao Branch reciprocal accounts on December 31, 2016?
a. P 40,300
b. P 54,700
c. P 47,500
d. P 43,500

SOLUTION:

Answer: C

To compute the adjusted balances of the reciprocal accounts a reconciliation statement is to be prepared as follows:

(HO Books)
(Branch books) Investment in
Home Office Davao Branch
Account Account
Unadjusted balances, December 31, 2016 P 54, 700 P 51, 100
Add (deduct) the following adjustments
1. Shipment charged to Davao branch
but actually sent to Cebu branch ( 24,000)
2. Shipment charged to Aklan branch
but actually sent to Davao branch
3. No effect.
4. Home office collection of Davao Branch
accounts receivable ( 7,200)
5. Merchandise returned by Davao branch.
Still in transit to home office. ( 2,400)
6. Overstatement of Davao branch net
Net income (P 32,550 – P 25,350) ( 7,200)
Adjusted balances, December 31, 2016 P 47,500 P 47,500
5. The branch manager of Tower Cosmetics in Cebu submitted a report as of May 31, 2016 containing the following
information:

Petty Cash Fund P 1,500


Sales 198,720
Sales Returns 3,600
Accounts Wriiten Off 1,920
Shipments from Home Office 136,080
Accounts Receivable – May 31, 2015 43,800
Accounts Receivable – May 31, 2016 49,140
Inventory – May 31, 2015 37,170
Inventory – May 31, 2016 41,370
Expenses (reimbursed by H.O) 57,930

Assuming all cash collected by the branch is remitted to Tower Cosmetics home office, the remittances for the
period amounted to:
a. P 187,860
b. P 189,780
c. P 195,120
d. P 198,720

SOLUTION:

Answer: A

The P 187, 860 is computed as follows:

Accounts receivable, 5/31/2015 P 43, 800


Net Sales (P 198,720 – P 3,600) 195, 120
Total P 238, 920
Less: Accounts Receivable, 5/31/2016 P 49, 140
Accounts written off 1, 920 51, 060
Remittance P 187,860

6. On December 31, the Investment in Branch account in the home office books shows a balance of P 50,000. The
following facts are ascertained:
1. Merchandise billed at P 12,500 is in transit on December 31 from the home office to the branch.
2. The branch collected a home office accounts receivable for P 3,500. The branch did not notify the home
office of such collection.
3. On December 30, the home office sent cash of P 7,500 to the branch, but this was charged to General
Expense; the branch has not received the cash as of December 31.
4. Branch profit for December was recorded by the home office at P 2,400 instead of P 2,040.
5. The branch returned supplies of P 1,500 to the home office but the home office ha snot yet recorded the
receipt of the supplies.

Assume all the other transactions have been properly recorded.

What is the unadjusted balance of the Home Office account on the branch books on December 31?
a. P 64, 140
b. P 39, 140
c. P 14, 000
d. P 13, 000
SOLUTION:

Answer: B

P 39, 140 is computed as follows:

Investment in Branch account balance,12/31 P50,000


(Home Office books)
Add (Deduct):
Merchandise in transit (12,500)
Collection of Home Office accounts receivable by Branch 3,500
Erroneous recording of Branch profit ( 360)
Supplies returned by Branch ( 1,500)
Home Office account balance,12/31 (Branch books) P39,140

7. A reconciliiation of the Dagupan Branch account of Mandaluyong Company and the Home Office account carried
in the branch’s books shown the following discrepancies at December 31,2016.
1. A credit for merchandise allowance for P 300 was taken by the branch as P 360.
2. A charge by the branch of P 550 for an advance taken by the president when he visited the branch has
not yet been recorded by the home office.
3. The branch has not taken up P900 covered by a debit memo from the home office as share in advertising
expenses.

The Invesrment in Dagupan Branch account in the home office books had a debit balance of P 43,000 at December
31,2016. The reciprocal accounts were in agreement at the beginning of the year.

The unadjusted balance of the Home Office account in the branch’s books at December 31,2016 was:
a. P 43, 500
b. P 42, 950
c. P 41, 990
d. P 41, 490

SOLUTION:

Answer: D

The P 41, 490 unadjusted balance of Home Office account is computed as follows:

Unadjusted balance, Investment in Branch account, 12/31 P43,000


Less: Merchandise allowance (error) P 60
Branch advances to President 550
Advertising expense charged to Branch 900 1,510
Unadjusted balance, Home office account, 12/31 P41,490
8. The following were found in your examination of the interplant accounts between the Home Office and the
Butuan Branch:

a. Transfer of fixed assets from Home Office amounting to P53,960 was not booked by the branch.
b. P10,000 covering marketing expense of another branch was charged by Home Office to Butuan
c. Butuan recorded a debit note on inventory transfers from Home Office of P75,000 twice.
d. Home Office recorded a cash transfer of P65,700 from Butuan Branch as coming from Davao Branch
e. Butuan reversed a previous debit memo from Cagayan de Oro Branch amounting to P10,500. Home Office
decided that this charge is appropriately Davao Branch’s cost.
f. Butuan recorded a debit memo from Home Office of P4,650 as P4,560

The net adjustments DR (CR) to the Investment in Butuan Branch account to the Home Office account are:
Investment in Butuan Home Office
a. P (75,700) P20,950
b. 75,700 ( 20,950)
c. ( 55,700) 75,000
d. ( 65,700) ( 74,000)

SOLUTION:

Answer: A

Dr. (Cr.) Adjustment to Investment in Butuan Branch account


Marketing expense of another branch charged to Butuan (b) P(10,000)
Butuan’s remittance credited to Davao branch (d) ( 65,000)
Dr. (Cr.) adjustment to Butuan Branch
account in the home office books P(75,000)

Dr, (Cr.) Adjustment to Home Office account:


Fixed assets transfer not book by Butuan (a) P(53,960)
Inventory transfer recorded twice by Butuan (c) 75,000
Error in recording DM for P4,650 as P4,560 (f) ( 90)
Dr. (Cr.) adjustment to Home Office account
In Butuan Branch books P 20,950

9. After examining on a comparative basis the inter-office account of the Bulacan Company with its suburban
branch and the similar account carried on the latter’s books, the following discrepancies at the close of the business
on June 30, 2016 were seen:
a. A charge for labor by the Home Office P500 was recorded twice by the branch.
b. A charge of P895 was made by the Home Office for freight on merchandise, but the amount was recorded
by the Branch as P89.50.
c. A charge of P980 (furniture and fixture) on the Home Office books was taken up by the Branch as P890.
d. A credit by the Home Office for P350 (merchandise allowances) was taken up by the Branch as P400.
e. The Home Office charged the Branch P425 for interest on open account which the Branch failed to take up
in full; instead, the Branch sent to the Home Office a wrong adjusting memo, reducing the charge by P100
and set up a liability for the next amount.
f. The Home Office received P5,000, from the sale of a truck which it erroneously credited to the Branch; the
Branch did not change the Home Office therewith.
g. The Branch by mistake sent the Home Office a debit note for P370 representing its proportion of a bill for
repairs of truck; the Home Office did not record it.
h. The Branch inadvertently received a copy of the Home Office entry dated July 19, 2014 correcting item (f)
and entered a credit in favor of the Home Office as of June 30, 2016.
At June 30, 2016, the unadjusted balance of the Investment in Branch account on the Home Office books showed
P175,520. At the beginning of the year, the interoffice accounts were in balance.

What is the unadjusted balance of the Home Office account on the branch books on June 30, 2016?

a. P184,279.50
b. P160,725.50
c. P18,729.00
d. P165,279.50

SOLUTION:

Answer: A

Unadjusted balance of investment in branch account, 6/30 P175,520.00


(a) Charge for labor 500.00
(b) Charge for freight ( 805.50)
(c) Purchase of furniture and fixture ( 90.00)
(d) Merchandise allowance ( 50.00)
(e) Charge for interest ( 425.00)
(f) Proceeds from sales of truck 5,000.00
(g) Charge for truck repairs ( 370.00)
(h) Proceeds from sales of truck 5,000.00
Unadjusted balance of Home Office account, 6/30 P184,279.50

10. Rustans, Philippines has two merchandise outlets, its Home Office in Manila and its Cebu City branch. For
control purposes, all purchases are made by the Home Office and shipped to the Cebu City branch at cost plus 10%.
On January 1, 2016 the inventories of the Home Office in Manila and the Cebu City branch are P13,600 and P3,960
respectively. During 2016, the Home Office purchased merchandise costing P40,000 and shipped 40% of it to the
Cebu City branch. At December 31, 2016, the following journal entry to prepare the books for the next accounting
period was prepared by the branch”

Sales 32,000
Inventory, December 31 4,840
Inventory, January 1 3,960
Shipments from main store 17,600
Expenses 10,480
Home Office 4,800

What was the actual branch income for 2016 on a cost basis assuming the use of the provisions of the Statement of
Financial Accounting Standards?

a. P4,800
b. P6,320
c. P6,480
d. P6,840
SOLUTION:

Answer: B

Sales P32,000
Cost of sales:
Inventory, Jan. 1 3,960
Shipment from Home Office 17, 600
Inventory, Dec. 31 ( 4,840) 16,720
Gross Profit 15,280
Expenses 10,480
Net income per branch books 4,800
Add: Overvaluation of cost of sales
Billed Price (above) 16,720
Cost to H.O ( 16, 720 / 110%) 15,200 1,520
Actual branch income at cost basis P 6,320

11. On September 1, Star Company opened a branch in Dagupan City, shipping to it merchandise billed at P60,000.
During the month, additional shipments were made at a billed price of P24,000. Returns by the branch of bad-order
goods were credited for P1,680. At the end of the month, the branch reported its inventory of P33,600 and its net
loss for the month at P5,200 Shipments to and from the branch were consistently billed at 120% off cost.

a. P28,000 and P2,920, respectively


b. P28,000 and (P5,200), respectively
c. P33,600 and P2,920, respectively
d. P33,600 and P5,200, respectively

SOLUTION:

Answer: A

Branch Inventory at Cast:


Branch inventory at billed price P33,600
Divide by the billing percentage of cost 120%
Branch inventory of cost P 28,000

Branch net income as far as the Home office is concerned:

Branch net loss, as reported (P5,200)


Add: Overvaluation of Cost of Sales of the Branch:
Total Shipment to Branch:
Billed price (P60,000 + P24,000) P84,000
Cost (84,000 120%) 70,000 P14,000
Less: Branch returns –
Billed price P 1,680
Cost(P1,680 120%) 1,400 280
Net Shipment P13,720
Less: Inventory, 9/30-
Billed price P33,600
Cost 28,000 5,600 8,120
Branch Net Income P 2,92
12. Makati Company bills its Valenzuela Branch for merchandise at 140% of cost. At the end of January, 2016, the
branch reported the following information.

Merchandise from
Home Office
(At Billed Price)

Inventory, January 1 P 7,560


Shipments received 28,280
Inventory, January 31 8,400

What should be the balance of the allowance account for overvaluation of the branch inventory at
January 31 before adjustment?
a. P 2,400
b. P 2,160
c. P 9,080
d. P10,240

SOLUTION:

Answer: D

The balance of the Allowance for Overvaluation of Branch Inventory account represents the overvaluation of branch
inventory on January 1 and overvaluation of the shipment received. Computation is as follows:

Billed Billing Over-


Price + Percentage = Cost Valuation

Inventory. January 1 P 7,560 140% P 5,400 P2,160


Add: Shipment 28,280 140% 20,200 8,080

Balance of allowance before adjustment P 10,240

13. The Binondo branch of China Products Inc. buys merchandise from third parties and receives
merchandise from the home office for which it is billed at 20% above cost. Below are excepts from the
trial balances and data on the home office and Binondo branch for the month just eneded.

Home Office
Allowance for overvaluation of branch merchandise P370,000
Shipments to Branch 850,000

Branch
Beginning inventory 1,440,000
Shipments from home office 1,020,000
Purchases 410,000

Month end additional data:


Ending inventory of branch P1,460,000
From home office at Billed Price P1,170,000
From outsiders (at cost) 290,000

The total cost of goods sold of the Binondo branch at cost (net of overvaluation) for the month just ended
amounted to.

a. P1,410,000
b. P1,385,000
c. P1,235,000
d. P1,850,000

SOLUTION:

Answer: C

Beginning inventory P1,440,000


Purchase 410,000
Shipment from home office 1,020,000
Goods available for sale 2,870,000
Ending inventory 1,460,000
Cost of sales 1,410,000
Less: Overvaluation
Beginnning inventory and shipments 370,000
Less:Ending inventory
Billed price P1,170,000
Cost(P1,170,000/120%) 975,000 195,000 ` 175,000
Cost of goods sold (net) P 1,235,000

14. Shopper Company started a branch office in Iloilo City on June 1,2016. On this date, the company shipped
to its Branch merchandise billed at P90,000. On June 15, another shipment was made at billed prices of
P36,000. During the month, the branch was credited for P2,520 for the damaged goods returned by the
branch. On June 30,2016, branch reported the following:

Inventory, June 30 P50,400


Net loss for the month (P7,800)

Shipments to and from the branch wee uniformly billed at 120% of cost.

a. No net income or loss


b. Net income of P4,380
c. Net income of P12,180
d. Net loss of P7,800
SOLUTION:

Answer: B

According to the Home Office books, Iloilo branch will have a P4,30 net income as computed below:

Branch net loss (P7,800)


Add: Overvaluation of Cost of Sales of branch –
Total shipment to Branch:
Billed price(90,000+P36,000) P126,000
Cost (P126,000 120%) 105,000 P21,000

Less: Branch returns


Billed price P 2,520
Cost (P2,520 120%) 2,100 420
Net Shipment to Branch P 20,580
Less: Inventory, 6/30
Billed price P 50,400
Cost (P50,400 120%) 42,000 8,400 12,180
Branch Net Income P 4,380

15. Tarlac branch of Quezon City Company, at the end of its first quarter of operations, submitted the following
statement of comprehensive income.

Sales P300,000
Cost of sales:
Shipments from Home Office P28,000
Local purchases 30,000
Total P 310,000
Inventory at end 50,000 P 260,000
Gross margin on sales P 40,000
Expenses 35,000
Comprehensive income P 5,000

Shipments to the branch wee billed at 140% of cost. The branch inventory as at September 30 amounted to P50,000
of which P6,600 was locally purchased, Markup on local purchases, 20% over cost. Branch expenses incurred by
Head office amounted to P2,500

On September 30, the branch inventory at cost and the net income realized by the home office from the Tarlac
branch operation are:

Branch inventory at cost Net income realized


a. P37,600 P72,600
b. P50,000 P55,000
c. P31,600 P 5,000
d. 37,600 P70,100
SOLUTION:

Answer: D

P37,600 is computed as follows:

Acquired from Home Office:


Billed price (P50,000-P6,600 P43,400
Divide by billing percentage of cost 140% P 31,000
Local purchases 6,600
Branch inventory at cost, 9/30 P 37,600

Below is the computation of Home Office income from branch operation of P70,100.
Branch net income (P5,000 – P2,500 expense) `` P 2,500
Add: Overvaluation of Branch Cost of Sales:
Shipment from Home Office:
Billed price P280,000
Cost(P280,000 + 140%) 200,000 P80,000
Less: Inventory, end-
Billed price (P50,00-P6,600) P43,400
Cost (43,400 + 140%) 31,000 12,400 67,600
Branch net income realized by Home Office P 70,100

16. Ayala Branch was billed by Home Office for merchandise at 140% of cost. At the end of its first month, Ayala
Branch submitted among other things, the following data:

Merchandise from Home from Home Office (at billed price) P 98,000
Merchandise purchased locally by Branch P55,000
Iventory, December 31 of which P7,000 are local purchase 28,000
Net sales for month 180,000

SOLUTION:

Answer: B

Branch inventory, at cost, 12/31:


Acquired from Home Office (P21,000 / 140%) P 15,000
Local purchases 7,000
Total P 22,000

Branch Gross Profit:


Net sales P 180,000
Cost of sales insofar as Home Office is concerned:
Shipment from Home Office, at cost
(P98,000 / 140%) P70,000
Purchases 40,000
Cost of goods available for sale 110,000
Inventory, at cost, 12/31:
Acquired from Home Office
(21,000 140%) P 15,000
Local purchases 7,000 22,000 88,000
Gross profit insofar as Home Office is concerned P 92,000
17. The Coffee Blend Corporation decided to open a branch in Manila. Shipments of merchandise to the branch
totaled P54,000 which included a 20% mark-up on cost. All accounting records are to be kept at the home office.

The branch submitted the following report summarizing its operation for the period ended December 31, 2016.

Sales on account P74,000


Sales on cash basic 22,000
Collections of account 60,000
Expenses paid 38,000
Expenses unpaid 12,000
Purchase of merchandise for cash 26,000
Inventory on hand, December 31; 80% from home office 30,000
Remittance to home office 55,000

The branch 12/31 inventory at cost and the branch net income (loss) as far as the home office is concerned are:

Branch Branch
Inventory at Cost Net income (loss)
a. P26,000 (P1,000)
b. P25,000 (P4,000)
c. P26,000 P1,000
d. P20,000 P 800

SOLUTION:

Answer: C

Below is the computation of Branch ending Inventory at cost:

Acquired from Home Office (80% x P30,000) / 120% P 20,000


Add: Acquired from outsiders (20% x P30,000) 6,000
Branch inventory at cost, 12/31 P 26,000

The P1,000 net income is derived as follows:

Sales (P74,000 + P22,000) P 96,000


Cost of sales insofar as Home Office is concerned:
Shipment from Home Office at cost
(54,000 / 120%) P45,000
Purchases 26,000
Cost of goods available for sale 71,000
Inventory, at cost, 12/31 26,000 45,000
Gross profit P 51,000
Expenses (38,000+ P12,000) 50,000
Branch net income insofar as Home Office is concerned P 1,000
18. Trial balances before adjustments for the home office and the branch of the King Company show the following
items on December 31. The home office bills the branch at 20% above cost.

Home Office Branch

Allowance for overvaluation of branch merchandise P 3,600


Shipment to branch 8,000
Purchases P2,500
Shipment from home office 9,600
Merchandise inventory, December 1 15,000

What part of the branch inventory as of December 1 represented purchases from outsiders?
a. P3,000
b. P5,000
c. P2,000
d. P1,800

SOLUTION:

Answer: A

Merchandise Inventory, December 1 P 15,000


Less: Merchandise acquired from Home Office at Billed Price
Overvaluation (3,600 – P 1,600) P 2,000
Cost (P2,000 / 20%) 10,000 12,000
Merchandise acquired from outsiders P 3,000

19. The Manila Sales Co. established a branch in San Pablo City early last year, it shipped merchandise and billed
the branch for P300,000 prior to its opening. For the year, it made additional shipments at billed price of P120,000.
Within the year the branch shipped back P7,500 inventory and got the credit memo for said returns. On the last
working day of the year, an inventory count was made. Ending inventory of P1855,000 was established consisting of
purchases from third parties at P20,000, with the balance coming from home office shipments at billed price. The
home office billed the branch at 20% above cost. The total purchases of the branch from outside suppliers amounted
to P72,6500. The total goods available for sale by the branch at cost (net of overvaluation and returns amounted to:

a. P416,250
b. P485,000
c. P422,500
d. P435,250
SOLUTION:

Answer: A

Total shipment from office P 420,000


Returns ( 7,500)
Purchases 72,500
Good available for sale, at Billed Price P 485,000
Less: Overvaluation of shipment:
Billed Price P420,000
Cost (P420,000 / 120) 350,000 70,000
Returns:
Billed Price P 7,500
Cost (7,500 / 120%) 6,250 ( 1,250) 68,750
Goods available for sale, at Cost P 416,250

20. The income statement submitted by the Bulacan Branch to the Home Office for the month of December,2016 is
shown below. After effecting the necessary adjustments the true net income of the Branch was ascertained to be
P156,000. The Branch inventories were:

12/01/2016 12/31/2016
Merchandise from home office P70,000 P 84,000
Local purchases 10,000 16,000
Total P 80,000 P 100,000

Sales P 600,000
Cost of sales:
Inventory, December 1 P 80,000
Shipments from home office 350,000
Local purchases 30,000
Total available for sale P 460,000
Inventory, December 31 100,000 360,000
Gross margin P 240,000
Operating expenses 180,000
Total comprehensive income for December 2016 P 60,000

What is the balance of the “Allowance for Overvaluation in Branch Invetory” account at December 31, 2016?

a. P10,000
b. P16,000
c. P24,000
d. P34,000
SOLUTION:

Answer: C

Before computing the balance of the allowance account, the percent of billing price to cost should be computed first
as follows:

Branch net income, per Home Office P 156,000


Branch net income, per Branch 60,000
Realized mark-up on merchandise from the
Home Office already sold by the Branch P 96,000

Shipments from Home Office 350,000


Less: Increase is portion of Branch inventory
Acquired from Home Office 14,000
Portion already sold by Branch 336,000
Less Mark-up Thereon (above) 96,000
Cost of portion already sold by Branch P 240,000

Percent of billing price to cost P336,000/240,000 140%

The balance of the “Allowance for Overvaluation in Branch inventory” accounted December 31, 2016 after
adjustment represent the overvaluation of the branch ending inventory acquired from the Home Office computed as
follows:

Billed price P 84,000


Cost (P84,000 / 140%) 60,000
Balance of the allowance account P 24,000

21. Mahiyain Commercial Corporation operates a branch in Iloilo City. Selected accounts taken from the books of
Mahiyain and its branch show balances as of December 31,2016 as follows:

Home Office Branch

Merchandise inventory, January 1 P 12,000 P 8,000


Purchase 150,000 30,000
Shipments from home office - 93,750
Shipments to branch 75,000 -
Branch inventory allowance 19,750 -
Sales 115,000 176,500
Merchandise inventory, December 31 14,000 10,350

The ending inventory of the branch includes items costing P4,350 which were acquired from suppliers other than the
home office.

As far as the home office is concerned, the cost of sales of the Iloilo City branch was:

a. P 97,120
b. P102,850
c. P121,400
d. P131,850
SOLUTION:

Answer: B

Branch inventory, January 1 P 8,000


Purchases 30,000
Shipments from home office 93,750
Merchandise available for sale P 131,750
Less: Branch inventory, Dec31 10,350
Branch cost of sales, per branch books P 121,400
Less: Mark-up on merchandise from the home office
already sold by the branch:
Branch inventory allowance P 19,750
Less: Mark-up on portion of Dec.31 inventory
acquired from home office:
(P10,350 – P4,350) x 25/125 1,200 18,550
Branch cost of sales, as far as the home office is concerned P 102,850

Note: Shipments of merchandise from the home office to the branch are billed as 125% of cost, determined as
follows:

Shipments from Home Office = P93,750


= 140%
Shipments to Branch = P75,000

22. The Neneng Corporation established its San Pedro branch in March 2016. During the first year of operations, the
home office shipped to the branch merchandise which had cost P120,000. Three-fourths of these merchandise was
sold by the branch for P141,000. Operating expenses of the branch amounted to P27,000.

How much total comprehensive income will the branch report if merchandise is billed by the home office to the
branch at 25% above cost?
a. P 800
b. P 1,200
c. P 1,500
d. P 8,000

SOLUTION:

Answer: C

Sales P 141,000
Less: Cost of sales at Billed Price (sch.1) 112,500
Gross profit 28,500
Expenses 27,000
Total comprehensive income to be reported by the Branch P 1,500

Schedule 1
Cost of shipment to branch P 120,000
Add:25% mark-up 30,000
Billed price of shipment to branch 150,000
Portion sold x 3/4
Cost of sales at billed price P 112,500
23. A branch store in Marikina was established by Marco Co. on March 1. Shipments of merchandise, billed to this
branch at 125% of cost, were as follows:

March 5 P 120,000
March 10 50,000
March 20 35,000

On March 24, the branch returned defective merchandise worth P3,050 and on March 31, it reported a net loss of
P6,200 and merchandise inventory of P85,000.

In the home office books, the branch total comprehensive income (loss) is:

a. P (6,200)
b. P 17.190
c. P 20,240
d. P 23,390

SOLUTION:

Answer: B

Reported branch loss P ( 6, 200)


Add: Overvaluation in branch cost of sales
Shipment to branch P 205,000
Less: Returns 3,050
Ending inventory 85,000 88,050
Cost of sales, at Billed Price 116,950
Cost of sales, at Billed cost to Home Office
(116,950/125%) 93,560 23,390
Branch total comprehensive income, per Home Office books P 17,190

24. The Chivas Regal owns the Royal Crown in Quezon City and a branch in Davao City. During 2016, the home
office shipped to the branch supplies costing P120,000 at a billed price of 20% above cost. The inventories of
supplies at the branch were as follows: January 1, 2016, P90,000; December 31,2016, P108,000. On December 31,
2016 the home office holds inventories of P160,500 which includes P10,500 held on consignment.

How much is the inventories in a combined statement of financial position as of December 31, 2016?

a. P210,000
b. P240,000
c. P270,000
d. P300,000

SOLUTION:

Answer: B

The combined inventories on Dec,31 2016 statement of financial position is computed as follows:
Home Office (P160,500 – P10,500) P 150,000
Branch, at cost (P108,000 / 120%) 90,000
Combined inventories, 12/31 P 240,000
25. The Iloilo Company operates a branch in Davao, and the profit and loss data for the home office and the branch
for 2016 follows:
Home office Branch

Sales P250,000 P 75,000


Purchases from outsiders 200,000 15,000
Shipments to branch
Cost to home office 30,000
Billing price to branch 37,500
Expenses 40,000 10,000
Inventories Jan, 1,2016
Home office, at cost 80,000
Branch:
From outsiders, at cost 7,500
From Home Office at 20% above cost 24,000
Inventories, Dec. 31, 2016
Home office, at cost 55,000
Branch:
From outsiders, at cost 5,500
From Home office at 2016 billing 26,000

The combined total comprehensive income (loss) of the home office and the branch on December 31, 2016 is:
a. P30,800
b. P(33,800)
c. P33,800
d. P27,000

SOLUTION:

Answer: C

Sales P 325,000
Less: Cost of sales
Jan. 1 inventories, at cost (sc.1) P 107,500
Purchases 215,000
Merchandise available for sale P 322,500
Less: Dec, 31 inventories, at cost (sch.1) 81,300 241,200
Gross profit on sales P 83,800
Less: Expenses 50,000
Total comprehensive income P 33,800

Schedules 1:
Inventories
Jan. 1 Dec. 31
Home Office P 80,000 55,000
Branch, at cost
Acquired from outsiders 7,500 5,500
Acquired from Home Office
Jan. 1 (P 24,000 / 120%) 20,000
Dec. 31 (P 26,000 / 125%) 20,800
Combined P 107,500 P 81,300
2016 Billing (7,500 / 30,00) = 140%
26. Manila Inc. established a branch in Cebu to distribute part of the goods purchased by the home office. The home
office prices inventory shipped to the branch at 20% above cost. The following account balances were taken form
the ledger maintained by the home office and the branch:

Manila Inc, Cebu Branch

Sales P 600,000 P 210,000


Beginning inventory 120,000 60,000
Purchases 500,000 -
Shipment to branch 130,000 -
Shipment from home office - 156,000
Operating expenses 72,000 36,000
Ending inventory 98,000 48,000

All of the branch inventory is acquired from the home office –

The combined total comprehensive income of the home office and the branch is:

a. P 170,000
b. P 70,000
c. P 278,000
d. P 132,000

SOLUTION:

Answer: A

Sales
Cost of Sales:
Beginning inventory:
Home Office P 120,000
Branch, at cost (P60,000/120%) 50,000 P 170,000
Purchases 500,000
Total 670,000
Ending inventory:
Home Office 98,000
Branch, at cost (P48,000/120%) 40,000 138,000 532,000
Gross profit 278,000
Operating expenses 108,000
Combined total comprehensive income P 170,000
27. Selected accounts from the December 31, 2016 trial balances of Heart Co. and its branch follow:
Heart Branch
Inventory, Jan. 1 P 46,000 -
Investment in Branch 116,600 -
Purchases 380,000 -
Shipments from home office - 209,000
Freight in - 10,450
Expenses 104,000 58,100
Home Office - (106,600)
Sales (310,000) (280,000)
Shipments to branch (200,000) -
Branch merchandise markup (22,000) -

As of December 31, 2016, a shipment with a billing price of P11,000 was in transit to the branch. Freight cost,
typically 5% of the billing price, is inventoriable. Merchandise on hand at year-end wee” at home office, P64,000 at
cost; at branch, P33,000 at billing price.

What is the combined total comprehensive income of Heart Company and its branch for 2016?

a. P 77,000
b. P 84,900
c. P 76,000
d. P 76,100

SOLUTION:

Answer: C

Sales (P310,000 + P280,000) P 590,000


Cost of sales
Inventory, 1 /1 (Schedule 1) P 67,100
Purchases P 380,000
Freight in (P220,000 x 5%) 11,000 391,000
Goods available for sale P 458,100
Inventory, 12/31 (schedule 1} P 104,000
Freight in (P 44,000 x 5%) 2,000 106,200 351,900
Gross profit P 238,100
Expenses (P 104,00 + P 58,100) 162,000
Combined total comprehensive income P 76,000

Schedule 1: Combined inventories – at cost:


Inventories
January 1 December 31
Home office, at cost P 46,000 P 64,000

Branch, at cost:
Inventory, January 1:
Billed price P 23,100
Mark-up (schedule 2) 2,000 21,000
Inventory, December 31:
At cost [(P33,000 + P11,00) / 110 %] 40,000
Combined P 67,100 P 104,000
*Billing % (P 209,000 + P 11,000) / P200,200 = 110%

Schedule 2: Mark-up on Branch beginning inventory

Branch merchandise markup before adjustment P 22,000


Less: overvaluation of shipments [(P 209,000 + P 11,000) – P 200,000] 20,000
Mark up of branch beginning inventory P 2,000

28. Apo Supply Company is engaged in merchandising both at its Home Office in Makati and at its Branch in
Davao City. Selected accounts taken from the trial balances of the Home Office and the branch as of December 31,
2016 follow:
Makati Branch
Debits

Inventory, January 1, 2016 P 23,000 P 11,550


Davao Branch P 58,300 -
Purchases 190,000 105,000
Freight in from Home Office - 5,500
Sundry expenses 52,000 28,000

Credits

Home Office -P - P 53,300


Sales 155,000 140,000
Sales to Branch 110,000 -
Allowance for Overvaluation of
Branch Inventory at Jan. 1, 2016 1,000 -

Additional information:

- The Davao City branch gets all of its merchandise from the home office. The home office bills the goods at
cost plus a 10% mark-up, At December 31,2016, a shipment with billed value of P5,000 was still in transit.
Freight on this shipment was P250 and is to be treated as part of the inventory.

- Inventories eon December 31, 2016, excluding the shipment in transit, follow:
Home Office, at cost P 30,000
Branch, at billed price (excluding freight of P520) 10,400

What is the combined total comprehensive income (loss) of the home office and the branch on December 31, 2016?

a. P 30,470
b. P 20,870
c. P (10,000)
d. P (30,470)
SOLUTION:
Answer: A

Sales P 295,000
Cost sales:
Inventory, 1/1:
Home Office P 23,000
Branch, at cost (11,550 – 1,000) 10,550
Freight-in (5,500 + 250) 5,750 39,300
Purchases, Home Office 190,000
Total 229,300
Inventory 12/31:
Home Office P 30,000
Branch, at cost [(10,400 + 5,000)
/110%] 14,000
Freight- in (P 520 + P250) 770 44,770 84,330
Gross profit 110,470
Sundry Expenses 30,000
Combined total comprehensive income P 30,470

29. On November 2, 2016, the home office of Toby Sport Company recorded a shipment of merchandise to its
Bulacan Branch as follows:
Investment in branch – Bulacan 60,000
Shipments to Branch 50,000
Allowance for overvaluation of
Branch inventory 8,000
Cash (for freight charges) 2,000

The Bulacan branch sells 40% of the merchandise to outside customers during the rest of the period. The books of
the home office are closed on December 31 of each year.

On January 10,2017, the Bulacan branch transfer half of the original shipment to the Baguio branch, and the Bulacan
branch pays P1,000 freight for the shipment. If the shipment had been made by the home office to Baguio Branch,
the freight charges would have been P1,500.

What is the entry of the Bulacan branch to record the receipt of the shipment from the home office on November 2,
2016?
a. Shipments from home office 50,000
Accounts receivable 8,000
Freight in 2,000
Home office 60,000

b. Shipment from home office 60,000


Home office 60,000

c. Shipment from home office 58,000


Freight in 2,000
Home office 52,000

d. Shipment from home office 50,000


Freight out 2,000
Home office 52,000
SOLUTION:

Answer: C

Choice ( c ) is correct, because the branch should record the shipment from office at billed price (P50,000 +
P8,000), and should treat the freight changed to the office as inventoriable cost.

30. Using The same data in No. 29, at what amount should the 60% of the merchandise remaining unsold at
December 31, 2016 be included in the inventory of the Bulacan Branch?

a. P31,200
b. P36,000
c. P36,800
d. P34,800

SOLUTION:

Answer: B

Shipments from home office at billed price P 58,000


Unsold 60%
Ending inventory P 34,800
Freight in (P2,000 x 60%) 1,200
Total P 36,000

31. Using The same data in No.29, what is the entry in the books of Bulacan branch to record the transfer
on January 10,2017?

a. Baguio branch 31,000


Shipment from home office 31,000

b. Home office 31,000


Inventory 31,000
c. Home office 31,000
Inventory 30,000
Cash 1,000

d. Home office 31,000


Cash 1,000
Inventory 30,000

SOLUTION:

Answer: D

In the books of Bulacan branch (sending branch) the inter-branch transfer should be treated as if it was returned to
the home office. Inventory account should be treated as if it was returned to the home office. Inventory account
should be credited in place of the Shipment from Home Office account and freight-in which was already closed at
the end 2013. Therefore entry (d) is correct (P29,000 + P2,0000).
32. Using the same data in No. 29, what is the entry in the books of baguio branch to record the transfer on
January 10,2017?

a. Shipments from Bulacan Branch 30,200


Bulacan branch 30,200

b. Shipments from home office 29,000


Freight in 1,500
Home office 30,500
Cash 1,000

c. Shipments from home office 29,000


Freight in 1,500
Home Office 30,500

d. Shipments from home office 30,000


Freight in 1,000
Home office 31,000

SOLUTION:

Answer: C

In the books of Baguio branch (receiving branch) the inter-branch transfer should be treated as if it was received
from the home office. And the freight to be recognized should be the freight from the office. Therefore choice ( c ) is
correct.

33. Using the same data in No. 29, what is the entry in the home office books to record the inter-branch transfer on
January 10,2017?

a. Investment in branch – Baguio 30,500


Excess freight 1,500
Investment in branch Bulacan 32,000

b. Investments in branch – Baguio 30,500


Investments in branch – Bulacan 30,500

c. Investment in Branch – Bulacan 32,500


Investment in branch – Baguio 32,500

d. Investment in branch – Baguio 30,500


Excess freight 500
Investment in branch – Bulacan 31,000
SOLUTION:

Answer: A

In the books of the home office the inter-branch transfer can be cleared by debiting the receiving branch (Baguio)
and crediting the sending branch (Bulacan). Excess Freight account should be changed for the difference which is
treated as an expense of the home office. Therefore choice (a) is correct

Alternative entry: If the Allowance for Overvaluation of Branch Inventory account is classified by branch:

Investment in Branch – Baguio 30,500


Allowance for Overvaluation of Branch Inventory
Bulacan (P 8,000 x 50%) 4,000
Excess freight 1,500
Ivestment in Branch – Bulacan 32,000
Allowance for Overvaluation Branch
Inventory – Baguio 4,000

34. Papa. Inc. of Makati opens a sales agency in Pasig City and a working fund of P100,000 is established on
imprest basis. The first payment from the fund is P5,000 for rent of the store space.

What is the entry in the books of the home office to record the payment of rent by the agency?

a. Rent expense – Pasig agency 5,000


Cash 5,000

b. Pasig agency 5,000


Cash 5,000

c. Rent expense – Pasig agency 5,000


Working fund 5,000

e. No entry.

SOLUTION:

Answer: D

The expenses paid by the branch are not recorded in the home office book. It is only recognized upon replenishment
of the working fund (pretty cash fund).
35. Mama, Inc. opened a sales agency in San Pedro Laguna in 2016. The following is a summary of the transactions
of the sales agency:

Sale orders sent to Home office P 120,000


Sales orders filled by home office in 2016 95,000
Freight on shipment of agency 2,000
Collections, net of 10% discount 81,000
Selling expenses paid from the agency working fund 5,500
Administrative expenses charged to agency 5% gross sales
Samples shipped to agency:
Cost 8,200
Inventory, December 31, 2016 4,550

The company’s gross profit rate on agency sales is 30% excluding the freight cost on shipments yo agency.

What is the total comprehensive income of the agency for 2016?


a. P 3,600
b. P 5,600
c. P 1,600
d. P 6,300

SOLUTION:

Answer: A

Sales P 95,000
Sales discount (P 81,000 / 90%) x 10% 9,000
Net sales 86,000
Cost of sales (P95,000 x 70% ) + 200 68,500
Gross profit 17,500
Expenses:
Selling expenses P 5,500
Administrative expenses (P 95,000 x 5%) 4,750
Samples expenses (P 8,200- P 4,550) 3,650 13,900
Net income P 3,600

36. A Makati home office transfers inventory to its Pasig Branch at 140% of cost. During 2016, the reciprocal
account in the statement of comprehensive income of the home office amounts to P 328,125. On December 31,2016,
the home office adjusted the branch income summary by debiting the Allowance for Overvaluationof Branch
inventory account in the amount of P 81,250. The branch’s statement of financial position at the beginning of the
year shows P 105,000 of inventory acquired from the home office.

How much is the ending inventory of the branch per books?


a. P 200,000
b. P 161,250
c. P 280,000
d. P 80,000
SOLUTION:

Answer: C

Branch beginning inventory — acquired from home office P 105,000


Shipment from home office — at billed (P 328,125 x 140%) 459,375
Goods available for sale at billed price 564,375
Cost of sales (P 81,250 / 40%) x 140% (284,375)
Branch ending inventory per books P 280,000

37. On July 31, 2016, the home office in Manila establishes a sales agency in Bulacan. The following assets are
sent to the agency:
Cash (working fund to be operated under the imprest system) P 22,000
Samples of merchandise 36,000

During the month of August, the following transactions occured:


 The sales agency submits sales order of P 272,000 sales per invoice was billed at P 268,000. Cost of sales
to customers is P 124,000.
 Collections during the month amount to P 58,200, net of 3% discount.
 Home office disbursements chargeable to the agency are as follows:
Furniture P 40,000
Salaries for the month 21,600
Annual rent of office space 36,000
 On August 31, the sales agency working fund is replenished. Paid vouchers submitted by the sales agency
amounting to P 17,925. Samples are useful until December 31, 2016 which, at this time, are believed to
have a salvage value of 15% cost. Furniture is deprecited at 18% per annum.

What is the total comprehensive income of the sales agency for the month of August?
a. P 91,425
b. P 93,225
c. P 92,955
d. P 58,425

SOLUTION:

Answer: C

Sales P 268,000
Sales discount (P 58,200/ 97%) x 3% 1,800
Net sales 366,200
Cost and expenses:
Cost of sales P 124,000
Salaries 21,600
Rent expense (P36,000 x 1/12) 3,000
Expenses 17,925
Samples (P 36,000 x 85%) x 1/5 6,120
Depreciation (P 40,000 x 18% x 1/12) 600 P 173,245
Net income P 92,955
38. The home office in Makati shipped merchandise costing P 55,500 to Pasig branch, prepaid the freight
amounting to P 4,200. The home office transfers inventory to the branch at a 20% mark up above cost. Pasig branch
was subsequently instructed by the home office to transfer the merchandise to Alabang branch wherein the latter
paid freight of P 2,800. If the shipment was made directly from Makati to Alabang, the freight cost would have been
P 6,200.

Which of the following is true as a result of the interbranch transfer of merchandise?


a. The home office debits Alabang Branch Current for P 73,600
b. Alabang branch debits the Home Office for P 70,000
c. Pasig branch credits freight in for P 6,200.
d. The home office will credit Pasig Branch Current for P 70,800

SOLUTION:

Answer: D

Choice (d) is correct due to the following entries to record the interbranch transfer of merchandise:

Pasig Branch Books:


Home Office 70,800
Freight in 4,200
Shipment from home office 66,600
To record transfer of merchandise to Alabang

Alabang Branch Books:


Shipment from home office 66,600
Freight in 6,200
Cash 2,800
Home office 70,000
To record receipt of merchandise from Pasig

Home Office Books:


Alabang branch current 70,000
Excess freight 800
Pasig branch current 70,800
To record interbranch transfer of merchandise.

39. The following are some of the account balances on the books of the home office and its branch on December 31,
2016:
Home Office Books Branch Books
Inventory, January 1,2016 P 20,000 P 58,000
Shipments from home office 150,000
Purchases 900,000 200,000
Shipments to branch 145,000
Allowance for overvaluation of branch inventory 52,500
Sales 1,200,000 720,000
Operating expenses 290,000 110,000

Per physical count, the ending inventory of the is P 42,000 including goods purchased from outsiders of P 27,700
while the ending inventory of the home office is P 120,000. Home office bills its branch for merchandise shipments
at 30% above cost.
What is the amount of the unrealized inventory profit in the books of the home office on December 31,2016?
a. P 9,000
b. P 7,600
c. P 12,000
d. P 3,300

SOLUTION:

Answer: C

The unrealized inventory profit balance on December 31 is the difference between the branch ending inventory at
billed price and cost. Computed as follows:

Branch ending invty. per physical count — from HO (P42,000 — P 27,700) P 14,300
Shipment to transit:
Shipment from HO at BP (P 145,000 / 130%) P 188,500
Shipment from HO per books 150,000 37,700
Correct branch ending inventory at billed price P 52,000
Branch ending at cost (P52,000 / 130%) 40,000
Unrealized inventory profit, December 31, 2011 P 12,000

40. Using the data in No. 39, how much is the combined total comprehensive income on December 31,2016?

a. P 538 700
b. P 547,400
c. P 541,700
d. P 498,200

SOLUTION:

Answer: A

The combined net income is computed by preparing a combined income statement as follows:

Sales P 1,920,000
Cost of sales:
Inventory, January 1 (Sch.1) P 69,000
Purchases 1,100,000
Goods available 1,169,000
Inventory, December 31 (Sch.1) 187,700 981,300
Gross Profit 938,700
Expenses 400,000
Combined net income P 538,700

Schedule 1:
Inventory at cost
January 1 December 31
Home office P 20,000 P 120,000
Branch: Acquired from HO (Sch.2) 30,000 40,000
Acquired from outsiders (P 58,000 – P 39,000) 19,000 27,700
Total 49,000 67,700
Combined P 69,000 P 187,700
Schedule 2:

Allow for overvaluation before adjustment P 52,800


Overvaluation in the Shipments:
Shipment from HO at BP (P 145,000 x 130%) P 188,500
Shipment to branch at cost 145,000 43,500
Overvaluation in the branch beginning inventory P 9,000

Branch beginning inventory at cost (P 9,000 / 30%) P 30,000

Branch ending inventory at cost (per No. 39) P 40,000


ST. PAUL UNIVERSITY SURIGAO
Cor. San Nicolas & Rizal Streets
8400 Surigao City, Philippines

COLLEGE OF BUSINESS & TECHNOLOGY


MOCK BOARD EXAMINATION

Subject: ESSENTIALS OF ACCOUNTING

Name: Date:
INSTRUCTION: Read the questions carefully. Use the bubble sheet and shade the circle that corresponds to
your answer for each question. Avoid erasures.

1. The following July information is for Marley Company:


Standards:
Material 3.0 feet per unit @ P4.20 per foot
Labor 2.5 hours per unit @ P7.50 per hour

Actual:
Production 2,750 units produced during the month
Material 8,700 feet used; 9,000 feet purchased @ P4.50 per foot
Labor 7,000 direct labor hours @ P7.90 per hour

What is the material price variance (calculated at point of purchase)?


A. P2,700 U B. P2,700 F C. P2,610 F D. P2,610 U

2. Mango Inc. acquired on January 1, 2019 all the issued and outstanding common shares of Celine Inc. for
P310,000 and Celine Inc. is dissolved. On this day, the assets and liabilities of Celine Inc. show:
Cash P 30,000
Merchandise inventory 90,000
Plant and equipment 160,000
Goodwill 50,000
Liabilities ( 60,000)
Per appraisal, plant and equipment and merchandise inventory were valued at P190,000 and P75,000,
respectively. What is the amount of goodwill resulting from this transaction?
A. P125,000 B. P 40,000 C. P 75,000 D. P 90,000

3. On May 1, 2018, Jose and Maria formed a partnership and agreed to share profits and losses in the ratio
of 3 :7, respectively. Jose contributed a computer that cost him P50,000. Maria contributed P200,000
cash. The computer was sold for P75,000 on May 1, 2018 immediately after the formation of the
partnership. What amount should be recorded in Jose's capital account information of the partnership?
A. P55,000 B. P51,500 C. P75,000 D. P50,000

4. Ratcliff Company produces two products from a joint process: X and Z. Joint processing costs for this
production cycle are P8,000.
Sales price Disposal cost per Further processing Final sale price
Yards per yard at split-off yard at split-off per yard Per yard
X 1,500 P6.00 P3.50 P1.00 P 7.50
Z 2,200 9.00 5.00 3.00 11.25

If X and Z are processed further, no disposal costs will be incurred or such costs will be borne by the
buyer. Using a physical measure, what amount of joint processing cost is allocated to X (round to the
nearest dollar)?
A. P4,000 B. P4,757 C. P5,500 D. P3,243

5. A review of the assets and liabilities of the No Good Company, in bankruptcy on June 30, 2018,
discloses the following:
 A mortgage payable of ₱350,000 is secured by land and buildings valued at ₱560,000.
 Notes payable of ₱175,000 is secured by the equipment valued at ₱140,000.
 Assets other than those referred to, have an estimated value of ₱157,500.
 Liabilities other than those referred to, total ₱420,000, which included claims with priority of ₱52,500.
What is the estimated deficiency to unsecured creditors?
A. ₱87,500 B. ₱35,000 C. ₱402,500 D. ₱315,000

6. The following partial transactions took place between the home office and its two branches, Bacolod
branch and Cebu Branch.
 Upon the instruction of the home office, Cebu Branch affected a fund transfer of P25,000 to
Bacolod Branch
 Bacolod branch collected a Cebu Branch’s account receivable of P35,000 less 2% discount.
 Cebu Branch paid P250,000 representing the traveling expenses of Mr. Jay Ayala, a senior vice
president, when the latter attended the regional conference in Canada. Of the amount paid, 60%
was charged to the home office. 25% to Bacolod branch and the balance to Cebu Branch.
 Home office shipped merchandise costing P200,000 to Cebu Branch. Freight of P3.000 was paid
by the home office. It is the policy of the company to bill its branches at 25% above cost.
 Upon the instruction of the home office, Cebu reshipped the above merchandise to Bacolod
Branch. Freight of P1,500 was paid by Bacolod Branch. Had the goods been shipped directly to
Bacolod Branch? The freight would have been only P4,200. What is the balance of the
Investment in Cebu Branch account in the home office books?
A. P524,800 B. P520,000 C. P271,800 D. P527,800

7. P Corporation used debentures with a par value of P610,000 to acquire 100 percent of the net assets of S
Company on January 1, 2019 and S Company is dissolved. On that date, the fair value of the bonds
issued by P Corp. was P564,000, and the following balance sheet data were reported by S Co.:

Balance Sheet Item Historical cost Fair value


Cash and Receivables P 55,000 P 50,000
Inventory 105,000 200,000
Land 60,000 100,000
Plant and Equipment 400,000 300,000
Less: Accumulated Depreciation ( 150,000)
Goodwill 10,000
Total assets P 480,000
Accounts Payable P 50,000 P 50,000
Common Stock 100,000
Additional Paid-in Capital 60,000
Retained Earnings 270,000
Total Liabilities and Equities P 480,000

P Corporation incurred an out of pocket expenses of P20,000. How much goodwill is to be recognized on
the books of P as a result of the business combination?
A. P0 B. P10,000 C. P20,000 D. P30,000

8. The Grand Company will issue share at P10 par value common stock for all the net assets of the Nuts
Company. Grand’s common has current market value of P40 per share. Nuts balance sheet accounts are
shown below:
Current assets P320,000
Property and equipment 880,000
Liabilities 400,000
Common stock, P4 par 80,000
Additional paid-in capital 320,000
Retained earnings 400,000
The fair value of current assets is P400,000 while that of property and equipment is P1,600,000. All the
liabilities are correctly stated. Grand issued sufficient shares so that the fair market value of the stock
equals the fair market value of Nuts net assets plus goodwill of P200,000. How much must be the cost of
investment if goodwill of P200,000 must be recognized?
A. 2,200,000 B. 1,800,000 C. 45,000 D. 55,000

9. The Red Company owes ₱15,000,000 on the mortgages of its building to City Bank. The building has a
net book value of ₱20,000,000 and a fair value of ₱18,000,000. When Red Company filed for
liquidation, it owed interest of ₱90,000; when the building is sold for ₱18,000,000, the interest due on
the mortgage is ₱200,000. What amount will the bank receive if the unsecured creditors received 80% of
theirclaims?
A. ₱15,000,000 B. ₱15,160,000 C. ₱15,178,000 D. ₱15,200,000

10. The Visayas Company operates a branch in Mindanao. Operating data from the home office and the
branch for 2016 are presented below:

Home office books Branch books


Sales P512,000 P157,000
Purchases 420,000 40,000
Shipments to branch 60,000
Shipments from home office 80,000
Expenses 120,000 25,000
Inventories, January 1:
Home office 160,000
Branch-
Acquired from outsiders 15,000
Acquired from home office billed
Price of 22 ½ above cost 49,000
Inventories, December 31:
Home office 110,000
Branch-
Acquired from outsiders 11,000
Acquired from home office 52,000

What is the combined comprehensive income (loss) of the home office and the branch?
A. P 9,000 B. P (7,000) C. P 2,000 P (9,000)

11. JNA Partnership is entering into liquidation and as the liquidator you are given the following account
balances:
Cash P775,000 Liabilities P1,100.000
Noncash 6,750,000 Loan from A 150,000
J,Capital (20%) 1,275,000
N,Capital (60%) 1,625,000
A Capital (60%) 3,375,000
Total assets P7, 525,00 Total liabilities & capital P7,525,000

During June, noncash assets with a book value of P 175,000 for the liquidation expenses it incurred and
it also paid its liabilities to outside creditors. However, creditors whose account balances amount to
P150,000 decided to condone JNA’s liabilities .Three fourths of the cash received from the sale of
noncash assets were distributed to the partners. What is A’s interest after the first cash distribution?
A. P3,105,000 B. P2,905,000 C. P2,955,000 D. P2,805,000

12. On January 1, 2019, R Corporation purchased 80 percent of S Corporation’s P10 par common stock for
P975,000. On this date, the carrying amount of S’s net assets was P1,000,000. The fair values of S’s
identifiable assets and liabilities were the same as their carrying amounts except for plant assets (net),
which were P100,000 in excess of the carrying amount and with a remaining life of 5 years. For the year
ended December 31, 2019, S had net income of P190,000 and paid cash dividends totaling P125,000.
In the December 31, 2019, consolidated balance sheet, non-controlling interest should be reported at
A. P200,000 B. P213,000 C. P229,000 D. P233,000

13. C Corporation recently purchased 80 percent of the stock of T Decks, Inc., for P232,000. At the date of
purchase the consolidated balance sheet showed P40,000 of goodwill from the acquisition.
The book value of T Deck’s net assets at the time of acquisition was:
A. P192,000 B. P232,000 C. P240,000 D. P290,000

14. Cena,Batista, and Lashley share profits in 5:3:2 ratio. Their capital accounts prior to liquidation (which
is expected to results in substantial gains) are as follows:

Cena P18,000 Cr. balance


Batista P27,000 Cr. balance
Lashley P 3,000 Cr. balance
The partners wish to distribute cash as it becomes available so that the capital accounts may be brought
into the profit and loss ratio as rapidly as possible. Who is the partner to receive the first available cash
and up to how much?
A. Cena, up to P54,000 C. Batista, up to P54,000
B. . Batista up to P16,200 D. Lashley, up to P16,200

15. Apex Company acquired 70% of the outstanding stock of Nadir Corporation on January 2, 2019. The
consolidated balance sheet prepared immediately after the business combination contained the
following:

Current assets P146,000 Current liabilities P 28,000


Fixed assets – net 370,000 Capital stock 350,000
Goodwill 8,100 Retained earnings 111,000
Minority interest (MINA) 35,100
Total P524,100 Total P 524,100

Of the excess payment for the investment in Nadir, P10,000 was ascribed to undervaluation of its fixed
assets; the balance of the excess payment was ascribed to goodwill.

How much is the cost of investment paid by Apex in acquiring the 70% interest in Nadir?
A. P117,000 B. P90,000 C. P 100,000 D. P 135,100

16. A reconciliation of the Investment in Tarlac Branch Account of Manila Company and the Home Office
account carried in the books of the branch shows the following discrepancies at December 31, 2018:
1. A credit for merchandise allowance for P3, 000 was taken by the branch as P3, 600.
2. A charge by the branch of P5, 500 for an advance taken by the president when he visited the branch
has not yet been recorded by the home office.
3. The branch has not taken up P9, 000 covered by a debit memo from the home office as share in
advertising expenses.
The Investment in Tarlac Branch account in the home office books had a debit balance of P430, 000 at
December 31, 2018. The reciprocal accounts were in agreement at the beginning of the year. What is the
unadjusted balance on the Home Office account in the books of the branch on December 31, 2018?
A. P414, 900 B. P419, 900 C. P429, 500 D. P404, 900

17. Troy Company holds 90 percent of the common stock of Brent Company. In 2019, Troy reports sales of
P800,000 and cost sales of P600,000. For this same period, Brent has sales of P300,000 and cost of sales
of P180,000. During 2019, Troy sold merchandise to Brent for P100,000. The subsidiary still possesses
40 percent of this inventory at the end of 2019. Troy had established the transfer price based on its
normal markup. What are consolidated sales and cost of sales?
A. P1,000,000 and P690,000 C. P1,000,000 and P740,000
B. P1,000,000 and P705,000 D. P970,000 and P696,000

18. L, M, and N are partners. On January 2, 2017, their capital balances and profit and loss ratio are as
follows:
L P 625,000 (60%)
M 1,250,000 (25%)
N 1,500,000 (15%)
N withdrew P250,000 during the year. Net loss on December 31, 2017 totalled P500,000. Hence, the
partners decide to liquidate to partnership. It is uncertain how much of the assets will ultimately yield
but favourable realization is expected. It is therefore agreed to distribute cash as it becomes available.
There are unpaid liabilities of P125,000 and cash of P17,500.
What is the book value of noncash assets before realization?
A. P2,625,000 B. P2,607,500 C. P2,750,000 D. P2,732,500

19. On June 1, 2018, Makati Company established a sales agency in Fairview, Quezon City. Upon the
establishment of the sales agency the Makati office sent merchandise samples costing P8, 000 and a cash
working fund of P3, 000 to be maintained on the imprest basis. During the month of June, the sales
agency reported to the home office sales orders. These were billed at P70, 000 of which P40, 000 was
collected. The sales agency paid expenses of P2, 800 but was reimbursed by the home office.
On June 30, 2018, the sales agency samples were valued at P6, 000. It was estimated that the gross profit
on goods shipped to fill agency sales orders averaged 40% of cost. What is the net income of the sales
agency for the month ended June 30, 2018?
A. P 10, 400 B. P15, 200 C. P10, 000 D. P23, 200

20. Parent Corporation owns 90 percent of Subsidiary 1 Company's stock and 75 percent of Subsidiary 2
Company's stock. During 2019, Parent sold inventory purchased in 2019 for P48,000 to Subsidiary 1 for
P60,000. Subsidiary 1 then sold the inventory at its cost of P60,000 to Subsidiary 2. Prior to December
31, 2019, Subsidiary 2 sold P45,000 of inventory to a nonaffiliate for P67,000 and held P15,000 in
inventory at December 31, 2019.
Based on the information given above, what amount should be reported in the 2019 consolidated income
statement as cost of goods sold?
A. P36,000 B. P12,000 C. P48,000 D. P45,000

21. On January 1, 2018, Jomig’s Grill, Inc. granted a franchise to Ms. Jinggay Artugue to sell Jomig’s
products. The franchise agreement provides the following terms:

• Initial franchise fee in the amount of P15,000,000 payable at P5,000,000 down payment on January 1,
2018 and the balance payable in five equal annual installments every December 31. Ms. Jinggay Artugue
issued a 5-year non-interest bearing promissory note. The prevailing market rate for the similar note is
10%.
• Contingent franchise fee equal to 5% of the sales revenue of Ms. Jinggay.
As of December 31, 2018, Jomig’s Grill Inc. has not yet performed substantially all material services or
conditions required of the franchise contract. For the year ended December 31, 2018 Ms. Jinggay Artugue
reported sales revenue in the amount of P 1,000,000. What is the amount of total income to be reported by
Jomig’s Grill, Inc. for the year ended December 31, 2018?
A. P0 B. P50,000 C. P808,157 D. P800,000

22. On July 1, 2018, ACE Corporation borrowed 1,680,000 yen from a Japanese Lender evidenced by an
interest-bearing note due July 1, 2019. The Philippine Peso equivalent of the note principal was as
follows:
July 1, 2018 - Date borrowed P210,000
Dec. 31, 2018 - ACE’s year-end 240,000
July 1, 2019 - Date repaid 280,000
In its income statement for 2019, what amount should ACE include as a foreign Exchange gain or loss?
A. P70,000 gain B. P 40,000 gain C. P70,000 loss D. P 40,000 loss

23. On July 1, 2018, ACCENTURE Company lent P308,000 to a US supplier, evidenced By an interest-
bearing-note due on July 1, 2019. The note is equivalent to $8,000 on the loan date. The note principal
was appropriately included at P328,000 in ACCENTURE’s December 31, 2013 balance sheet. The note
was repaid to ACCENTURE on July 1, 2019. Due date when the exchange rate was P39 to $1. In its
income statement for the year Ended December 31, 2019 what amount should ACCENTURE company
include As a foreign currency transaction gain or loss?
A. P0 B. P 16,000 gain C. P 26,000 gain D. P 16,000 loss

24. The partnership of Agnes, Baby, and Cora has the following account balances before liquidation:
Cash P100,000
Liabilities 17,000
Agnes, Capital 69,000
Baby, Capital ( 8,000)
Cora, Capital 58,000

This partnership is in the process of being liquidated. Agnes and Baby are each entitled to 40% of all
profits and losses with the remaining 20% to Cora.
What is the maximum amount that Baby has to contribute to this partnership because of the deficit
capital balance?
A. P48,000 B. P19,000 C. P84,000 D. P29,000

25. On January 2, 2018, JJG Company signed an agreement to operate as a franchisee of Figaro, Inc. for an
initial franchise fee of P3, 125,000 for 10 years. Of this amount, 40% was paid when the agreement was
signed and the balance payable in four semi-annual payments beginning June 30, 2018; JJG Company
signed a non-bearing note for the balance. JJG’s credit rating indicates that it can borrow money at 24
percent on the loan of this type. Substantial services costing P802, 500 have been rendered by Figaro
Inc. The present value of an annuity of P1 at 12% for 4 periods is P3.04. If the collection of the note is
not reasonable assured, the realized gross profit for the year ended December 31, 2018 is:
A. P1,321,345.50 B. P1,069,031.50 C. P1,316,861.00 D. P1,338,307.00

26. Certain balance sheet accounts of a foreign subsidiary in FIL-AM, Inc at December 31, 2019 have been
translated into Philippine pesos as follows:
Current rate Historical Rate
Accounts receivable P 120,000 P 100,000
Prepaid insurance 55,000 50,000
Copyright 75,000 85,000
What was the total amount included in FIL-AM’s December 31, 2019 consolidated balance sheet for
the above accounts?
A. P255,000 B. P 235,000 C. P240,000 D. P250,000

27. REH Corp. (a Philippine-based company) sold parts to a foreign customer on December 1, 2019, with
payment of 10 million foreign currencies to be received on March 31, 2020. The following exchange
rates apply:
Forward rate
Dates Spot Rate (for March 31, 2020)
December 1, 2019 P .0035 P .0034 (4 months)
December 31, 2019 .0033 P .0032 (3 months)
March 31, 2019 .0038 N/A

REH’s incremental borrowing rate is 12 percent. The present value factor for three months at an annual
rate of interest of 12 percent (1 percent per month) is 0.9706.
Assuming that REH entered into no forward contract, how much foreign exchange gain or loss should it
report on its 2019 income statement with regard to this transaction?
A. P 5,000 gain B. P 3,000 gain C. P 2,000 loss D. P 1,000 loss

28. On June 30, 2018, Mr. Tuason entered into a franchise agreement with TM Company to sell their
products. The agreement provides for an initial franchise fee of P1, 250,000, payable as follows: P350,
000 cash to be paid upon signing of the contract, and the balance in five equal annual payments every
December 31, starting December 31, 2018. Mr. Tuazon signs 15% interest bearing note for the balance.
The agreement further provides that the franchise must pay a continuing franchise fee equal to 5% of its
month gross sales. On October 30, the franchisor completed the initial services required in the contract
at a cost of P787, 500 and incurred indirect cost of P42, 900. The franchisee commenced business
operations on November 2, 2018. The gross sales reported to the franchisor are; November sales, P121,
000 and December sales, P147, 500. The first installment payment was made in due date. Assuming the
collectibility of the note is not reasonably assured, in the statement of comprehensive income for the
year ended December 31, 2018, how much is the net income of TM Company?
A. P234,125 B. P301,625 C. P220,700 D. P200,825

29. Jolly Foods Company, a local company, bought furniture from Ailments Corporation, a US company,
for 35,000 US Dollars in 2019. Pertinent exchange rates relating to this transaction are as follows:
Buying Rate Selling Rate
Receipt of order P 47.10 P 47.20
Date of shipment 47.25 47.45
Balance sheet date 49.50 49.60
Settlement date 49.45 49.50

What is the foreign exchange gain or loss of Jolly Foods Company for 2019?
A. P 78,750 loss B. P 78,750 gain C. P 75,250 loss D. P 75,250 gain

30. The partnership of Randy, Manzano, Rosauro and Dagdag is to be liquidated. The capital balances of
each partner prior to the liquidation were Randy, P25,000; Manzano, (P6,000), Rosauro, P12,000 and
Dagdag, P40,000 and undistributed profits of P20,000. The beginning cash and liabilities amounted to
P5,000 and P60,000 respectively. Partners distribute the profits by giving P20,000 salaries to each
partner with the balance to be divided in a 3:1:2:4 ratio to Randy, Manzano, Rosauro and Dagdag each
repecively. If in the first installment sale, P5,000 were spent for liquidation and P3,000 was withheld,
how much cash will Randy receive in the 2nd installment sale, if the remaining NCAs were sold for
P14,000 and Dagdag received a total of P5,000 in the first sale?
A. 3,750 B. 5,100 C. 4,200 D. 4,050
31. On September 15, 2016, Ace Builders, Inc. won the bid for the construction of a 1,000 room hotel for
Victoria Hotels, Inc. on the reclamation area for P1.2 billion. On the terms of payment, parties agreed to
the following:
a. One percent mobilization fee (deductible from the final bill) payable within fifteen days after the signing
of the contract.
b. Retention of 10% on all billings, payable with the final bill after the acceptance of entire completed
project; and
c. Progress billings on construction within seven days from date of acceptance.
By the end of 2016, the company had presented only one progress billings for 10% completion which
Victoria Hotels, Inc. evaluated and accepted on December 28 for payment in January. The Company
used the percentage of completion method of accounting. How much fee was received by Ace
Builders, Inc.?
A. P 9,800 B. P12,000,000 C. P10,800,000 D. P 1,200,000

32. Toad Co charges initial franchise fees of P345,000 with the agreement that P75,000 is to be paid when
the contract would be signed and the balance in 5-annual payments. The present value of the future
payments discounted at 10% is P204,702. The franchisee also has the option to purchase equipment
from the franchisor for P36,000 which the franchisor is selling for P45,000. The down payment
represents fair measure of the services already rendered by the franchisor and the collectability of the
note is reasonably assured.
Determine the amount of franchise revenue to be recognized from the franchise fees.
A. 279,702 B. 270,702 C. 75,000 D. 66,000

33. Desert Co sold a franchise which was signed on January 5, 2019, called for a P67,500 down payment
plus two annual payments of P22,500 starting January 1, 2020, representing the value of initial franchise
services by Desert Co. Furthermore, the franchise agreement requires the franchisee is to pay 5% of its
gross revenues to provide reasonable profits on continuing franchise services to be performed by Desert
Co. The business opened early in 2019 and its sales amounted to P1,125,000. Assuming a 10% interest
rate is appropriate, determine Desert Co’s total revenue for 2019. (The present value of an annuity of P1
at 10% for 2 periods is 1.7355)
A. 67,500 B. 106,549 C. 162,799 D. 166,704

34. Rose Corp reported the following in their trial balance as of December 31 of the current year:
Installment accounts receivable-end 150,000 Administrative expenses 60,000
Cash sales 500,000 Credit sales 375,000
Freight-in 10,000 Loss on repossession 10,000
Installment sales 600,000 Inventory beginning 50,000
Purchase returns 15,000 Selling expenses 40,000
Repossessions 20,000 Trade-ins 25,000
Purchases 985,000

A P10,000 repossession was not recorded by the company. The credit sales is 25% more than the cash
sales and the installment sales are 20% more than the credit sales. The total ending inventory which
included repossessions of P5,000 and trade-ins of P15,000 amounted to P185,000.
From the data above, determine the shipments on installment sales.
A. 362.033 B. 366,102 C. 300,000 D. 296,667

35. Jet Construction Company began operation on January 2, 2016. During the year, the company entered into a
contract with Angel Company to construct a manufacturing facility. At that time, Jet estimated that it would
take five years to complete the facility at a total cost of P1,800,000. The total contract price for the construction
of the facility is P2,500,000. During the year, the company incurred P440,000 in construction costs related to
the construction project. The estimated cost to complete the contract is P1,560,000. Angel Company was billed
and paid 30% of the contract price subject to a 10% retention. Using the percentage of completion method, how
much is the excess of Construction in Progress over Contract Billings over Construction in Progress?
A. P125,000 (current liability) C. P125,000 (current asset)
B. P200,000 (current asset) D. P200,000 (current liability)

36. WG Company began operations on January 2019. The company uses the installment method of
accounting. The following information pertains to WG Company operations in 2019.
Installment Sales P 1,000,000
Regular Sales 1,200,000
Cost of Installment Sales 500,000
Cost of Regular Sales 700,000
Operating Expenses 450,000
Collection on Installment Sales 400,000
Collection on Regular Sales 600,000
How much is the realized gross profit in 2019?
A. P 700,000 C. P 550,000
B. P 1,000,000 D. P 1,750,000

37. On May 1, 2016, Nueva Builders Company obtained a contract to build a coliseum. The coliseum was to built at
a total costs od P5,000,000 and is scheduled for completion by May 1, 2018. The contract contains a penalty
clause to the effect that the other party was to deduct P10,000 from the P6,000,000 contract price for each week
of delay. Completion was delayed five weeks. Below are data pertaining to the construction period:
2016 2017 2018
Cost incurred to date P500,000 P2,340,000 P2,650,000
Estimated costs to complete 2,000,000 260,000 -
Cash collected 400,000 4,350,000 2,975,000
Using the percentage-of-completion method, what is the realized gross profit(loss) for the year ended December
30, 2018?
A. P325,000 B. P(35,000) C. P(10,000) D. P240,000

38. The W.W Construction Corporation began construction work under a three-year contract. The contract price
was P800,000. WW uses the percentage-of-completion method for financial accounting purposes. The income
to be recognized each year is based on the proportion of cost incurred to total estimated costs for completing the
contract. The financial statement presentations relating to this contract a December 31, 2018 is presented below:
Statement of Financial Position
Accounts receivable-construction billings P15,000
Construction in progress 50,000
Less: contract billings 47,000
Cost of incomplete contract in excess of billings 3,000

Statement of Comprehensive Income


Income (before tax) on the contract
recognized in 2018 P10,000

The amount of cash collected in 2018 and the initial gross profit on this contract are:
Cash Collected Initial Gross Profit
A. P32,000 P800,000
B. P32,000 P160,000
C. P60,000 P750,000
D. P30,000 P790,000

39. The following information came from the books and records of Golden Tree Corporation and its branch.
The balances are as of December 31, 2019, the fourth year of the corporation’s existence.
Home office Branch
Dr. (Cr. ) Dr. (Cr.)
Sales P(850,000)
Shipments to branch P(240,000)
Shipments from home office 360,000
Purchases 180,000
Expenses 160,000
Inventory, January 1, 2019 72,000
Unrealized profit in branch inventory (136,000)
There are no shipments in transit between the home office and the branch. Both Shipments accounts are
properly recorded. The closing inventory at billed prices includes merchandise acquired from the home
office in the amount of P54,000 and P30,000 acquired from vendors for a total of P84,000.
How much of the beginning inventory of the branch was acquired from “outsiders”?
A. P24,000 B. P40,000 C. P56,000 D. P32,000

40. My Home, Inc. sells appliances on installment basis. Below are some of the information from the
records of the company.

2018 2017 2016


ADVANCED ACCOUNTING

Cost of sales P850,000 P686,000 P596,160


Gross profit on sale 32% 30% 28%
Collections on:
2018 sales 425,000
2017 sales 258,000 320,000
2016 sales 185,000 152,000 280,000

During 2017, write-offs of 2016 unpaid accounts were amounting to P7,200. During 2018, repossessions
were made on defaulted accounts on 2017 sales for which unpaid balance amounted to P4,200. The fair value of
the repossessed merchandise is P3,800.
How much is the total deferred gross profit as of December 31,2018?
A. P443,680 B. P440,404 C. P428,080 D. P440,176

41. Sony and Philip formed a joint venture to purchase and sell a special type of merchandise. The venturers
agreed to contribute cash of P168,750 each to be used in purchasing the merchandise, and to share
profits and losses equally. They also agreed that each shall record his own joint venture transactions in
their respective records. Upon termination of the joint venture, the following data are available:
Sony Philip
Joint venture P146,250 cr P131,625 cr
Inventory taken 4,500 14,062.50
Expenses paid out of JV cash 6,750 12,375

What is the amount of Joint Venture sales?


A. P 653,062.50 B. P 634,500 C. P 652,500 D. P 633,937.50

42. For the 2019 summer session, Far Eastern University assessed its students P 300,000 for tuition and
fees. However, the net amount realized was only P 290,000 because of the following reductions:
Tuition remissions granted to faculty members’ families P 3,000
Class cancellation refunds 7,000
How much unrestricted current funds revenues from tuition and fees should Far Eastern University
report for the period?
A. P 290,000 B. P297,000 C. 293,000 D. 300,000

43. MT Company, which began operations on January 2,2018 appropriately uses the installment method of
accounting. The following data pertain to 2018 operations:
Installment sales P900,000 Operating expenses P72,000
Regular sales 375,000 Collections (including
Interest of P24,000) 312,000
Cost of regular sales 215,000 Installment accounts
Written-off due to
defaults 44,000
Cost of installment sales 630,000 Repossessed accounts 100,000
Fair value of repossessed
Merchandise 54,000 Reconditioning cost 4,000

What is the net income for the year ended December 31,2018?
A. P151,600 B. P127,600 C. P158,400 D. P165,600

44. For the summer session of 2019, Virgen Milagrosa University assessed its students P 1,700,000 (net of
refunds), covering tuition and fees for educational and general purposes. However, only P 1,500,000
was expected to be realized because scholarships totaling P150,000 were granted to students, and tuition
remissions of P50,000 were allowed to faculty members’ children attending Virgen Milagrosa. What
amount should Virgen Milagrosa include in the unrestricted current funds as revenues from student
tuition and fees?
A. P 1,500,000 B. P 1,650,000 C. 1,550,000 D. 1,700,000

Page 9 of 13
ADVANCED ACCOUNTING

45. ACA Video Company sells Betamax equipment. It maintains its accounting records on a calendar year
basis. On October 1,2017, ACA Video Company sold a television set to Mr. Santiago. The cost of the
set was P18,000, and the set was sold for P24,000. A down payment of P6,000 was received along with
a contract calling for the subsequent payment of P1,000 on the first day of each month starting on the
following month. No interest was added to the contract. Mr. Santiago paid the monthly installments
promptly on November 1 and December 1 in 2017. He also made seven installment payments in 2018
after which he defaulted on the contract. The set was then repossessed on November 1,2018. Assuming
the repossessed set has a fair value of P4,000, what is the gain (loss) on repossession to be recognized?
A. P(2,750) B. P2,750 C. P 750 D. P1,500

46. On January 2,2017, Mustang Company sola a car to Mr. De Jesus for P1,050,000. On this date, the car
cost P735,000. Mr. De Jesus paid P150,000 as down-payment and signed a P900,000 interest bearing
note at 10 percent. The note was payable in three annual installments of P300,000 beginning January
1,2018. Mr. De Jesus made at timely payment for the first installment on January 1,2018 of P390,000
which included interest of P90,000 to date of payment. Mustang Company uses the installment method
of accounting. In its December 31,2018 Statement of Financial Position, what amount should Mustang
Company report as deferred gross profit?
A. P180,000 B. P153,000 C. P270,000 D. P225,000

47. Agency III establish a petty cash fund for Maintenance and Other Operating expenses (MOOE)
amounting to P10,000. The entry for this transaction would be:
A. No entry
B. Memorandum entry
C. Petty Cash Fund 10,000
Cash-National Treasury, MDS 10,000
D. Petty Cash Fund 10,000
Cash-Disbursing Officers 10,000

48. Agency JJJ grants cash advance to Ms. Violet for travelling expense amount P12,000. The entry for the
cash advance would be:
A. Cash-Disbursing Officer 12,000
Cash-National Treasury, MDS 12,000
B. Cash-Collecting Officer 12,000
Cash-National Treasury, MDS 12,000
C. Travel Expense 12,000
Cash-National Treasury, MDS 12,000
D. Advances to Officers and Employees 12,000
Cash-National Treasury, MDS 12,000

49. In July, 2017, Sta. Lucia Company who uses the installment method of accounting sold land costing
P90,000 for P240,000, receiving P35,000 cash as down payment and a mortgage note for the balance
payable in monthly installments. Installment received in 2017 reduced the principal of the note to a
balance of P200,000. The buyer defaulted on the note at the beginning of 2018 and the property was
repossessed. The property had an appraised value of P165,000 at the time of repossession. The realized
gross profit in 2017 and the gain (loss) on repossession in 2018 amounted to:
Realized Gross Profit Gain (loss) on Repossesssion
A. P15,000 (P90,000)
B. 25,000 90,000
C. 9,000 (2,500)
D. 2,500 3,500

50. Bistrol Corporation uses the weighted-average method in its process costing system. This month, the
beginning inventory in the first processing department consisted of 800 units. The costs and percentage
completion of these units in beginning inventory were:
Cost Percent Complete
Materials costs P15,700 75%
Conversion costs P7,700 20%

Page 10 of 13
ADVANCED ACCOUNTING

A total of 8,400 units were started and 7,500 units were transferred to the second processing department
during the month. The following costs were incurred in the first processing department during the
month:
Materials costs P 186,300
Conversion costs P 329,800
The ending inventory was 70% complete with respect to materials and 60% complete with respect to
conversion costs.
Note: Your answers may differ from those offered below due to rounding error. In all cases, select the
answer that is the closest to the answer you computed. To reduce rounding error, carry out all
computations to at least three decimal places.
The cost per equivalent unit for materials for the month in the first processing department is closest to:
A) P21.44 B) P21.96 C) P20.25 D) P23.25

51. Romeo and Parco are partners in a merchandising business. During 2018 they withdrew their salary
allowances of P34 000 and P59 ,000 respectively. Bonus is given to Parco based on 25% of net income
after salaries and bonus. Remaining profit or loss are shared in the ratio of 3:2 by Romeo and Parco. The
partners capital accounts show the following:
Romeo Parco
Beginning Balance P85,000 P67,000
Additional investments 40,000 43,000
Withdrawal other than salary allowance ( 35,000) ( 20,000)
Subtotal 90,000 90,000
What is the total amount of net income for the period if Romeo’s equity balance after dividing net income is P]
42,200?
A. P201, 750 B. P163,400 C. P197,400 D. P180,000

52. For the last several years, Malay Corporation has encountered a declining market for its major product
line. Attempts to diversity have let to additional disappointments. This unfortunate set of circumstances
has left the company with significant debt and an inability to service its debt. The existing debt consists
of P20,000,000 of principal and P875,000 of accrued interest. Discussions with the creditors have
resulted in a propose restructuring of debt. The restructuring would consist of the following actions:
a. Exchanging preferred stock with a fair value of P5,100,000 and a par value of P5,000,000 in
exchange of full settlement of P5,500,000 of principal debt.
b. Exchanging land with a value of P4,000,000 and a book value of P3,000,000 in exchange for
P4,500,000 of principal debt.
c. The remaining debt and accrued interest would be repaid over the next 10 years with semi-annual
payments due every six months. The annual stated rate would be 8.5%

Past operating losses have resulted in a deficit in retained earnings of P3,400,000. In addition to the deficit, the
company’s equity includes common stock at par value of P6,000,000 and contributed capital in excess of par
value in the amount of P1,000,000. What is the adjusted retained earnings (deficit) after restructuring?
A. P1,500,000 B. P (1,500,000) C. P (1,450,000) D. P1,450,000

53. The accounting records of Omar Company contained the following information for last year:
Beginning Ending
Direct materials inventory P 9,000 P 7,000
Work in process inventory P17,000 P31,000
Finished goods inventory P10,000 P15,000
Manufacturing costs incurred
Direct materials used P 72,000
Overhead applied P 24,000
Direct labor cost (10,000 hours) P 80,000
Depreciation P 10,000
Rent P 12,000
Taxes P 8,000
Cost of goods sold P157,000* *
Selling and administrative costs incurred
Advertising P35,000
Page 11 of 13
ADVANCED ACCOUNTING

Rent P20,000
Clerical P25,000
*Does not include over- or underapplied overhead.

The cost of goods manufactured for the year was:


A) P190,000 B) P162,000 C) P168,000 D) P135,000

54. The following statement of financial position is presented for the partnership of Colt, Mark, and Clock,
who share profits and losses in the ratio of 4:3:3.
Assets
Cash P 90,000
Other assets 830,000
Colt, loan 20,000
Total assets P940,000
Liability and Equity
Accounts payable P210,000
Clock, loan 30,000
Colt, capital 310,000
Mark, capital 200,000
Clock, capital 190,000
Total liabilities and capital 940,000
Assume the partners decide to liquidate the partnership. If the other assets are sold for P700,000, how much
of the available cash should be distributed to Colt?
A. P230,000 B. P238,000 C. P258,000 D. P310,000

55. Good Company, as a result of experiencing financial difficulties, has its debt restructured. According to
the restructuring agreement, land owned by Good will be transferred to the creditor in full settlement of
Good’s debt which totaled P300,000. The land originally cost P290,000 and had a fair market value of
P270,000 on the date it was transferred to the creditor. As a result of this debt restructure, Good would
report which of the following:

Loss on Gain on
Transfer of Land Restructuring of Debt
A. P -0- P10,000
B. P -0- P30,000
C. P20,000 P10,000
D. P20,000 P30,000

56. HiTech Products manufactures three types of remote-control devices: Economy, Standard, and Deluxe.
The company, which uses activity-based costing, has identified five activities (and related cost drivers).
Each activity, its budgeted cost, and related cost driver is identified below.
Activity Cost Cost Driver
Material Handling P 225,000 Number of parts
Material Insertion 2,475,000 Number of parts
Automated 840,000 Machine Hours
Machinery
Finishing 170,000 Direct Labor Hours
Packaging 170,000 Orders Shipped
Total P 3,880,000
The following information pertains to the three product lines for next year:
Economy Standard Deluxe
Units produced 10,000 5,000 2,000
Orders Shipped 1,000 500 200
Number of parts per unit 10 15 25
Machine Hours per unit 1 3 5
Labor hours per unit 2 2 2

Page 12 of 13
ADVANCED ACCOUNTING

What is HiTech's pool rate for the material-handling activity?


A. P1.00/part B. P2.25/part C. P6.62/labor hour D. P13.23/part.

57. The following July information is for Marley Company:


Standards:
Material 3.0 feet per unit @ P4.20 per foot
Labor 2.5 hours per unit @ P7.50 per hour

Actual:
Production 2,750 units produced during the month
Material 8,700 feet used; 9,000 feet purchased @ P4.50 per foot
Labor 7,000 direct labor hours @ P7.90 per hour
What is the labor rate variance?
A. P3,480 U B. P3,480 F C. P2,800 U D. P2,800 F

58. The following information is taken from the statement of affairs of the Solvent Corporation:
Assets pledged with fully secured creditors (current
fair value, P166,000) P208,000
Assets pledged with partially secured creditors (current
fair value, P112,000) 144,000
Free assets (current fair value, P104,000) 124,000
Liabilities with priority 26,000
Fully secured creditors 76,000
Partially secured creditors 136,000
Unsecured creditors 276,000
Determine the estimated amount to be paid to unsecured creditors wit priority:
A. P14,500 B. P16,812 C. P26,000 D. P20,000

59. The RST Partnership is in the process of liquidation. The account balances prior to liquidation are given
below:
Debits Credits
Cash P72,000 Liabilities P40,000
Rita, drawing 10,000 Sara, loan 8,000
Sara, drawing 15,000 Tita, loan 25,000
Tita, drawing 20,000 Rita, capital 49,000
Operating loss 21,000 Sara, capital 18,000
Loss on realization 12,000 Tita, capital 10,000
The partners share profits in the following ratio: Rita, 1/6: Sara, 2/6: and Tita. 3/6. Upon liquidations of the
partnership, Rita should have received:
A. P 0 B. P32,000 C. P33,500 D. P35,500

60. Apex Company acquired 70% of the outstanding stock of Nadir Corporation on January 2, 2019. The
consolidated balance sheet prepared immediately after the business combination contained the
following:

Current assets P146,000 Current liabilities P 28,000


Fixed assets – net 370,000 Capital stock 350,000
Goodwill 8,100 Retained earnings 111,000
Minority interest (MINA) 35,100
Total P524,100 Total P 524,100
Of the excess payment for the investment in Nadir, P10,000 was ascribed to undervaluation of its fixed
assets; the balance of the excess payment was ascribed to goodwill.
How much is the underlying equity of Apex in the net assets of Nadir Corporation?
A. P71,900 B. P90,000 C. P 63,800 D. P 81,900

END

Page 13 of 13
Advance Financial Accounting and Reporting
Set E

1. M & J Corp. which sells goods on installment basis, recognizes at year end gross profit on collections which is consisted of cost and gross
profit. It reported the following:
January 1 December 31
Installment receivables
2011 P120,100 0
2012 1,722,300 P337,200
2013 0 2,050,450
Sales and cost of sales for the three years are as follows:
2011 2012 2013
Sales P1,900,000 P2,610,000 P3,010,0000
Cost of sales 1,235,000 1,425,000 1,896,300
In 2013 the company repossessed merchandise with resale value of P8,500 from customers who defaulted in payments. The sales were made in
2012 for P27,000 on which P16,000 was collected prior to default. As collections are made, the company debits cash and credits installment
receivable. For default and repossessions, the company debits installment receivable. The amount of adjustment on the inventory of repossessed
merchandise to the extent of the unrealized gross profit was
a. Zero
b. A decrease of P6,240
c. A decrease of P2,500
d. A decrease of P3,740

2. On October 2013, Haybol Realty Co. sold to Mae Balay a property for P500,000 which is carried in its books for P250,000. The company
received P100,000 on the date of the sale and a mortgage note for P400,000 payable in twenty (20) semiannual installments of P20,000 plus
interest on the unpaid principal at 16% per annum.
The realized profit to be recognized by Haybol Realty Corp. in 2013 if gross profit is recognized periodically in proportion to
collections would be
a. P50,000
b. P100,000
c. P60,000
d. P250,000

3. Quincy Enterprises uses the installment method of accounting and has the following data at year-end:
Gross margin on cost 66 2/3%
Unrealized gross profit P192,000
Cash collection including down payments 360,000
What was the total amount of sale on installment basis?
a. P480,000
b. P648,000
c. P552,000
d. P840,000

4. Lovebirds Corporation sells goods on the installment basis. For the year just ended, the following were reported:
Cost of installment sales P 525,000
Loss on repossession 13,500
Fair value of repossessed merchandise 112,500
Account defaulted 180,000
Deferred gross profit, end 108,000
How much was the collections for the year?
a. P210,000 b. P264,000
c. P390,000 d. P415,715

5. Ondoy Company began operations on January 1, 2011 and appropriately uses the installment method of accounting. The following data are
available for 2011 and 2012
2011 2012
Installment sales 1,200,000 1,500,000
Cash collections from:
2007 sales 400,000 500,000
2008 sales 600,000
Gross profit on sales 30% 40%
The realized gross profit for 2012 is
a. 600,000
b. 240,000
c. 390,000
d. 440,000
6. ACE Company’s operations for the month just ended originally set up a 60,000 direct labor hour level, with budgeted direct labor of P960,000
and budgeted variable overhead of P240,000. The actual results revealed that direct labor incurred amounted to P1,148,000 and that the
unfavorable variable overhead variance was P40,000. Labor trouble caused an unfavorable labor efficiency variance of P120,000, and new
employees hired at higher rates resulted in an actual average wage rate of P16.40 per hour. The total number of standard direct labor hours
allowed for the actual units produced is
A.P52,500 B. P60,000 C.P62,500 D. P70,000
7. TAMARAW, Inc. has a maintenance shop where repairs to its motor vehicles are done. During last month’s labor strike, certain recorded were
lost. The actual input of direct labor hours was 1,000, and the resulting direct labor budget variance was a favorable P3,400. The standard direct
labor rate was P28.00 per hour, but an unexpected labor shortage necessitated the hiring of higher-paid workers for some jobs and had resulted in
a rate variance of P800. The actual direct labor rate was
A.P27.20 per hour B. P28.80 per hour C. P30.25 per hour D. P31.40 per
hour
8. The total variance is $5,000. The total materials variance is $2,000. The total labor
variance is twice the total overhead variance. What is the total overhead variance?
a.$500 b.$1,000
c.$1,500 d.$2,000
9. The formula for the materials price variance is
a.(AQ × SP) – (SQ × SP).
b.(AQ × AP) – (AQ × SP).
c.(AQ × AP) – (SQ × SP).
d.(AQ × SP) – (SQ × AP).
10. The formula for the materials quantity variance is
a.(SQ × AP) – (SQ × SP).
b.(AQ × AP) – (AQ × SP).
c.(AQ × SP) – (SQ × SP).
d.(AQ × AP) – (SQ × SP).
11. A company uses 6,300 pounds of materials and exceeds the standard by 300 pounds.
The quantity variance is $900 unfavorable. What is the standard price?
a.$1.00.
b.$2.00.
c.$3.00.
d.Cannot be determined from the data provided.
12. A company purchases 15,000 pounds of materials. The materials price variance is $3,000
favorable. What is the difference between the standard and actual price paid for the materials?
a.$1.00.
b.$.20.
c.$5.00.
d.Cannot be determined
13. A company uses 20,000 pounds of materials for which they paid $4.50 a pound. What is
the materials price variance?
a.$.50.
b.$1.00.
c.$2.50.
d.Cannot be determined from the data provided
14. If the materials price variance is $1,200 F and the materials quantity and labor variances are each $900 U, what is the total materials variance?
a.$1,200 F
b.$900 U
c.$300 F
d.$1,350 U
15. Batanes Construction Company recognized gross loss of P42,000 on its long-term project which has accumulated costs of P490,000. To finish the
project, the company estimates that it has to incur additional cost of P735,000. The contract price is:
a. P798,000 b. P1,330,000 c. P1,225,000 d. P1,183,000
16. On November 30, 2009, Loveless Company authorized NBSB Corp. to operate as a franchisee for an initial franchise fee of P1,950,000. Of his
amount, P750,000 was received upon signing the agreement and the balance, represented by a note, is due in four annual payments starting
November 30, 2010. Present value of P1 at 12% for 4 periods is 0.6355.
Present value of an ordinary annuity of P1 at 12% for 4 periods is 3.0374. The period of refund will elapsed on January 31, 2010. The franchisor
has performed substantially all of the initial services but the operations of the store have yet to start. Collectibility of the note is reasonably
certain. How much is the unearned franchise fee on the year ended December 31, 2009?
a. P1,661,220 b. P750,000 c. P991,220 d. P0
for questions 17-18.
Omega Inc. started a 4-year contract to build a dam. Activities commenced on February 1, 2007. The total contract price amounted to P12 million, and
it was estimated that the work would be completed at a total cost of P9.5 million. In the construction agreement the customer agreed to accept increases
in wage tariffs additional to the contract price.
The following information refers contract activities for the financial year ending December 31, 2007:
 Costs for the year:
Materials……………………………………….. P1,400
Labor…………………………………………….. 800
Operating overheads………………………. 150
Subcontractors……………………………….. 180
 Current estimate of total contract costs indicates the following:
a. Materials are to be P180,000 higher than expected.
b. Total labor costs are to be P300,000 higher than expected. Of this amount, only P240,000 would be brought about by
increased wage tariffs. The other amount would be due to inefficiencies.
c. A savings of P300,000 is expected on operating overheads.
 During the current financial year the customer requested a variation to the original contract and it was agreed that the contract price
would be to increased by P900,000. The total estimated cost of this extra work is P750,000.
 By the end of 2007, certificates issued by quantity surveyors indicated a 25% stage of completion.
17. Compute the amount of gross profit or loss to be recognized in 2007 using contract costs in proportion to estimated contract costs (percentage of
completion method):
a. P568,000 c. P610,000
b. P577,000 d. P755,000
18. Compute the amount of gross profit or loss to be recognized in 2007 using percentage of the work certified (percentage of completion method –
output method using actual cost approach):
a. P568,000 c. P610,000
b. P577,000 d. P755,000
19. Tam’s Pizza, Inc. charges an initial franchise fee of P50,000 for the right to operate as a franchisee of Tam’s Pizza. Of this amount, P10,000 is
payable when the agreement was signed and the balance is payable in five annual payments of P8,000 each. In return for the initial franchise fee,
the franchiser will help locate the site, negotiate with the lease or purchase of the site, supervise the construction activity, and provide the
bookkeeping services. The credit rating of the franchisee indicates that money can be borrowed at 8%. The present value of an ordinary annuity
of five receipts of P8,000 each discounted at 8% is P31,941.68.
If the initial downpayment is not refundable and no future services are required by the franchiser, but collection of the note is so uncertain that
recognition of the note as an asset is unwarranted, the entry should be:
a. Cash………………………………………………………………… 10,000.00
Notes Receivable………………………………………………. 40,000.00
Discounts on Notes Receivable…………. 8,058.32
Unearned Franchise Fees…………………. 41,941.68
b. Cash……………………………………………………………….. 10,000.00
Notes Receivable……………………………………………… 40,000.00
Discounts on Notes Receivable…………. 8,058.32
Revenue from Franchise Fees………….. 41,941.68
c. Cash ……………………………………………………………… 10,000.00
Revenue from Franchise Fees……………………. 10,000.00
d. Cash……………………………………………………………… 10,000.00
Unearned Franchise Fees………………………. 10,000.00
20. A total materials variance is analyzed in terms of
a. price and quantity variances.
b. buy and sell variances.
c. quantity and quality variances.
d .tight and loose variances.
Chapter 4
Installment Sales

Installment sales problems have appeared very often in the CPA exam. Therefore, candidates should be familiar with the accounting techniques applicable
to this topic.
When a sale is made on the installment basis, the buyer usually makes a down payment and promises to pay the balance in regular installments over a
specified period of time. Profit on installment sales is recognized only when earned. Although there are several theoretical points at which the profit can be
assumed to be earned, for CPA examinations purposes, the choice is generally limited to the installment method.
Installment Method
Under this method, income is recognized only when collections are made. Problems requiring the use of the installment method of recognizing income
have appeared quite regularly in the CPA exam. The following are the typical problems often encountered in the CPA exam:

1. Computation of Gross Profit Rate for each year of sales.


2. Computation of Realized Gross Profit for each year of sales.
3. Computation of Deferred Gross Account balance at the end of year.
4. Computation of Gain or Loss on repossessions.
Computation of Gross Profit Rate
To compute the realized gross profit in proportion to the collections made, it is necessary to determine the gross profit rate for each year’s operations. The
following are the formulas in computing gross profit rate:
Gross Profit
Current year sales: Gross Profit Rate = Installment Sales

Prior year sales:

Deferred Gross Profit (Beg.) – Prior Year Sales


Gross Profit Rate =
Installment Accounts Receivable (Beg.) – Prior Year Sales

Computation of Realized Gross Profit


Once the gross profit rates are known, it is possible to compute the realized gross profit based on cash collections. The formula to be used is:
Realized Gross Profit = Collections (excluding interest) x Gross Profit Rate (based on sale)
Missing Factors. In as much as the realized gross profit under the installment method depends upon cash collections of receivables, it is important that the
amounts collected must be known. However, in some problems, the collections are not specifically stated. Such collections must be reconstructed from
related information available from the data given. The candidate should remember the following format in computing the collections:

Current Year Sales Prior Year Sales


Installment accounts receivable – beginning xx xx
Installment accounts receivable – end (xx) (xx)

Total credits xx xx
Credit for repossessions (unpaid balance) (xx) (xx)
Credit for installment A/C written off (xx) (xx)
Credit representing collections xx xx

Computation of Deferred Gross Profit, End


To compute the balance of Deferred Gross profit at the end of the year, the following formula may be used:
Installment Account Receivable – End x GPR = Deferred Gross Profit – End
Or
Deferred Gross Profit – before adjustment xx
Less: Realized gross profit xx
Deferred Gross Profit - End xx
Computation of Gain or Loss on Repossession
If a customer does not make an installment payment at the specified time, it is necessary to repossess the merchandise in order for the seller to
minimize his loss.
The gain or loss on repossession is computed as follows:

Fair value of repossessed merchandise xx


Less: Unrecovered cost -
Unpaid balance xx
Less: deferred gross profit (unpaid balance x GP rate) xx xx
Gain (loss) on repossession xx

The fair value of repossessed merchandise at the time of repossession should be before reconditioning cost and before adding a normal gross
profit from sale of repossessed merchandise.
Trade In
This type of installment sales used by car dealers, whereby an old car is received as down payment from the buyer for sale of the new car. Usually the
old car traded-in is overvalued to induce the trade-in. for problem solving purposes the overvaluation is computed using a formula below:

Trade-in value allowed on the old car Pxx


Less: Actual value
Estimated selling price Pxx
Less: Normal gross profit from the sale of used car Pxx
Reconditioning costs xx xx xx
Overallowance on the old car Pxx
The overallowance is treated as a deduction from the selling price of the new car. When there is overallowance on the old car traded-in, the gross profit
rate is computed as follows:
Gross profit ÷ Net Sales (net of overallowance)
The realized gross profit is also computed as follows:
Collections (cash + actual value of old car) x GPR
PROBLEMS
1. Oro Company began operations on January 1, 2012 and appropriately uses the installment sales method of accounting. The following
data are available for 2012 and 2013:
2012 2013
Installment sales P1,500,000 P1,800,000
Gross profit on sales 30% 40%
Cash collections from:
2012 sales 500,000 600,000
2013 sales - 700,000
The realized gross profit for 2013 is:
a. P720,000
b. 520,000
c. 460,000
d. 280,000
2. Roco Corp., which began business on January 1, 2013, appropriately uses the installment sales method of accounting for
income tax reporting purposes. The following data are available for 2013:
Installment accounts receivable, 12/31/2013 P200,000
Installment sales for 2013 350,000
Gross profit on sales 40%
Under the installment method, what would be Roco’s deferred gross profit at December 31, 2013?
a. P20,000
b. 90,000
c. 80,000
d. 60,000
3. Gray Co., which began operations on January 1, 2013, appropriately uses the installment method of accounting. The following information
pertains to Gray operations for the 2013:

Installment sales P500,000


Regular sales 300,000
Cost of installment sales 250,000
Cost of regular sales 150,000
General and administrative expenses 50,000
Collections on installment sales 100,000
In its December 31, 2013 statement of financial position, what amount should Gray report as deferred gross profit?
a. P250,000
b. 200,000
c. 160,000
d. 75,000
4. Filstate Co. is a real estate developer that began operations on January 2, 2013. Filstate appropriately uses the installment method of
revenue recognition. Filstate sales are made on the basis of a 10% downpayment, with the balance payable over 30 years. Filstate gross
profit percentage is 40%. Relevant information for Filstate first year of operations is as follows:

Sales P16,000,000
Cash collections 2,020,000
The realized gross profit and deferred gross profit at December 31, 2013 are:
a. P808,000 and P5,592,000
b. 5,040,000 and 808,000
c. 5,600,000 and 808,000
d. 808,000 and 6,400,000
5. Long Co., which began operations on January 1, 2013, appropriately uses the installment method of accounting. The following information
pertains to Long’s operations for the year 2013:
Installment sales P1,000,000
Regular sales 600,000
Cost of installment sales 500,000
Cost of regular sales 300,000
General and administrative expenses 100,000
Collections on installment sales 200,0000
What is the total comprehensive income on December 31, 2013?
a. P400,000
b. 200,000
c. 300,000
d. 100,000
6. Kiko Co. began operations on January 1, 2013 and appropriately uses the installment method of accounting. The following information
pertains to Kiko’s operations for 2013:

Installment sales P800,000


Cost of installment sales 480,000
General and administrative expenses 80,000
Collections on installment sales 300,000
The balance in the deferred gross profit account at December 31, 2013 should be:
a. P120,000
b. 150,000
c. 200,000
d. 320,000
7. Tayag Corp., which began operations in 2013, accounts for revenues using the installment method. Tayag’s sales and collections for the
year were P60,000 and P35,000, respectively. Uncollectible accounts receivable of P5,000 were written off during 2013. Tayag’s gross
profit rate is 30%. On December 31, 2013, what amount should Tayag report as deferred revenue?
a. P10,500
b. 9,000
c. 7,500
d. 6,000
8. Laya Corp., which began operations on January 2, 2013, appropriately uses the installment sales method of accounting. The following
information is available for 2013:
Installment accounts receivable, December 31, 2013 P800,000
Deferred gross profit, December 31, 2013
(before recognition of realized gross profit for 2013) 560,000
Gross profit on sales 40%
For the year ended December 31, 2013, realized gross profit on sales should be:
a. P320,000
b. 340,000
c. 320,000
d. 240,000
9. Dulce Co., which began operations on January 1, 2012, appropriately uses the installment method of accounting to record revenues.
The following information is available for the years ended December 31, 2012 and 2013:
2012 2013
Installment sales P1,000,000 P1,800,000
Gross profit realized on sales made in:
2012 150,000 90,000
2013 - 200,000
Gross profit percentages 30% 40%
What amount of installment accounts receivable should Dulce report in its December 31, 2013, statement of financial position?
a. P1,225,000
b. 1,300,000
c. 1,700,000
d. 1,775,000
10. On January 2, 2012, Black Co. sold a used machine to White, Inc. for P900,000, resulting in a gain of P270,000. On that date, White paid
P150,000 cash and signed a P750,000 note bearing interest at 10%. The note was payable in three annual installments of P250,000 beginning
January 2, 2013. Black appropriately accounted for the sale under the installment method. White made a timely payment of the first
installment on January 2, 2013, of P325,000, which included accrued interest of P75,000. What amount of deferred gross profit should Black
report at December 31, 2013?
a. P150,000
b. 172,500
c. 180,000
d. 225,000
11. White Plains, Inc. sells residential lots on installment basis. The following data was taken from the accounting records of the company as at
December 31, 2013:

Installment accounts receivable, January 1 P755,000


Installment accounts receivable, December 31 840,000
Deferred gross profit, January 1 339,750
Installment sales 950,000
Complete (1) the realized gross profit on December 31, 2013 and (2) the balance of the Deferred Gross Profit account on December 31,
2013.
a. (1) P389,250; and (2) P378,000
b. (1) 427,500; and (2) 389,250
c. (1) 330,750; and (2) 427,000
d. (1) 378,000; and (2) 339,750
12. In August, 2012, Mega World Inc. sold condominium units costing P1,440,000 for P2,400,000 receiving P350,000 cash and a mortgage note
for the balance payable in monthly installments. Installment received in 2010 reduced the principal of the note to
a balance of P2,000,000. The buyer defaulted on the note at the beginning of 2013, and the property was repossessed. The property had a
fair market value of P1,150,000 at the time of repossession.
Compute the gain (loss) on repossession if (1) profit is recognized at the point of sale and (2) gross profit is recognized in proportion to
collections.
a. (1) P(850,000); and (2) P(50,000)
b. (1) (850,000); and (2) (450,000)
c. (1) 850,000; and (2) (450,000)
d. (1) (50,000); and (2) 50,000
13. Sarao Motors sells locally manufactured jeeps on installment basis. Data presented below related to the company’s operations for
the last three calendar years:
2013 2012 2011
cost of installment sales P8,765,625 P7,700,000 P4,950,000
Gross profit rates on sales 32% 30% 38%

Installment accounts receivable, 12/31:


From 2013 sales 9,728,125
From 2012 sales 3,025,000 8,387,500
From 2011 sales 1,512,500 4,812,500
On December 31, 2013 how much is the (1) total realized gross profit and (2) deferred gross profit?
a. (1) P3,044,250; and (2) P4,020,500
b. (1) 3,044,250; and (2) 4,125,000
c. (1) 3,733,750; and (2) 4,020,500
d. (1) 6,993,250; and (2) 4,020,500
14. Polo Company appropriately uses the installment sales method of recognizing revenue. On December 31, 2013, the accounting records
show unadjusted balances of the following:

Installment accounts receivable – 2011 P12,000


Installment accounts receivable – 2012 40,000
Installment accounts receivable – 2013 130,000
Deferred gross profit – 2011 10,500
Deferred gross profit – 2012 28,900
Deferred gross profit – 2013 96,000
Gross profit rates:
2011 35%
2012 34%
2013 32%
For the year ended December 31, 2013, compute (1) total realized gross profit and (2)
the total cash collections in 2013:
a. (1) P182,000; and (2) P135,400
b. (1) 76,000; and (2) 233,000
c. (1) 158,000; and (2) 368,400
d. (1) 106,000; and (2) 97,600
15. Bally Company, which began operations on January 2, 2013 appropriately, uses the installment method of revenue recognition. The
following data pertains to the company’s operations for the 2013:
Installment sales P1,000,000
Cost of installment sales 500,000
Collections on installment sales 150,000
Installment accounts receivable written off 50,000
What is the balance of Deferred Gross Profit account – 2013 on December 31, 2013?
a. P500,000
b. 150,000
c. 400,000
d. 320,000
16. Nike Company, which began operations on January 5, 2012, appropriately uses the installment method of revenue recognition. The
following information pertains to the company’s operations for 2012 and 2013:
2012 2013
Sales P300,000 P450,000
Collections from:
2012 sales 100,000 50,000
2013 sales -0- 150,000
Accounts written off from
2012 sales 25,000 75,000
2013 sales -0- 150,000
Gross profit rates 30% 40%
What amount should Nike Company report as deferred gross profit in its December 31, 2013 statement of financial position?
a. P75,000
b. 80,000
c. 112,000
d. 125,000
17. The following accounts appeared in the accounting records of Adidas Sales Company as of December 31, 2013:
Installment accounts receivable – 2012 P15,000 Repossessions P3,000
Installment accounts receivable – 2013 200,000 Installment sales 425,000
Inventory, December 31, 2012 70,000 Regular sales 385,000
Purchases 555,000 Deferred gross profit - 2012 54,000
Additional information:
Installment accounts receivable – 2012, January 1, 2013 P120,00
Inventory of new and repossessed merchandise, December 31, 2013 95,000
Gross profit rate on regular sales 30%
Repossession was made during the year, 2013. It was a 2012 sale and the corresponding uncollected balance at the time of repossession was
P7,200.
Compute (1) the total realized gross profit for 2013 and the (2) loss on repossession:
a. (1) P129,510; and (2) P960
b. (1) 129,510; and (2) 1,464
c. (1) 245,000; and (2) 960
d. (1) 85,500; and (2) 1,464
18. Mango Company, which sells appliances started operations on January 10,2013 operates on a calendar year basis, and uses the installment
method of revenue recognition. The following data were taken from the 2010 and 2011 accounting records:
2012 2013
Installment sales P480,000 P620,000
Gross profit rates based on cost 25% 20%
Cash collection on 2012 sales 130,000 240,000
Cash collection on 2013 sales 160,000
What is the amount of realized gross profit to be recognized on December 31,2013?
a. P124,500
b. P100,000
c. P92,000
d. P74,667
19. Lacoste Corporation has been using the cash method of revenue recognition. All sales are made on account with notes receivable given
by the customers. The income statement for 2013 presented the following data:
Revenues – collection on principal P32,000
Revenues – interes 3,600
Cost of goods purchases (includes 45,200
inventory of goods on hand P2,000)
The balances due on the notes on December 31 were as follows:
Notes receivable P62,000
Unearned interest income 7,167
Assuming the use of the installment method of revenue recognition, what is the realized gross profit on December 31,2013?
a. P16,080
b. P25,586
c. P18,060
d. P43,633
20. Sta. Lucia Realty Corporation sells residential subdivision lots on installment basis. The following data were taken from the company’s
accounting records as of December
31,2013. The company uses a uniform gross profit rate:
Installment accounts receivable:
January 1,2013 P1,510,000
December 31,2013 1,680,000
Unrealized gross profit – January 1,2013 679,500
Installment sales – 2012 1,180,000
Installment sales - 2013 1,900,000
a. P778,500
b. P679,500
c. P756,500
d. P630,500
21. The following information pertains to a sale of real estate by RR Co. to SS Co. on December 31,2012:
Carrying amount P2,000,000
Sales price:
Cash P300,000
Purchase money mortgage 2,700,000 3,000,000
The mortgage is payable in nine annual installments of P300,000 beginning December 31,2013 plus interest of 10%. The December 31,2013
installment was paid as scheduled,together with interest of P270,000. RR uses the cost recovery method to account for the sale. What
amount of income should RR recognize in 2013 from the real estate sale and its financing?
a. P570,000
b. P370,000
c. P270,000
d. P0
22. Action Inc. sold a fitness equipment on installment basis on October 1,2013. The unit cost to the company was P60,000 but the installment
selling price was set at P85,000. Terms of payment included the acceptance of a used equipment with a trade-in value of P30,000. Cash of
P5,000 was paid in addition to the traded-in equipment with the balance to be paid in ten monthly installments due at the end of each month
commencing the month of sale.
It would require P1,250 to recondition the used equipment so that it could be resold for P25,000. A 15% gross profit was usual from sale of
used equipment. The realized gross profit from the 2013 collections amounted to
a. P4,000
b. P34,000
c. P10,000
d. P8,000
23. M & J Corp. which sells goods on installment basis, recognizes at year end gross profit on collections which is consisted of cost and gross
profit. It reported the following:
January 1 December 31
Installment receivables
2011 P120,100 0
2012 1,722,300 P337,200
2013 0 2,050,450
Sales and cost of sales for the three years are as follows:
2011 2012 2013
Sales P1,900,000 P2,610,000 P3,010,0000
Cost of sales 1,235,000 1,425,000 1,896,300
In 2013 the company repossessed merchandise with resale value of P8,500 from customers who defaulted in payments. The sales were made
in 2012 for P27,000 on which P16,000 was collected prior to default. As collections are made, the company debits cash and credits installment
receivable. For default and repossessions, the company debits installment receivable. The amount of adjustment on the inventory of
repossessed merchandise to the extent of the unrealized gross profit was
d. Zero
e. A decrease of P6,240
f. A decrease of P2,500
e. A decrease of P3,740
25. On October 2013, Haybol Realty Co. sold to Mae Balay a property for P500,000 which is carried in its books for P250,000. The company
received P100,000 on the date of the sale and a mortgage note for P400,000 payable in twenty (20) semiannual installments of P20,000 plus
interest on the unpaid principal at 16% per annum.
The realized profit to be recognized by Haybol Realty Corp. in 2013 if gross profit is recognized periodically in proportion to
collections would be
a. P50,000
b. P100,000
c. P60,000
d. P250,000
26. Quincy Enterprises uses the installment method of accounting and has the following data at year-end:
Gross margin on cost 66 2/3%
Unrealized gross profit P192,000
Cash collection including down payments 360,000
What was the total amount of sale on installment basis?
e. P480,000
f. P648,000
g. P552,000
h. P840,000
26. The Brownout, Inc. began operating at the start of the calendar year 2013 uses the installment method of accounting:
Installment sales P400,000
Gross margin based on cost 66 2/3%
Inventory, Dec. 31,2013 80,000
General and administrative expenses 40,000
Accounts receivable, Dec. 31,2013 320,000
The balance of the deferred gross profit account at December 31,2013 should be:
a. P192,000
b. P96,000
c. P128,000
d. P80,000
27. Tear Drops Corp. started operations on 1 January 2012 selling home appliances and furniture on installment basis. For 2012 and 2013 the
following represented operational details.
In thousand Pesos
2012 2013
Installment sales P1,200 P1,500
Cost of installment sales 720 1,050
Collections on installment sales
2012 630 450
2013 0 900
On 7 January 2013, an installment sale account in 2010 defaulted and the merchandise with a market value of P15,000 was repossessed. The
related installment receivable balance as of date of default and repossession was P24,000.
The balance of the unrealized gross profit as of the end of 2013 wa
a. P218,400
b. P192,000
c. P360,000
d. P275,000
28. Four J Co. sold goods on installment. For the year just ended the following were reported:
Installment sales P3,000,000
Cost of installment sales 2,025,000
Collections on installment sales 1,800,000
Repossessed accounts 200,000
Fair market value of repossessions 120,000
a. (P15,000)
b. P15,000
c. (P80,000)
d. P5,000
29. A refrigerator was sold to Fernandina Castro for P16,000, which included a 40% markup on selling price. She made a down payment of 20%,
payment of four of the remaining 16 equal payment and defaulted on further payments. The refrigerator was repossessed, at which time the
fair value was determined to be P6,800.
The repossession resulted to the following (loss) gain:
a. P(1,040)
b. P1,040
c. P4,056
d. P2,960
30. The Company uses the installment method of accounting to recognize income, Pertinent data are as follows:
2011 2012 2013
Installment sales P300,000 P375,000 P360,000
Cost of sales 225,000 285,000 252,000
Balances of Deferred Gross Profit at Year end
2011 P52,500 P15,000 P-
2012 - 54,000 9,000
2012 - - 72,000
The total balance of the Installment Accounts Receivable on December 31,2013 is:
a. P270,000
b. P277,500
c. P279,500
d. P300,000
31. In its first year of operations, Guijo Company’s sales were as follows:
Sales basis Mark-up on cost Sales
Cash 25% P250,000
Charge 33-1/3% 400,000
installment 50% 600,000
The cost of goods sold for the year was P900,000.
No. 31 – Continued
If collections on installment sales during the year amounted to P240,000, how much was the total gross profit realized at the end of the year?
a. P50,000
b. P60,000
c. P80,000
d. P230,000
32. A sale on installment basis was made in 2013 for P8,000 at a gross profit of P2,800. At the end of 2013, when the installment account
receivable had a balance of P3,500, it was ascertained that the customer would be unable to make further payments. The merchandise was
then repossessed and was appraised at a value of P1,500. The loss on repossession was:
a. P3,500
b. P2,000
c. P775
d. P1,775

33. On January 1,2012 Blim Company commenced its sales of gas stoves. Separate accounts were set up for installment and cash sales, but
perpetual inventory record was not kept. On the installment sales of a down payment of 1/3 was required, with the balance payable in 18
equal monthly installments.

The transactions of the Blim Company are as follows:


2012 2013
Sales:
New gas stoves for cash P27,000 P37,000
New gas stoves on installment
(including the 1/3 cash 235,000 330,000
down payment)
Purchases 193,000 215,000
Physical inventories at
December 31:
New gas stoves at cost 45,500 60,000
Cash collections on installment contracts, exclusive of down payments:
2012 sales 54,000 77,000
2013 sales - 70,000
No. 33 – Continued
The realized gross profit for the year 2013 that would be reported on the income statement amounted to:
a. P131,530
b. P140,000
c. P123,350
d. P131,500
34. The data below are taken from the records of Jess Appliance Co., which sells appliances exclusively on the installment basis.
2011 2012 2013
Installment sales P365,500 P417,800 P610,750
Gross profit 36% 39% 40%
The balance in the Installment Accounts Receivable controlling accounts at the beginning and end of 2013 were:
2013
From sales made in: January 1 December 31
2011 P17,400 P-
2012 205,400 25,800
2013 - 305,520
There was one repossession recorded during 2013, it related to a 2012 sale. The repossessed appliance was sold at its fair value of P200,
which equaled the uncollected balance in the customer’s installment accounts receivable.
The total realized gross profit on prior year sales on December 31, 2013 and the gain (loss) from the sale of the repossesses appliance are:
a. P76,230 and P(78)
b. P76,230 and P78
c. P69,966 and P78
d. P75,230 and P78
35. Mr. Matias Manuel is a dealer in appliance who sells on an installment basis. A refrigerator which originally cost P924 was sold by him for
P1,650 to Jose Santos who made a down payment of P220, but defaulted in subsequent payments.
No. 35 – Continued
Mr. Manuel repossessed the refrigerator at an appraised value of P460. To improve its salability, he expended P60 for reconditioning. He
was able to sell the refrigerator to Pedro Reyes for P1,000 at a down payment of the first installment of P250.
The realized gross profit from the first installment sale (to Jose Santos) and from the second installment sale (to Pedro Reyes) are:
a. P96.80 and P100
b. P26.40 and P120
c. P96.80 and P120
d. P26.40 and P100
36. The Bengal Furniture Company appropriately used the installment sales method in accounting for the following installment sale. During
2013 Bengal sold furniture to an individual of P3,000 at a gross profit of P1,200. On June 1 2013, this installment account receivable had a
balance of P2,200 and it was determined that no further collections would be made. Bengal therefore repossessed the merchandise. When
reacquired, the merchandise was appraised as being worth only P1,000. In order to improve its salability, Bengal incurred costs P100 for
reconditioning. What should be the loss on repossessions attributable to this merchandise?
a. P220
b. P320
c. P880
d. P1,100
37. Standard Sales Corporation accounts for sales on the installment basis. The balances of control accounts for Installment Contracts
Receivable at the beginning and end of 2013 were:
Jan. 1,2013 Dec. 31,2013
Installment contract receivable - 2011 P24,020 -
Installment contract receivable – 2012 344,460 P67,440
Installment contract receivable – 2013 - 410,090
No. 37 – continued
During 2013, the company repossessed a refrigerator which had been sold in 2012 for P5,400 and P3,200 had been collected prior to
default. The company sales and cost of sales figures are summarized below:

2011 2012 2013


Net sales P380,000 P432,000 P602,000
Cost of sales 247,000 285,120 379,260

The resale price of the repossessed merchandise is P2,000 after reconditioning cost of P200 and a normal gross profit of 35%.

The total realized gross profit on December 31,2013 and the gain (loss) on repossession are:

a. P172,892.5 and P(381)


b. P172,852.5 and P(452)
c. P142,500 and P(452)
d. P142,500 and P452
38. The 680 Appliance Company reports gross profit on the installment basis. The following data are available:

2011 2012 2013


Installment sales P240,000 P250,000 P300,000
Cost of goods – installment sales 180,000 181,250 216,000
Gross profit 60,000 68,750 84,000

Collections:
2011 installment contracts P45,000 P75,000 P72,500
2012 installment contracts 47,500 80,000
2013 installment contracts 62,500
Defaults:
Unpaid balance of 2011
Installment contracts P12,500 P15,000
Value assigned to repossessed
Merchandise 6,500 6,000
Unpaid balance of 2012
Installment contracts 16,000
Value assigned to repossessed
Merchandise 9,000
No. 38 - Continued
The total realized gross profit after loss on repossession for 2013 is:
a. P49,775
b. P57,625
c. P48,975
d. P56,625
39. Partial trial balance of Lakan Appliance Corporation as of the end of the fiscal year September
30,2013 follows:

Debit Credit
Deferred gross profit – 2012 P50,000

Installment account receivable - 2012 P12,500


Installment account receivable – 2013 150,000
Installment sales 375,000
Inventory, September 30,2012 62,500
Loss on repossession 3,750
Purchases 435,000
Repossessions 2,500
sales 312,500
The post closing trial balance on September 30,2012 shows the following balances of certain accounts:

Installment contract receivable - 2012 P100,000


Deferred gross profit – 2012 50,000
The gross profit rate on regular sales during the year was 30%
The inventory of new and repossessed merchandise on September 30,2013 amounted to P75,000. Unpaid balance on repossessed
merchandise sale of 2012 is P6,250.
The total realized gross profit on December 31,2013 is:
a. P141,875
b. P101,250
c. P40,625
d. P140,875
40. Carlos Labung Appliance Co., sold a stove, costing P1,000 for P1,600 on September 2012. The down payment was P160, and the same
amount was to be paid at the end of each succeeding month. Interest was charged on the unpaid balance of the contract at ½ of 1% a
month, payments being considered as applying first to accrued interest and the balance to principal.
After paying a total of P640, the customer defaulted. The stove was repossessed in February 2013. It was estimated that the stove had a
value of P560 on a depreciated cost basis.
The realized gross profit and the gain (loss) on repossession on December 31,2013 are:
a. P232.76 and P(52.07)
b. P240.00 and P(52.07)
c. P232.76 and P(40.00)
d. P240.00 and P(40.00)
41. The Julia Appliance company makes all sales on installment contracts and accordingly reports income on the installment basis. Installment
contracts receivables are accounted for by years. Defaulted contracts are recorded by debiting Loss on Repossession account and
crediting the appropriate Installment Contract Receivable account for the unpaid balance at the time of
default. All repossessions and trade-ins are recorded at realizable values. The following data
relate to the transactions during 2012 and 2013

2012 2013
Installment sales P150,000 P198,500
Installment contract receivable, Dec. 31:
2012 sales 80,000 25,000
2013 sales 95,000
Purchases 100,000 120,000
New merchandise inventory, Dec. 31 at cost 10,000 26,000
Loss on repossessions 6,000
The company auditor disclosed that the inventory taken on December 31,2013 did not include certain merchandise received as a trade-in on
December 2,2013 for which an allowance was given. The realizable value of the merchandise is P1,500 which was also the allowance on the
trade-in. No entry was made to record this merchandise on the books at the time it was received. In 2013, a 2012 contract was defaulted and
the merchandise was repossessed. At the time of default, the repossessed merchandise had a fair value of P2,500. The repossessed
merchandise was neither recorded nor included in the physical inventory on December 31,2013.
The total realized gross profit at December 31,2013 and the adjusted gain (loss) on repossession are:
Realized Gross profit Gain(Loss) on repossesion
a. P70,000 P1,100
b. P70,000 (P1,100)
c. P50,400 P1,100
d. P50,400 (P1,100)
42. Kanlaon Corporation started operations on January 1,2012, selling home appliances and furniture sets both under cash and under installment
basis. Data on the installment sales operations for the two years ended December 31, 2012 and 2013 are as follows:

2012 2013
Installment sales P400,000 P500,000
Cost of installment sales 240,000 350,000
Cash collections on:
2012 installment contracts 210,000 150,000
2013 installment contracts - 300,000
The balance of the Deferred Gross profit account on December 31,2013 is:
a. P130,000
b. P160,000
c. P190,000
d. P76,000
43. United Trading accounts for sales under the installment method. On January 1,2013 its ledger accounts included the following balances:

Installment Receivable, 2011 P38,500


Installment Receivable, 2012 155,000
Deferred Gross Profit, 2011 11,550
Deferred Gross Profit, 2012 62,000
Installment sales in 2013 were made at a 42% gross profit rate. December 31,2013 account balances before adjustments were as follows:

Installment Receivable, 2011 P-0-


Installment Receivable, 2012 42,000
Installment Receivable, 2013 100,500
Deferred Gross Profit, 2011 11,550
Deferred Gross Profit, 2012 62,000
Deferred Gross Profit, 2013 75,810
The total realized gross profit on December 31,2013 is:
a. P90,350
b. P97,510
c. P98,910
d. P97,350
44. Presented below is the unadjusted trial balance, as of December 31,2013 of Moslim Products Corporation:
Cash P5,000
Installment Accounts Receivable - 2012 40,000
Installment Accounts Receivable - 2013 140,000
Inventory, December 31,2013 200,000
Other Assets 497,000
Trade Accounts Payable P50,000
Unrealized Gross Profit - 2011 10,000
Unrealized Gross Profit – 2012 86,000
Unrealized Gross Profit – 2013 100,000
Capital stock 600,000
Retained Earnings 80,000
Repossession Gain 6,000
Operating expenses 50,000 ________
P932,000 P932,000
The cost of goods sold had been uniform over the years at 60% of sales, and the company adopts perpetual inventory procedures. On the
installment sales, the company charges installment accounts receivable and credits inventory and unrealized gross profit accounts.
Repossessions of merchandise have been made during 2013 due to some customers’ failure to pay maturing installments. The analysis of
these transactions have been summarized as follows:
Inventory P7,500
Unrealized gross profit - 2011 800
Unrealized gross profit – 2012 2,400
Installment accounts receivable - 2011 2,000
Installment accounts receivable – 2012 6,000
Repossession gain 2,700
The repossessed merchandise were unsold at December 31,2013 and it was ascertained that these were booked, upon repossession, at their
original cost. A fair valuation would be a sales price of P10,000 after recorditioning cost of P1,000 and a normal gross profit.
The realized gross profit from 2013 sales and the gain (loss) on repossession on December 31,2013 are:
a. P44,000 and (P200)
b. P44,000 and P200
c. P56,000 and P300
d. P56,000 and P200
45. The following selected accounts appeared in the trial balance of Union Sales as of December 31,2013

Debit Credit
Installment Accounts Receivable, 2012 sales P15,000
Installment Accounts Receivable, 2013 sales 200,000
Inventory, December 31,2012 70,000
Purchases 555,000
Repossessions 3,000
Regular Sales P385,000
Installment sales 425,000
Unrealized Gross Profit, 2012 54,000
Additional information:
Installment Accounts Receivable, 2012 sales,
As of December 31,2012 P120,000
Inventory of new and repossessed
Merchandise, December 31,2013 95,000
Gross profit rate on regular sales during the year 30%
Repossession was made during the year on a 2012 sale and the corresponding uncollected amount at the time of repossession was P7,750.
The total realized gross profit on December 31,2013 and the (loss) on repossession are:
a. P85,5000 and P(1,262.5)
b. P129,262.5 and P(1,262.5)
c. P43,762.5 and P1,262.5
d. P119,622.5 and P1,262.5
46. The books of Paiyakan Company show the following account balances on December 31,2013:

Accounts receivable P313,750


Deferred gross profit (before adjustment) 38,000
Analysis of the accounts receivable reveals the fo llowing:
Regular accounts P207,500
2012 installment accounts receivable 16,250
2013 installment accounts receivable 90,000
Sales on installment basis in 2012 were made at 30% above cost, and in 2013 at 33-1/3% above cost. Expenses paid relating to installment
sales were P1,500.
How much is the total comprehensive income on installment sales?
a. P10,000
b. P10,250
c. P11,000
d. P11,500
47. The Famcor Sales Company employs the perpetual inventory basis in the accounting for new cars. On August 15,2012, a new car costing
P165,000 and with a list price of P220,000 was sold to Rose Castro. The company granted Ms. Castro an allowance of P85,000 on the trade-in
of her old car, the current value if which was estimated to be P81,700; the balance of P135,000 was payable as follows: P35,000 cash at the
time of purchase and twenty monthly payments of P5,000 starting September 1, 2012.
On April 1,2013, Ms. Castro defaulted in the payment of the March 1,2013, installment. The new car sold was repossessed, and its value to
the seller was P40,000.
The total realized gross profit and the gain (loss) on repossession on December 31,2013 are:
a. P32,616.62 and P(13,298)
b. P32,616.62 and P13,298
c. P37,388.62 and P15,810.62
d. P27,844.62 and P(15,810.62)
48. The Jade Appliances Company started business on January 1,2013. Separate accounts were established for installment and cash sales. On
installment sales, the price was 106% of the cash sales price. A standard installment contract was used whereby a down-payment of ¼ of the
installment price was required, with the balance payable in 15 equal monthly installment. (the interest charge per month is 1% of the unpaid
cash sale price equivalent at each installment.)
Installment receivable and installment sales were recorded at the contact price. When contracts were defaulted, the unpaid balances were
charged to Bad Debts Expense. The following data are available:

Sales:
Cash sales P126,000
Installment sales 265,000
Repossessed sales 230
Inventory, January 1,2013:
Merchandise inventory 58,060

Purchases, 2013
New merchandise 209,300
Inventories, physical, December 31,2013
New merchandise 33,300
Repossessed inventory 180

Cash collections on installment contract 2013:


Down payments 66,250
Subsequent installments (including interest of P9,252.84 on
all contracts except on defaulted contracts) 79,341
Five contracts totaling P1,060 were defaulted, in each case after 3 monthly installments were paid.
Interest should be recognized in the period earned.
The total realized gross profit on December 31,2013 is:
a. P99,024.85
b. P99,084.87
c. P99,184.85
d. P95,024.85
49. The following data were taken from the records of Camille Appliance Company before its accounts were closed for the year 2013. The
company sells exclusively on the installment basis and its uses the installment method of recognizing profit:

2009 2010 2011


Installment sales P400,000 P440,000 P420,000
Cost of installment sales 240,000 272,800 256,200
Operating expenses 100,000 94,000 96,000
Balances as of December 31:
Inst. Contracts Receivable -2011 220,000 110,000 28,000
Inst. Contracts Receivable -2012 250,000 92,000
Inst. Contracts Receivable -2013 238,000
During 2013, because some customers can no longer be located, the company wrote off P9,000 of the 2011 installment accounts and P2,800
of the 2012 installment accounts as uncollectible.
Also during 2013, a customer defaulted and the company repossessed merchandise appraised at P2,400 after costs reconditioning estimated
at P400. The merchandise had been purchased in 2011 by a customer who still owed P5,000 at the date of the repossession.
The total comprehensive income on December 31,2013 is:
a. P157,156
b. P61,000
c. P60,156
d. P59,156
50. Jing Trading Company, which started operations on January 2,2012, sells video equipment on installment terms. Whenever a contract is in
default, Jing repossesses the merchandise and writes this off to a Loss on Defaulted Contracts account. Information regarding the repossessed
goods are not recorded in the books but are kept on a memo basis. Proceeds from the sale of these goods are credited to the Loss on
Defaulted Contracts account. The following information are taken from the books of Jing:
December 31
2013 2012
Installment contracts receivable, 2012 P2,000 P31,500
Installment contracts receivable, 2013 40,000 -
Sales 125,000 75,000
Loss on defaulted contracts 4,275 250
Allowance for defaulted contracts 2,250 2,250
Additional information:
a. No repossessed video equipment was sold in 2012 or 2013 for more than the unpaid balance of the original contract. A further analysis
of the Loss on Defaulted Contracts accounts showed the following breakdown:
2012 2013
Contracts Contracts
Contracts written off P3,750 P1,500
Less: sales of repossessed goods 800 175
Loss a defaulted contracts P2,950 P1,325
The repossessed goods on hand on December 31,2013, all of which were repossessed from 2012 contracts, are valued at P200.
b. The P2,000 balance of the Installment Contracts Receivable 2012 account is currently due and collectible.
c. The gross profit rates on installment sales were 40% in 2012 and 42% in 2013.
d. The rate of bad debts loss for 2013 is estimated to be the same as the 2012 experiences rate based on sales:
The required balance of the allowance for Defaulted Contracts account and the realized gross profit on December 31,2013 from 2012
sales are:
a. P3,675 and P10,300
b. P3,675 and P9,300
c. P3,675 and P10,300
d. P4,675 and P9,300

ANSWERS

1. C 11. A 21. D 31. D 41. B


2. C 12. A 22. D 32. C 42. D
3. B 13. A 23. D 33. A 43. A
4. A 14. B 24. A 34. B 44. B
5. C 15. C 25. D 35. C 45. B
6. C 16. A 26. C 36. B 46. B
7. D 17. A 27. A 37. B 47. A
8. D 18. D 28. A 38. A 48. A
9. C 19. A 29. B 39. A 49. C
10. A 20. A 30. B 40. A 50. A
SOLUTIONS AND EXPLANATIONS
1. The answer can be computed by using the basic formula, collections x gross profit rate.
2012 sales 2013 sales
Collections during 2013 P600,000 P700,000
Gross profit rate 30% 40%
Realized gross profit P180,000 P280,000
Total realized gross profit (P180,000 + P280,000) 460,000
2. Installment account receivable, 12/31/13 P200,000
Gross profit rate 40%
Deferred gross profit, December 31,2013 P80,000
3. Installment sales P500,000
Collections 100,000

Installment accounts receivable, 12/31/13 400,000


Gross profit rate (P250,000/P500,000) 50%
Deferred gross profit, 12/31/13 P200,000

Or
Deferred gross profit(P500,000 – P250,000) 250,000
Realized gross profit, 12/31/13 (P100,000x50%) 50,000
Deferred gross profit, 12/31/13 P200,000
4. Realized gross profit (P2,020,000 x 40%) P808,000

Deferred gross profit, 12/31/13:


Installment accounts receivable, 12/31/13
(P16,000,000 - P2,020,000) P13,980,000
Gross profit rate 40%
Deferred gross profit, 12/31/13 P5,592,000
5. Regular sales P600,00
cost of regular sales 300,000

Gross profit on regular sales P300,000


Realized gross profit on installment sales:
Collections P200,000
Gross profit rate (P500,000/P1,000,000) 50% 100,000

Total realized gross profit 400,000


General and administrative expense 100,000
Total comprehensive income P300,000
6.
Installment sales P800,000
Cost of installment sales 480,000

Deferred gross profit 320,000


Realized gross profit (P300,000 x 40%*) 120,000
Deferred gross profit, 12/31/13 P200,000

*Gross Profit Rate (P320,000/P800,000) = 40%


7. Installment sales P60,000
Less: Collections P35,000
Accounts written off 5,000 40,000

Installment accounts receivable, 12/31/13 20,000


Gross profit rate 30%
Deferred gross profit, 12/31/13 P6,000

8. Installment sales (P560,000/40%) P1,400,000


Less: installment accounts receivable, 12/31/13 800,000

Collections P600,000
Gross profit rate 40%
Realized gross profit P240,000

9.
2012 Sales 2013 Sales Total
Installment sales P1,000,000 P2,000,000
Collections (RGP/GPR)
During 2010 (P150,000/P30%) (500,000)
During 2011:
2010 sales (P90,000/30%) (300,000)
2011 sales (P200,000/40%) ________ (500,000)
Installment accounts receivable 12/31/13 P500,000 P1,200,000 P1,700,000

10. Deferred gross profit (gain) P270,000


Realized gross profit:
Down payment P150,000
Installment collections excluding interest:
(P325,000 – P75,000) 250,000

Total collections 400,000


Gross profit rate (P270,000/P900,000) 30% 120,000
Deferred gross profit, 12/31/13 P150,000
11. Installment accounts receivable, January 1 P755,000
Installment sales 950,000

Total P1,705,000
Less: Installment accounts receivable, Dec. 31 840,000

Collections 865,000
Gross profit rate (P339,750/P755,000) 45%
Realized gross profit 389,250

Installment accounts receivable, December 31 P840,000


Gross profit rate 45%
Deferred gross profit, December 31 P378,000
12. (1) Profit is recognized at the point of sale
Fair value of repossessed property P1,150,000
Less: Unrecovered cost (unpaid balance) 2,000,000
Loss on repossession P(850,000)
(2) Profit is recognized in proportion to collections
Fair value of repossessed property P1,150,000
Less: Unrecovered cost
Unpaid balance P2,000,000
Deferred gross profit (P2,000,000 x 40%) 800,000 1,200,000
Loss on repossession P(50,000)

13. (1) total realized gross profit


2013 2012 2011
Installment accounts receivable, 1/1/13 P12,890,625 P8,387,500 P1,512,500
Installment accounts receivable, 12/31/13 9,728,125 3,025,000 -0-
Collections during 2013 P3,162,500 P5,362,500 P1,512,500
Gross profit rates 32% 30% 28%
Realized gross profit, 12/31/13 P1,012,000 P1,608,750 P423,500

(Total, P3,044,250)
(2) deferred gross profit, December 31,2013:
2013 2012 2011
Installment accounts receivable, 12/31/13 P9,728,125 P3,025,000 P-0-
Gross profit rates 32% 30% 28%
Deferred gross profit, 12/31/13 P3,113,000 P907,500 P-0-
14. (1) Total realized gross profit
2011 2012 2013
Deferred gross profit before adjustment P10,500 P28,900 P96,000
Deferred gross profit, end:
2011 sales (P12,000 x 35%) 4,200
2012 sales (P40,000 x 34%) 13,600
2011 sales (P130,000 x 32%) ______ _______ 41,600
Realized gross profit, 12/31/13 P6,300 P15,300 P54,400

Total (P76,000)
(2) Total collections in 2013
2011 2012 2013
Installment accounts receivable, beg
2011 sales (P10,500/35%) P30,000
2012 sales (P28,900/34%) 85,000
2011 sales (P96,000/32%) P300,000
Installment accounts receivable, end 12,000 40,000 130,000
Collections during 2013 P18,000 P45,000 P170,000

Total (P233,000)

15.

Installment sales P1,000,000


Collections (150,000)
Accounts written off (50,000)

Installment accounts receivable, 12/31/13 800,000


Gross profit rate (P500,000/P1,000,000) 50%
Deferred gross profit, 12/31/13 P400,000
16. The balance of Deferred Gross Profit Account on December 31,2013 is computed follows:
2012 2013
Sales P300,000 P450,000
Collections (150,000) (150,000)
Accounts written off (100,000) (150,000)
Installment accounts receivable, 12/31/13 P50,000 P150,000
Gross profit rates 30% 40%
Deferred gross profit, 12/31/13 P15,000 P60,000

Total (P75,000)

17. (1) Realized gross profit, December 31,2013


Regular Sales P385,000
Cost of regular sales (70%) 269,500

Gross profit on regular sales (30%) 115,500


Realized gross profit on installment sales (Sched 1) 128,510
Total realized gross profit P245,010
Schedule 1:
2012 2013
Installment accounts receivable, 1/1/13 P120,000 P425,000
Installment accounts receivable, 12/31/13 15,000 200,000
Total credit 105,000 225,000
Less: credit for repossession (unpaid balance) 7,200 -0-
Collections P97,800 P225,000
Gross profit rates:
2012 sales (P54,000/P120,000) 45%
2013 sales (Schedule 2) _______ 38%
Realized gross profit, 12/31/13 P44,010 P85,500

Total (P129,510)

Schedule 2:
Installment sales P425,000
Cost of installment sales:
Inventory, January 1,2013 P70,000
Purchases 555,000
Inventory, December 31,2013 (New) (92,000)
Cost of sales 533,000
Cost of regular sales 269,500 263,500
Gross profit on installment sales P161,500
Gross profit rate (P161,500/P425,000) 38%
(2) loss on repossession
Repossession merchandise P3,000
Unrecovered cost:
Unpaid balance P7,200
Deferred gross profit (P7,200 x 45%) 3,240 3,960
Loss on repossession P(960)
18. Total realized gross profit on December 31,2013 is computed below:
2012 2013
Collections during 2013 P240,000 P160,000
Gross profit rates on sales 25%/125% 20%/120%
Realized gross profit P48,000 P26,667

Total (P74,667)
19.
Collections during 2013 P32,000
Gross profit rate:
Installment sales:
Notes receivable (P32,000 + P62,000 + P3,600) P97,600
Unearned interest income (P7,167 + P3,600) (10,767)
Installment sales P86,833
Cost of installment sales (P45,200 – P2,000) 43,200
Gross profit P43,633
Gross profit rate (P46,633/ P86,833) 50.25%
Realized gross profit P16,080
20. Collections during 2013 (P1,510,000 + P1,900,000 – P1,680,000) P1,730,000
Gross profit rate (P679,500/ P1,510,000) 45%
Realized gross profit, 2013 P778,500
21 Zero, because the total cost of P2,000,000 is not yet fully recovered. The total collections applying to principal as of December 31, 2013 is
only P330,000 (P300,000 + P30,000), so no income is yet to be recognized.
22. First the over- allowance on the equipment traded- in should be computed as follows:
Trade- in value P30,000
Actual value:
Estimated sales price 25,000
Less: Reconditioning Cost 1,250
Gross profit(25,000 x 15%) 3,750 5,000 20,000
Over allowance P10,000
The over allowance is treated as a deduction from the selling price of new equipment.
The realized gross profit can now be computed as show below:
Collections
Downpayment:
Cash 5,000
Actual value of Trade- in 20,000 25,000
Installment collection (3 mos. X 5,000) 15,000
Total 40,000
Gross Profit Rate – (15,000/ 75,000) 20%
Realized gross profit, 12/31/2013 8,000
23. the unrealized gross profit relating to the unpaid balance of P11,000 (P27,000-P16,000) is 3,740 (11,000x34%). The inventory of repossessed
merchandise is to be decreased by this amount.
24. Collection during 2013 100,000
Gross profit rate (250,000/500,000) 50%
REALIZED GORSS PROFIT 50,000
25. Installment accounts receivable-end:
Unrealized gross profit-end 192,000
Divide by GPR on sales (66-2/3% / 116-2/3%) 40% 480,000
ADD: Collections 360,000
Installment Sales P840,000
26. Installment accounts receivable P320,000
Gross Profit Rate on Sales (66- 2/3% / 166-2/3%) 40%

Deferred gross profit, 12/31/2013 P128,000


27.

In Thousand Pesos

2012 Sales 2013 Sales

Installment sales P1,200 P1,500


Collection:

During 2012 (630)

During 2013 (450) (900)

Repossession (unpaid balance) (24) -

Installment accounts receivable, 12/31/2013 96 600

Gross Profit rate (GP/IS) 40% 30%

Deferred Gross Profit, 12/31/2013 P38.4 P180

Total balance is P218,400 (P38,400 + 180,000)

28. Fair market value of repossessed merchandise P120,000

Less: Unrecovered cost 200,000

Unpaid balance 65,000 135,000

Loss on repossession P(15,000)


29. Fair value of repossessed merchandise P6,800

Unrecovered Cost:

Unpaid balance:

Sales 16,000

Collections:

Downpayment 3,200

Installment 3,200 6,400 9,600

Deferred gross profit (9,600 x 40%) 3,840 5,760

Gain on repossession P1,040

30.

2012 Sales 2013 Sales

Deferred gross profit – Dec.31,2013 P9,000 P72,000

Divide by GPR (GP/IS) 24% 30%

Installment accounts receivable, Dec.31,2013 P37,500 P240,000

Total balance of receivable on Dec. 31,2013 is

(P37,500 + 240,000) P277,500

31. Gross profit rate based on sales:

Cash (25%/125%) 20%

Charge (33-1/3% / 133-1/3%) 25%

Installment (50% - 150%) 33.33%

Total realized gross profit:

Cash sales (250,000 x 20%) P50,000


Charge sales (400,000 x 25%) 100,000

Installment Sales (240,000 x 33.33%) 80,000

Total P230,000

32. Appraised value of repossessed merchandise P1,500

Less: Unrecovered cost:

Unpaid balance 3,500

Less: Deferred gross profit (3,500 x 35% *) 1,225 2,275

Loss on repossession P775


*Gross profit rate (P2,800 / 8,000) 35%

33.

2012 2013

Sales Sales

Collections:

Downpayment (1/3 of sales) - P110,000

Collection of installment receivables 77,000 70,000

Total 77,000 180,000

Gross Profit rate (schedule 1) 44% 45%

Realized gross profit on Installment Sales 33,880 81,000

Realized gross profit on Cash Sales 2013 (P37,000 x 45%) - 16,650

Realized Gross Profit (P131,530) P33,880 97,650

Schedule 1 2013 2012

Sales Sales

Sales: Cash 37,000 27,000

Installment 330,000 235,000

Total 367,000 262,000

Cost of Sales:

Inventories, 1/1 45,500 -

Purchases 215,000 193,000

Total 260,500 193,000

Inventories 12/31 60,000 45,500

Cost of sales 200,500 147,500

Gross Profit P166,500 P114,500

Gross profit rate (GP/IS) 45% 44%

34. P76,230 represents the total realized gross profit based on 2013 collections of Installment Accounts Receivable of 2011 and 2012
sales.

2011 2012
Sales Sales

Collections:

Installment accounts receivable, 1/1/13 P17,400 P205,400


Installment accounts receivable, - 25,800

12/31/13

Total credits 17,400 179,600

Less: credit for repossession ______ 200

Collections during 2013 17,400 179,400

Gross profit rate 36% 39%

Realized gross proft, 12/31/13 P6,264 P69,966

Total realized gross profit:

(P6,264 + P69,966) P76,230

A P78 gain is realized from the sale of the repossessed merchandise as computed below:

Sales price P200 Unrecovered cost:

Unpaid balance P200

Less: deferred gross profit (P200 x 39%) 78 122

Gain on repossession P78

35. on the first installment, a profit of P96.80 is realized which is computed as follows:

Installment sales P1,650

Cost of sales 924

Gross profit P726

Gross profit rate 44%

Realized gross profit:

Collections P220

Gross profit rate 44%

Realized gross profit P96.8


On the second installment, a profit of P120 is realized as shown below:

Sales P1,000
Cost of repossessed merchandise:

Appraised value P460

Add: reconditioning cost 60 520


gross profit P480

Gross profit rate (P480/P1,000) 48%

Realized gross profit:

Collections: P250

Gross profit rate 48%

Realized gross profit P120


36. Appraised value of repossessed merchandise
P1,000
Unrecovered cost:
Unpaid balance P2,200
Less: deferred gross profit (P2,200 x 40%) 880 1,320
Loss on repossession (P320)

Gross profit rate (P1,200 + P3,000 ) =40%


37. The realized gross profit is computed as follows:
Year of sales
2011 2012 2013
Installment contract receivable, 1/1/13 P24,020 P344,460 P602,000
Installment contract receivable, 12/31/13 - 67,440 410,090
Total credit 24,020 277,020 191,910
Credit for repossession ______ 2,200 _______
Collections 24,020 274,820 191,910
Gross profit rate:
2011: 133,000/380,000 35%
2012:146,880/432,000 34%
2013:22,740/602,000 ______ _______ 37%
Realized gross profit P8,407 P93,438.8 P71,006.7
Total realized gross profit, 12/31/13:
2011 P8.407
2012 93,438.80
2013 71,006.70
Total P172,852.5
The loss on repossession is computed as follows:
Actual value of repossession merchandise:
Resale price P2,000
Less: Reconditioning cost P300
Gross profit (P2,000 x 35%) 700 1,000 P1,000

Unrecovered cost
Unpaid balance (P5,400-P3,200) P2,200
Less deferred gross profit (P2,200 x 34%) 748 1,452
Loss on repossession P(452)
38. This is computed by deducting the loss on repossession from the total realized gross profit: Year of Sales
2011 2012 2013 Total
Collections P72,500 P80,000 P62,500
Gross profit rate
2011:P60,000/P240,000 25%
2012:P68,750/P250,000 27.5%
2013:P84,000/P300,000 ______ ______ 28%
Realized gross profit P18,125 P22,000 P17,500 57,625

Loss on repossession
Value of repossessed merchandise P6,000 P9,000
Unrecovered cost:
Unpaid balance 15,000 16,000
Less: deferred gross profit
2011:P15,000x25% 3,750
2012:P16,000x27% _____ 4,400
Unrecovered cost 11,250 11,600
Loss on repossession (P5,250) (P2,600) (7,850)
Total realized gross profit after loss on repossession P49,775
39. The computation is as follows:
Year of sales
2012 2013
Installment contract receivable, 1/1/13 P100,000 P375,000
Installment contract receivable, 12/31/13 (12,500) (150,000)
Total credit 87,500 225,000
Credit for repossession (6,250) -
Collections 81,250 225,000
Gross profit rate (schedule ) 50% 45%
Realized gross profit (P141,875) P40,625 P101,250

Schedule 1 : gross profit rate


2012 sales:
Gross = Deferred gross profit – 2012, 9/30/2012 P50,000 = 50%
profit rate Inst. Contract rec’ble – 2012, 9/30/2012 100,000

2013 sales:

Installment sales P375,000


Less: cost of installment sales-
Cost of goods sold:
Inventories, 9/30/12 P62,500
Purchases 435,000
Cost of goods available 497,500
Less: inventories, 9/30/12
(P75,000-P2,500) 72,500
Cost of goods sold 425,000
Less: cost of regular sales (70% x P312,500) 218,750 206,250
Gross profit on installment sales P168,750
Gross profit rate: (P168,750/P375,000) 45%

40. The realized gross profit is computed as follows:


Collections applying to principal (Sch. 1) P620.69
Gross profit rate (P600/P1,600) 37.5%
Realized gross profit rate P232.76

The loss on repossession is computed below:

Fair value of repossessed merchandise P560


Less: unrecovered cost
Unpaid balance (sch. 1) P979.31
Less: deferred gross profit (P979.31 x 37.5%) 367.24 612.07
Loss on repossession (P52.07)

Schedule 1:

Date (1) Total (2) Applying to (3) Applying to (4) Balance of


payment Interest 005 principal (1) principal (4)
x (4) – (2) – (3)
Sept. 30 P1,600
Sept. 30 P160 - P160 1,440
Oct. 31 160 7.20 152.8 1287.20
Nov. 30 160 6.44 153.56 1,133.64
Dec. 31 160 5.67 154.33 979.31
640 P19.31 P620.69

41. P70,000 is the sum of the realized gross profit in 2012 and 2013 which are computed as follows:
2012 2013
Installment contract receivable, P80,000 P200,000
beg. (1/1/13)
Installment contract receivable, 25,000 95,000
beg. (1/1/13)
Total credits 55,000 105,000
Less: credit for repossession 6,000 -
Collections 49,000 105,000
Gross profit rate (schedule 1) 40% 48%
Realized gross profit 12/31/13 (P70,000) P19,600 P50,400

The P1,100 adjusted loss is determined as follows:


Value of repossessed merchandise P2,500
Unrecovered cost:
Unpaid balance 6,000
Less: deferred gross profit (P6,000 x 40%) 2,400 3,600
Adjusted loss on repossession (P1,100)

Schedule 1 – gross profit rates:

2012 Sales:
Installment sales P150,000
Cost sales:
Purchases P100,000
Merchandise inventory, 12/31 10,000 90,000
Gross profit P60,000
Gross profit rate (P60,000/P150,000) 40%

2013 Sales:

Adjusted installment sales


(P198,500 + P1,500, Trade-in) P200,000
Cost of sales:
Merchandise inventory, 1/1 P10,000
Purchases 120,000
Goods available for sale 130,000
Merchandise inventory, 12/31 26,000 104,000
Gross profit P96,000
Gross profit rate (P96,000/P200,000) 48%

42. The balance of deferred gross profit on Dec. 31,2013 is computed as follows:

2012 2013
Sales Sales
Installment sales P400,000 P500,000
Collections in 2012 (210,000)
Collections in 2013 (150,000) (300,000)
Installment contract receivable, 12/31/13 40,000 200,000
Gross profit rate (GP/IS) 40% 30%
Deferred gross profit, 12/31/13 (P76,000) P16,000 P60,000

43. Deferred gross profit before adjustment:

2011 sales P11,550


2012 sales 62,000
2013 sales 75,810
Total 149,360
Less: deferred gross profit, end (IAR end X GPR)
2011 sales -
2012 sales (P42,000 x 40%) P16,810
2013 sales(P100,500 x 42%) 42,210 59,010
Total realized gross profit, 12/31/13 P90,350

2012 GPR: P62,000/P155,000 = 40%

44. The total realized gross profit is computed below:


2013 Installment sales:
Unrealized gross profit, 2013 P100,000
Divided by GPR on sales ÷ 40% P250,000
Less: Installment receivable – 2013,12/31/13 140,000
Collection from 2013 sales 110,000
Gross profit rate 40%
Realized gross profit on 2013 sales P44,000

The gain (loss) on repossession is computed as follows:

Actual value of repossessed


merchandise:
Sales price P10,000
Less: reconditioning cost P1,000
Gross profit (P10,000 x 40%) 4,000 5,000 P5,000
Less: unrecovered cost
Unpaid balance:
2011 accounts P2,000
2012 accounts 6,000 8,000
Deferred gross profit:
2011 account(P2,000 x 40%) 800
2012 account(6,000 x 40%) 2,400 3,200 4,800
Gain on repossession P200
45. Total realized gross profit is computed below:
Year of sales
2012 2013
Sales Sales
Installment receivable, 1/1/13 P120,000 P425,000
Installment receivable, 12/31/13 (15,000) (200,000)
Defaulted balance (7,750) -
Collections 97,250 225,000
Gross profit rates 45% 38%
Realized gross profit, 12/31/13 P43,762.50 P85,500

Total (P129,562.50)

Gross profit rate:


2012 sales (P54,000/P120,000) 45%
2013 sales P425,000
Installment sales
Cost of installment sale:
Inventory, 1/1 70,000
Purchases 555,000
Inventory, 12/31 (95,000)
Repossession 3,000
Total 533,000
Cost of regular sale (70% x P385,000) 269,500 263,500
Gross profit 161,500

GPR(P161,500/P425,000) 38%
The loss on repossession is computed as follows:
Value of repossessed merchandise P3,000
Less: unrecovered cost:
Unpaid balance P7,750
Deferred gross profit (7,750 x 45%) 3,487.50 4,262.50
Loss on repossession P1,262.50

46.
Deferred gross profit, before adjustment P38,000
Less: deferred gross profit applicable to
Uncollected installment accounts:
2012: P16,250 x 30%/130% P3,750
2013:P90,000 x 25% 22,500 26,250
Realized gross profit P11,750
Less: Expenses 1,500
Net income on installment sales P10,250
47. The computation of the realized gross profit is shown below:
List price P220,000
Less: trade-in overallowance P85,000-P81,700 3,300
Adjusted selling price P216,700 100%
Less: cost of sales 165,000 76.14%
Gross profit 51,700 23.86%
Value of old car trade-in P 81,700
Cash received at time of sale 35,000
Installment collected: P5,000 x4 20,000
Total collections in 2013 136,700
Multiply by gross profit rate .2386
Realized gross profit as of December 31,2013 P32,616.62

Gain (loss) on repossession is computed as follows:


Adjusted selling price P216,700
Less: collections
In 2012 (No.47) P136,700
In 2013: P5,000 x 2 10,000 146,700
Defaulted balance P70,000
Multiply by cost rate .7614
Unrecovered cost P53,298
Value of repossessed car P40,000
Less: unrecovered cost 53,298
Repossession gain (loss) P(13,298)

48.
Cash sales P126,000
Installment sales collected
Downpayment (P265,000 x ¼) P66,250
Subsequent installments P79,341
Less: interest (9,252.84)
Interest on defaulted contracts (sch.1) (20.67) 70,067.49 136,317.49
Total collection P262,317.49
Gross profit rate (sch.2) 37.75%
Realized gross profit, 12/31/13 P99,024.85
Schedule 1 – interest on defaulted contracts:

The total interest is determined through the use of the following table:
Installment (1) Equivalent (2) Contact (3) Interest (4) Cash
number cash sales 1- sales income1% collection
(4-3) price2-4 x1
First month P1,000 P1,060 265
Second month 735 795 7.35 53
Third month 689.35 742 6.89 53
Fourth month 689.35 689 6.43 53
Total interest earned 20.67

Schedule 2 – gross profit rate:

The 37.75% gross profit rate is determined as follows:


Sales: P126,000
Cash sales 250,000
Installment sales at cash 376,000
sales price (P265,000/106%)
Total sales at cash sales
price
Cost of sales:
Merchandise inventory, January 1 P58,060
Purchases 209,300
Goods available for sale 267,360
Less: merchandise inventory, Dec. 31 33,300 234,060
Gross profit 141,940
Gross profit rate (P141,940/P376,000) 37.75%

49.
Total realized gross profit (Sch.1) P157,156
Loss on repossession (Sch.2) (1,000)
Total realized gross profit loss on repossession 156,156
Operating expenses 96,000
Net income, Dec. 31,2013 P60,156

Schedule 1 – realized gross profit

2011 2012 2013


Sales Sales Sales
Inst. Contract receivable, 1/1/13 P110,000 P250,000 P420,000
Inst. Contract receivable, 21/31/13 (28,000) (92,000) (238,000)
Accounts written off (9,000) (2,800) -
Defaulted accounts (5,000) - -
Collections 68,000 155,200 182,000
Gross profit rate (GP/IS) 40% 38% 39%
Realized gross profit (P157,156) P27,200 P58,976 P70,980

Schedule 2 - loss o repossession:

Appraised value of repossessed merchandise P2,400


Less: reconditioning cots 400
Actual value at time of repossession 2,000
Less: unrecovered cost
Unpaid balance 5,000
Deferred gross profit (P5,000 x 40%) 2,000 3,000
Loss on repossession P(1,000)

51. The computation of the required balance of the allowance for defaulted contracts account is shown below:
2013 Bad debts rate
Loss on defaulted contracts P250
Contracts written off 3,750
Sales of repossessed goods (800)
Value of repossessed goods (200)
Total 3,000
Divided by 2012 sales ÷75,000
Rate of bad debt loss 4%

Estimated loss from 2013 sales (125,000 x 4%) P5,000


Less: loss on defaulted contract – 2013 sales 1,325
Required balance of allowance, Dec. 31,2013 P3,675

The realized gross profit on Dec. 31,2013 from 2012 Sales is computed below:
Installment contract receivable – 2012, 1/1/13 P31,500
Installment contract receivable – 2012, 12/31/13 (2,000)
Installment contract, receivable written off – 2012 sales (3,750)
Collections during 2013 25,750
Gross profit rate – 2012 40%
Realized gross profit from 2012 sales, 12/31/13 P10,300
1. Reese Construction Corporation contracted to construct a building for $1,500,000. Construction began in 2007 and was completed in 2008. Data relating
to the contract are summarized below:
Year ended
December 31,
2007 2008
Costs incurred $600,000 $450,000
Estimated costs to complete 400,000 —
Reese uses the percentage-of-completion method as the basis for income recognition. For the years ended December 31, 2007, and 2008, respectively,
Reese should report gross profit of
a. $270,000 and $180,000.
b. $900,000 and $600,000.
c. $300,000 and $150,000.
d. $0 and $450,000.
2. Winsor Construction Company uses the percentage-of-completion method of accounting. In 2007, Winsor began work on a contract it had received which provided
for a contract price of $15,000,000. Other details follow:
2007
Costs incurred during the year $7,200,000
Estimated costs to complete as of December 31 4,800,000
Billings during the year 6,600,000
Collections during the year 3,900,000
What should be the gross profit recognized in 2007?
a. $600,000
b. $7,800,000
c. $1,800,000
d. $3,000,000
In 2007, Crane Corporation began construction work under a three-year contract. The contract price is $2,400,000. Crane uses the percentage-of-completion method
for financial accounting purposes. The income to be recognized each year is based on the proportion of costs incurred to total estimated costs for completing the
contract. The financial statement presentations relating to this contract at December 31, 2007, follow:
Balance Sheet
Accounts receivable—construction contract billings $100,000
Construction in progress $300,000
Less contract billings 240,000
Costs and recognized profit in excess of billings 60,000

Income Statement
Income (before tax) on the contract recognized in 2007 $60,000

3. How much cash was collected in 2007 on this contract?


a. $100,000
b. $140,000
c. $20,000
d. $240,000

4. What was the initial estimated total income before tax on this contract?
a. $300,000
b. $320,000
c. $400,000
d. $480,000

5. Eaton Construction Co. uses the percentage-of-completion method. In 2007, Eaton began work on a contract for $3,300,000 and it was completed in 2008.
Data on the costs are:
Year Ended December 31
2007 2008
Costs incurred $1,170,000 $840,000
Estimated costs to complete 780,000 —
For the years 2007 and 2008, Eaton should recognize gross profit of
2007 2008
a. $0 $1,290,000
b. $774,000 $516,000
c. $810,000 $480,000
d. $810,000 $1,290,000
Use the following information for questions 66 and 67.
Ramos, Inc. began work in 2007 on contract #3814, which provided for a contract price of $7,200,000. Other details follow:
2007 2008
Costs incurred during the year $1,200,000 $3,675,000
Estimated costs to complete, as of December 31 3,600,000 0
Billings during the year 1,350,000 5,400,000
Collections during the year 900,000 5,850,000
6. Assume that Ramos uses the percentage-of-completion method of accounting. The portion of the total gross profit to be recognized as income in 2007 is
a. $450,000.
b. $600,000.
c. $1,800,000.
d. $2,400,000.

7. Assume that Ramos uses the completed-contract method of accounting. The portion of the total gross profit to be recognized as income in 2008 is
a. $900,000.
b. $1,350,000.
c. $2,325,000.
d. $7,200,000.
Use the following information for questions 68 and 69.
Miley, Inc. began work in 2007 on a contract for $8,400,000. Other data are as follows:
2007 2008
Costs incurred to date $3,600,000 $5,600,000
Estimated costs to complete 2,400,000 —
Billings to date 2,800,000 8,400,000
Collections to date 2,000,000 7,200,000
8. If Miley uses the percentage-of-completion method, the gross profit to be recognized in 2007 is
a. $1,440,000.
b. $1,600,000.
c. $2,160,000.
d. $2,400,000.
9. If Miley uses the completed-contract method, the gross profit to be recognized in 2008 is
a. $1,360,000.
b. $2,800,000.
c. $1,400,000.
d. $5,600,000.
10. Parker Construction Co. uses the percentage-of-completion method. In 2007, Parker began work on a contract for $5,500,000; it was completed in 2008.
The following cost data pertain to this contract:
Year Ended December 31
2007 2008
Cost incurred during the year $1,950,000 $1,400,000
Estimated costs to complete at the end of year 1,300,000 —
The amount of gross profit to be recognized on the income statement for the year ended December 31, 2008 is
a. $800,000.
b. $860,000.
c. $900,000.
d. $2,150,000.
11. If the completed-contract method of accounting was used, the amount of gross profit to be recognized for years 2007 and 2008 would be
2007 2008
a. $2,250,000. $0.
b. $2,150,000. $(100,000).
c. $0. $2,150,000.
d. $0. $2,250,000.
12. Willingham Construction Company uses the percentage-of-completion method. During 2007, the company entered into a fixed-price contract to construct
a building for Richman Company for $30,000,000. The following details pertain to the contract:
At December 31, 2007 At December 31, 2008
Percentage of completion 25% 60%
Estimated total cost of contract $22,500,000 $25,000,000
Gross profit recognized to date 1,875,000 3,000,000
The amount of construction costs incurred during 2008 was
a. $15,000,000.
b. $9,375,000.
c. $5,625,000.
d. $2,500,000.
Use the following information for questions 73 and 74.
Carter Construction Company had a contract starting April 2008, to construct a $15,000,000 building that is expected to be completed in September 2009, at an
estimated cost of $13,750,000. At the end of 2008, the costs to date were $6,325,000 and the estimated total costs to complete had not changed. The progress
billings during 2008 were $3,000,000 and the cash collected during 2008 was $2,000,000. Carter uses the percentage-of-completion method.

13. For the year ended December 31, 2008, Carter would recognize gross profit on the building of
a. $0.
b. $527,083.
c. $575,000.
d. $675,000.
14. At December 31, 2008, Carter would report Construction in Process in the amount of
a. $6,900,000.
b. $6,325,000.
c. $5,900,000.
d. $575,000.
15. Kirby Builders, Inc. is using the completed-contract method for a $5,600,000 contract that will take two years to complete. Data at December 31, 2007, the
end of the first year, are as follows:
Costs incurred to date $2,560,000
Estimated costs to complete 3,280,000
Billings to date 2,400,000
Collections to date 2,000,000
The gross profit or loss that should be recognized for 2007 is
a. $0.
b. a $240,000 loss.
c. a $120,000 loss.
d. a $105,600 loss.
Use the following information for questions 76 through 78.
Melton Construction Co. began operations in 2007. Construction activity for 2007 is shown below. Melton uses the completed-contract method.
Billings Collections Estimated
Contract Through Through Costs to Costs to
Contract Price 12/31/07 12/31/07 12/31/07 Complete
1 $3,200,000 $3,150,000 $2,600,000 $2,150,000 —
2 3,600,000 1,500,000 1,000,000 820,000 $1,880,000
3 3,300,000 1,900,000 1,800,000 2,250,000 1,200,000

16. Which of the following should be shown on the income statement for 2007 related to Contract 1?
a. Gross profit, $450,000
b. Gross profit, $1,000,000
c. Gross profit, $1,050,000
d. Gross profit, $600,000
17. Which of the following should be shown on the balance sheet at December 31, 2007 related to Contract 2?
a. Inventory, $680,000
b. Inventory, $820,000
c. Current liability, $680,000
d. Current liability, $1,500,000
18. Which of the following should be shown on the balance sheet at December 31, 2007 related to Contract 3?
a. Inventory, $200,000
b. Inventory, $350,000
c. Inventory, $2,100,000
d. Inventory, $2,250,000
19 Flynn Construction Co. has consistently used the percentage-of-completion method of recognizing revenue. During 2007, Flynn entered into a fixed-price contract
to construct an office building for $12,000,000. Information relating to the contract is as follows:
At December 31
2007 2008
Percentage of completion 15% 45%
Estimated total cost at completion $9,000,000 $9,600,000
Gross profit recognized (cumulative) 600,000 1,440,000
Contract costs incurred during 2008 were
a. $2,880,000.
b. $2,970,000.
c. $3,150,000.
d. $4,320,000.
20. Noland Constructors, Inc. has consistently used the percentage-of-completion method of recognizing income. In 2007, Noland started work on a
$35,000,000 construction contract that was completed in 2008. The following information was taken from Noland's 2007 accounting records:
Progress billings $11,000,000
Costs incurred 10,500,000
Collections 7,000,000
Estimated costs to complete 21,000,000
What amount of gross profit should Noland have recognized in 2007 on this contract?
a. $3,500,000
b. $2,333,334
c. $1,750,000
d. $1,166,667
computational:
1. c $600,000
—————————— ×($1,500,000 – $1,000,000) = $300,000
$600,000 + $400,000

($1,500,000 – $1,050,000) – $300,000 = $150,000.

2. c $7,200,000
——————————— ×($15,000,000 – $12,000,000) = $1,800,000.
$7,200,000 + $4,800,000

3. b $240,000 – $100,000 = $140,000.

4. d $300,000 – $60,000 = $240,000

$240,000
————————— ×($2,400,000 – Total estimated cost) = $60,000
Total estimated cost

Total estimated cost = $1,920,000


$2,400,000 – $1,920,000 = $480,000.

5. c $1,170,000
—————- ×($3,300,000 – $1,950,000) = $810,000
$1,950,000

($3,300,000 – $2,010,000) – $810,000 = $480,000.

6. b $1,200,000
————— ×($7,200,000 – $4,800,000) = $600,000.
$4,800,000

7. c $7,200,000 – $4,875,000 = $2,325,000.

8. a $3,600,000
————— ×($8,400,000 – $6,000,000) = $1,440,000.
$6,000,000
9. b $8,400,000 – $5,600,000 = $2,800,000.

10. a [$1,950,000 ÷ ($1,950,000 + $1,300,000)] × $2,250,000 = $1,350,000


($5,500,000 – $3,350,000) – $1,350,00 = $800,000.

11. c $5,500,000 – $3,350,000 = $2,150,000.

12. b ($25,000,000 × .60) – ($22,500,000 × .25) = $9,375,000.

13. c ($6,325,000 ÷ $13,750,000) × $1,250,000 = $575,000.

14. a ($6,325,000 ÷ $13,750,000) × $1,250,000 = $575,000.


$6,325,000 + $575,000 = $6,900.000.

15. b $5,600,000 – ($2,560,000 + $3,280,000) = –$240,000.

16. c $3,200,000 – $2,150,000 = $1,050,000.

17. c $1,500,000 – $820,000 = $680,000.

18. a ($2,250,000 – $150,000) – $1,900,000 = $200,000.

19. b ($9,600,000 ×45%) – ($9,000,000 ×15%) = $2,970,000.

$10,500,000
20. d —————— ×($35,000,000 – $31,500,000) = $1,166,667.
$31,500,000

Ex. 18-110—Percentage-of-completion method.


Garnet Construction Co. contracted to build a bridge for $5,000,000. Construction began in 2007 and was completed in 2008. Data relating to the construction are:

2007 2008
Costs incurred $1,650,000 $1,375,000
Estimated costs to complete 1,350,000 —

Garnet uses the percentage-of-completion method.

Instructions
(a) How much revenue should be reported for 2007? Show your computation.
(b) Make the entry to record progress billings of $1,650,000 during 2007.
(c) Make the entry to record the revenue and gross profit for 2007.
(d) How much gross profit should be reported for 2008? Show your computation.

Solution 18-110
(a) $1,650,000
————— × $5,000,000 = $2,750,000
$3,000,000

(b) Accounts Receivable ......................................................................................................................... 1,650,000


Billings on Construction in Process ................................................................................. 1,650,000

Solution 18-110 (cont.)


(c) Construction Expenses...................................................................................................................... 1,650,000
Construction in Process .................................................................................................................... 1,100,000
Revenue from Long-Term Contracts ............................................................................... 2,750,000

(d) Revenue $5,000,000


Costs 3,025,000
Total gross profit 1,975,000
Recognized in 2007 (1,100,000)
Recognized in 2008 $ 875,000
Or
Total revenue $5,000,000
Recognized in 2007 (2,750,000)
Recognized in 2008 2,250,000
Costs in 2008 (1,375,000)
Gross profit in 2008 $ 875,000

Ex. 18-111—Percentage-of-completion method.


Stiner Builders contracted to build a high-rise for $14,000,000. Construction began in 2007 and is expected to be completed in 2010. Data for 2007 and 2008 are:
2007 2008
Costs incurred to date $1,800,000 $5,200,000
Estimated costs to complete 7,200,000 4,800,000

Stiner uses the percentage-of-completion method.


Instructions
(a) How much gross profit should be reported for 2007? Show your computation.
(b) How much gross profit should be reported for 2008?
(c) Make the journal entry to record the revenue and gross profit for 2008.

Solution 18-111
(a) $1,800,000
————— × $5,000,000 = $1,000,000
$9,000,000

(b) $5,200,000
—————— × $4,000,000 = $2,080,000
$10,000,000
Less 2007 gross profit 1,000,000
Gross profit in 2008 $1,080,000

(c) Construction in Process .................................................................................................................... 1,080,000


Construction Expenses...................................................................................................................... 3,400,000
Revenue from Long-Term Contracts ............................................................................... 4,480,000

Ex. 18-112—Percentage-of-completion and completed-contract methods.


On February 1, 2007, Nance Contractors agreed to construct a building at a contract price of $6,000,000. Nance estimated total construction costs would be
$4,000,000 and the project would be finished in 2009. Information relating to the costs and billings for this contract is as follows:

2007 2008 2009


Total costs incurred to date $1,500,000 $2,640,000 $4,600,000
Estimated costs to complete 2,500,000 1,760,000 -0-
Customer billings to date 2,200,000 4,000,000 5,600,000
Collections to date 2,000,000 3,500,000 5,500,000
Instructions
Fill in the correct amounts on the following schedule. For percentage-of-completion accounting and for completed-contract accounting, show the gross profit that
should be recorded for 2007, 2008, and 2009.
Percentage-of-Completion Completed-Contract
Gross Profit Gross Profit
2007 ________________ 2007 ________________

2008 ________________ 2008 ________________

2009 ________________ 2009 ________________

Solution 18-112
Percentage-of-Completion Completed-Contract
Gross Profit Gross Profit
2007 $750,000a 2007 —
2008 $210,000b 2008 —
2009 $440,000c 2009 $1,400,000d
a
$1,500,000
————— × $2,000,000 = $750,000
$4,000,000
b
$2,640,000
————— × $1,600,000 = $960,000
$4,400,000

Less 2007 gross profit (750,000)


2008 gross profit $210,000
c
Total revenue $6,000,000
Total costs 4,600,000
Total gross profit 1,400,000
Recognized to date (960,000)
2009 gross profit $ 440,000
d
Total revenue $6,000,000
Total costs 4,600,000
Total gross profit $1,400,000

Pr. 18-117—Long-term construction project accounting.


Benson Construction specializes in the construction of commercial and industrial buildings. The contractor is experienced in bidding long-term construction projects
of this type, with the typical project lasting fifteen to twenty-four months. The contractor uses the percentage-of-completion method of revenue recognition since,
given the characteristics of the contractor's business and contracts, it is the most appropriate method. Progress toward completion is measured on a cost to cost
basis. Benson began work on a lump-sum contract at the beginning of 2008. As bid, the statistics were as follows:
Lump-sum price (contract price) $4,000,000
Estimated costs
Labor $ 850,000
Materials and subcontractor 1,750,000
Indirect costs 400,000 3,000,000
$1,000,000
Pr. 18-117 (cont.)
At the end of the first year, the following was the status of the contract:
Billings to date $2,230,000
Costs incurred to date
Labor $ 464,000
Materials and subcontractor 1,098,000
Indirect costs 193,000 1,755,000
Latest forecast total cost 3,000,000

It should be noted that included in the above costs incurred to date were standard electrical and mechanical materials stored on the job site, but not yet installed,
costing $105,000. These costs should not be considered in the costs incurred to date.

Instructions
(a) Compute the percentage of completion on the contract at the end of 2008.
(b) Indicate the amount of gross profit that would be reported on this contract at the end of 2008.
(c) Make the journal entry to record the income (loss) for 2008 on Benson's books.
(d) Indicate the account(s) and the amount(s) that would be shown on the balance sheet of Benson Construction at the end of 2008 related to its construction
accounts. Also indicate where these items would be classified on the balance sheet. Billings collected during the year amounted to $1,980,000.
(e) Assume the latest forecast on total costs at the end of 2008 was $4,050,000. How much income (loss) would Benson report for the year 2008?

Solution 18-117
(a) Costs to date $1,755,000
Less materials on job site (105,000)
$1,650,000

Costs Incurred to Date


—————————— = Percentage of Completion
Total Estimated Costs

$1,650,000
————— = 55%
$3,000,000

(b) 55% × $4,000,000 = $2,200,000


Costs incurred 1,650,000
Gross profit $ 550,000
(c) Construction Expense ....................................................................................................................... 1,650,000
Construction in Process .................................................................................................................... 550,000
Revenue from Long-Term Project ................................................................................... 2,200,000
(d) Current Assets
Accounts receivable $250,000 ($2,230,000 – $1,980,000)

Solution 18-117 (cont.)


Current Liability
Billings in excess of contract costs and
recognized profit $30,000 ($2,230,000 – $2,200,000)
(e) Total loss reported in 2008
Contract price $4,000,000
Estimated cost to complete 4,050,000
Amount of loss to be reported $ (50,000)
Pr. 18-118—Accounting for long-term construction contracts.
The board of directors of Dodd Construction Company is meeting to choose between the completed-contract method and the percentage-of-completion method of
accounting for long-term contracts in the company's financial statements. You have been engaged to assist Dodd's controller in the preparation of a presentation to
be given at the board meeting. The controller provides you with the following information:
1. Dodd commenced doing business on January 1, 2008.
2. Construction activities for the year ended December 31, 2008, were as follows:

Total Contract Billings Through Cash Collections


Project Price 12/31/08 Through 12/31/08
A $ 515,000 $ 340,000 $ 310,000
B 690,000 210,000 210,000
C 475,000 475,000 390,000
D 200,000 100,000 65,000
E 480,000 400,000 400,000
$2,360,000 $1,525,000 $1,375,000

Contract Costs Estimated


Incurred Through Additional Costs to
Project 12/31/08 Complete Contracts
A $ 424,000 $101,000
B 195,000 455,000
C 350,000 -0-
D 123,000 97,000
E 320,000 80,000
$1,412,000 $733,000

3. Each contract is with a different customer.


4. Any work remaining to be done on the contracts is expected to be completed in 2009.

Instructions
(a) Prepare a schedule by project, computing the amount of income (or loss) before selling, general, and administrative expenses for the year ended December
31, 2008, which would be reported under:
(1) The completed-contract method.
(2) The percentage-of-completion method (based on estimated costs).

Pr. 18-118 (cont.)


(b) Prepare the general journal entry(ies) to record revenue and gross profit on project B (second project) for 2008, assuming that the percentage-of-completion
method is used.

(c) Indicate the balances that would appear in the balance sheet at December 31, 2008 for the following accounts for Project D (fourth project), assuming that
the percentage-of-completion method is used.
Accounts Receivable
Billings on Construction in Process
Construction in Process
(d) How would the balances in the accounts discussed in part (c) change (if at all) for Project D (fourth project), if the completed-contract method is used?

Solution 18-118
(a) (1) and (2)
Projects A B C D E
Contract price $515,000 $690,000 $475,000 $200,000 $480,000
Contract costs incurred 424,000 195,000 350,000 123,000 320,000
Additional costs
to complete 101,000 455,000 -0- 97,000 80,000
Total cost 525,000 650,000 350,000 220,000 400,000
Total gross profit
or (loss) $ (10,000) $ 40,000 $125,000 $ (20,000) $ 80,000

The amount reported as income (loss) under the completed-contract method for 2008 is:

Project A $(10,000)
B -0-
C 125,000
D (20,000)
E -0-
$ 95,000

The amount reported as income (loss) under the percentage-of-completion method for 2008 is:

Project A $(10,000)
B 12,000 $40,000 × ($195,000 ÷ $650,000)
C 125,000
D (20,000)
E 64,000 $80,000 × ($320,000 ÷ $400,000)
$171,000

(b) Construction in Process .................................................................................................................... 12,000


Construction Expenses...................................................................................................................... 195,000
Revenue from Long-term Contracts ................................................................................ 207,000
Solution 18-118 (cont.)
(c) Billings $100,000
Cash collections 65,000
Accounts receivable $ 35,000
Billings on Construction in Process 100,000

Costs incurred $123,000


Loss reported (20,000)
Construction in process $103,000

(d) The account balances would be the same.


Pr. 18-119—Long-term contract accounting (completed-contract).
Ponce Construction, Inc. experienced the following construction activity in 2008, the first year of operations.
Cash Cost Estimated
Total BillingsCollections Incurred Additional
Contract through through through Costs to
Contract Price 12/31/08 12/31/08 12/31/08 Complete
X $260,000 $165,000 $155,000 $182,000 $ 63,000
Y 330,000 115,000 115,000 100,000 247,000
Z 233,000 233,000 198,000 158,000 -0-
$823,000 $513,000 $468,000 $440,000 $310,000

Each of the above contracts is with a different customer, and any work remaining at December 31, 2008 is expected to be completed in 2009.
Instructions
Prepare a partial income statement and a partial balance sheet to indicate how the above contract information would be reported. Ponce uses the completed-
contract method.
Solution 18-119
Ponce Construction, Inc.
Income Statement
For the Year 2008
Revenue from long-term contracts (contract Z) $233,000
Cost of construction (contract Z) 158,000
Gross profit $ 75,000
Provision for loss (contract Y)* 17,000
*Contract costs through 12/31/08 $100,000
Estimated costs to complete 247,000
Total estimated costs 347,000
Total contract price 330,000
Loss recognized in 2008 $ 17,000
Solution 18-119 (cont.)
Ponce Construction, Inc.
Balance Sheet
As of 12/31/08
Current assets:
Accounts receivable ($513,000 – $468,000) $ 45,000
Inventories
Construction in process (contract X) $182,000
Less: Billings 165,000
Unbilled contract costs 17,000
Current liabilities:
Billings ($115,000) in excess of contract costs ($100,000) 15,000
Estimated loss from long-term contracts 17,000

ACCOUNTING 7
Instructions: Choose the most correct answer for each of the following questions. Write the letter of your choice in CAPITAL in the FIRST PAGE OF
THE GREEN BOOK PROVIDED.
Questions 1 & 2 are based on the following information.
On February 14, 2012, Therese Company established a sales agency in Tagbilaran. Upon establishment of the sales agency, the home office sent
samples costing P8,000 and a working fund of P3,000 to be maintained on the imprest basis. During the six months period, the sales agency reported to
the home office sales orders. These were billed at P70,000 of which of P40,000 was collected) the sales agency paid expenses of P5,800 but was
reimbursed by the home office.
On August 15, 2012, the sales agency samples were valued at P2,000. It was estimated that the gross profit on goods shipped to fill sales order
averaged 40% of cost.
1. The cost of sales of the sales agency for the six months period is
a. P42,000 c. P48,000
b. P44,000 d. P50,000
2. The net income of the sales agency for the six months period is
a. P16,200 c. P10,200
b. P14,200 d. P8,200
3. A branch’s ending inventory of merchandise shipped by the home office and purchased from outside vendors amounts to P 50,000. The post-
closing trial balance in the Unrealized Gross Profit in Branch Inventory account is P 6,000 due to the home office practice of shipping merchandise
at 20% above cost. The merchandise purchased from outside vendors contained in the ending inventory of the branch amounts to:
a. P 38,000 c. P 30,000
b. P 18,000 d. P 14,000

Questions 4 and 5 are based on the following information.


The income statement submitted by Loon Branch to the Home Office for the month of December 31, 2013 follows:

Sales P600,000
Cost of Sales:
Inventory, December 31, 2013 P80,000
Shipments from Home office 350,000
Purchased locally by branch 30,000
Total P460,000
Inventory, December 31, 2013 100,000 360,000
Gross Margin P240,000
Operating Expenses 180,000
Net Income for the month P 60,000

The Branch inventories consisted of:


12/1/2012 12/31/2012
Merchandise purchased from home P70,000 P84,000
Local purchases P10,000 P16,000
Total P80,000 P100,000
After effecting the necessary adjustments, the Home Office ascertained the true net income of the Branch to be P156,000.
4. At what percentage of cost did the home office bill the branch for merchandise shipped to it?
a. 100% c. 120%
b. 140% d. 150%
5. What is the balance of the Allowance for Overvaluation in the branch inventory at December 31, 2013?
a. P10,000 c. P16,000
b. P24,000 d. P34,000
Questions 6 and 7 are based on the following information.
The following information is extracted from the books and records of Elaine Company and its branch. The balances are at December 31, 2012 of the
company’s operations.
Home Office Branch
Sales P260,000
Shipments to branch P 78,000
Shipments from home office 104,000
Purchases 39,000
Expenses 78,000
Inventory, January 1, 2012 26,000
Allowance for overvaluation of branch inventory 31,200

However, no shipments in transit between home office and the branch were made. Both shipments accounts are properly recorded. The ending
inventory includes merchandise acquired from the home office in the amount of P26,000 and P7,800 acquired from outsiders acquired from the
home office in the amount of P26,000 and P7,800 acquired from outsiders for a total of P33,800.
6. What is the realized profit in branch inventory?
a. P21,000 c. P22,533
b. P31,200 d. P24,700
7. What is the amount of branch merchandise beginning inventory that was acquired from the home office?
a. P14,000 c. P15,600
b. P19,000 d. P20,800
Questions 8 and 9 are based on the following information.
Auto Supply Company is engaged in merchandising both at its home office in Cebu City and its branch in Toledo City. Selected accounts taken from the
trial balances of the home office and the branch as of December 31, 2012 follow:
Debits Cebu City Toledo Branch
Inventory, January 1, 2012 P 23,000 P 11,550
Toledo Branch 58,300
Purchases 190,000 105,000
Freight in from home office 5,500
Sundry Expenses 52,000 28,000

Credits
Home Office P 53,300
Sales P155,000 140,000
Sales to branch 110,000
Allowance for Overvaluation of branch inventory at January 1, 2012.
1,000

Additional information:
 The Toledo City branch gets all of its merchandise from the home office. The home office bills the goods at cost plus a 10% mark-up.
At December 31, 2012, a shipment with a billed value of P5,000 was still in transit. Freight on this shipment was P250 and is to be
treated as part of the inventory.
 Inventories on December 31, 2012, excluding the shipment in transit, follow:
Home office, at cost………………………………….……….. P30,000
Branch, at billed price (excluding freight of P520…… 10,000

8. What is the net income of the home office from own operations?
a. P30,470 c. P21,000
b. P20,000 d. P30,470
9. What is the net income of the branch in so far as the home office is concerned?
a. P870 c. P1,500
b. P10,470 d. P12,000
10. Durable Textile Company has a single branch in Bohol. On March 1, 2012, the home office accounting records included an Allowance for
Overvaluation of Inventories – Bohol Branch ledger account with a credit balance of P32,000. During March, merchandise costing P36,000 was
shipped to the Bohol Branch and billed at a price representing a 40% markup on the billed price. On March 31, 2012, the branch prepared an
income statement indicating a net loss of P11,500 for March and ending inventories at billed prices of P25,000. What is the amount of adjustment
for allowance for Overvaluation of Inventories to reflect the true branch net income?
a. P39,257 debit c. P39,333 debit
b. P46,000 credit d. P46,000 debit

Questions 11 and 12 are based on the following information.


Yul Trading Corp. operates a branch in Talisay City. At the close of business on December 31, 2012, Talisay Branch account in the home office books
showed a debit balance of P225,770. The interoffice accounts were in agreement at the beginning of the year. For purposes of reconciling the interoffice
accounts, the following facts were ascertained:
1. An office equipment costing the home office P3,5000 was picked up by the branch as P350.
2. Insurance premium of P675 charged by the home office was taken up twice by the branch.
3. Freight charges on merchandise made by the home office for P1,125 were recorded in the branch book as P1,215.
4. Home office credit memo representing a discount on merchandise for P800 was not recorded by the branch.
5. The branch failed to take up a P700 debt memo from the home office representing the share of the branch in the advertising.
6. The home office inadvertently recorded a remittance for P3,000 from the Cebu branch as a remittance from its Talisay branch.
11. What is the balance of the Home Office account before adjustment as of December 31, 2012?
a. P225,000 c. P228,485
b. 225,770 d. 226,485
12. What is the adjusted balance of the Home Office account as of December 31, 2012?
a. P225,000 c. P225,770
b. 226,485 d. 228,770
Questions 13 and 14 are based on the following information.
PTT Corporation retails merchandise through its home office store and through a branch store in a distant city. Separate ledgers are maintained by the
home office and the branch. The branch store purchases merchandise from the home office (at 120% of home office cost), as well as from outside
supplies. Selected information from the December 31, 2012 trial balances of the home office and branch is as follows:
Home Office Branch
Sales P120,000 P60,000
Shipments to branch 16,000 ----
Purchases 70,000 11,000
Inventory, January 1, 2012 40,000 30,000
Shipments from home office ----- 19,200
Expenses 28,000 12,000
Unrealized profit in branch inventory 7,200 -----
Additional information:
 The entire difference between the shipment accounts is due to the practice of billing the branch at cost plus 20%.
 The December 31, 2012 inventories are P40,000 and P20,000 for the home office and the branch, respectively. (The branch purchased 16%
of its ending inventory from outside supplies.)
 Branch beginning and ending inventories include merchandise acquired from home office is inventoried at 120% of home office cost.
13. What is the realized profit in branch inventory?
a. P4,000 c. P2,800
b. 7,200 d. 4,400
14. What is the net income of the branch as far the home office is concern?
a. P50,200 c. P10,600
b. 15,000 d. 12,200
Questions 15 and 16 are based on the following information.
Kulas Corporation has one branch operation located 500 miles away from the home office. The branch office sales merchandise which is shipped to it
from the home office. The merchandise is transferred at cost but the branch pays reasonable freight charges. The branch office makes sales and incurs
and pays operating expenses. At the end of the current accounting period the true adjusted balance for the home office account on the branch’s books
and the branch office account on the home office’s books is P500,000.
The following items may or may not be reconciling items. The current year is 2012.
1) The home office has shipped merchandise to the branch office which cost P10,000 and which incurs P500 freight charges paid by the
home office but charged to the branch. This merchandise is received by the branch on January 5, 2012.
2) The branch has transmitted P17,000 in cash back to the home office as a partial payment on such purchased merchandise. This cash is
received by the home office on January 6, 2012.
3) The branch office returns some defective merchandise to the home office. The cost of the returned merchandise is P750. The branch
office pays P25 of freight costs which will be charged back to the home office.
4) On December 1, 2012, the home office sends a check for P25,000 to replenish the branch’s charged back to the home office.
5) The branch pays an advertising expense of P800 that should have been paid by the home office since it applied to advertising fees
incurred by the home office of its own benefit.
6) The home office allocated P12,000 of general and administrative expenses to the branch. The branch had not entered the allocation as
of the end of the year.
7) The home office pays insurance premiums on the branch store. The amount paid by the home office is P1,000 but the branch
erroneously records it as P776.00
15. What is the unadjusted balance of the Home Office account?
a. P481,425 c. P500,000
b. 452,276 d. 518,575
16. What is the unadjusted balance of the Branch account?
a. P433,701 c. P518,575
b. 500,000 d. 452,276
Questions 17 through 20 are based on the following information.
Selected information from the trial balances for the home office and the branch of Lalay Company at December 31, 2012 is provided. The branch
acquires merchandise from the home office and outside suppliers.
Home Office Branch
Sales P60,000 P30,000
Shipments to branch 8,000
Allowance for overvaluation of branch inventory 3,600
Shipments from home office 10,000
Purchase (outsiders) 35,000 5,500
Merchandise inventory 12.01.12 20,000 15,000
Expenses 14,000 6,000
Additional information:
Merchandise inventory, December 31, 2012:
Home office P20,000
Branch (P7,500 from home office and P2,500 from outsiders) 10,000

17. The billing rate of home office to branch for merchandise shipments is
a. 120% of cost c. 130% of cost
b. 125% of cost d. 135% of cost
18. How much of the December 1 inventory of the branch represent purchases from outsiders and goods shipped from home office
a. Home office, P5,000 and Outsiders, P10,000 c. Home office, P8,000 and Outsiders, P7,000
b. Home office, P15,000 and Outsiders, P00,000 d. Home office, P12,000 and Outsiders, P3,000
19. The net income reported by the branch is
a. P4,500 c. P3,500
b. P5,600 d. P2,500
20. The combined net income for Home office and branch operations is
a. P22,500 c. P25,100
b. P24,600 d. P21,500
21. Clang-clang Corporation’s home office ships merchandise to its Toledo branch at a billing price of 125% of cost. During 2012 the home office
makes the following entry:
Toledo Branch 75,000
Shipments to Toledo branch 75,000
At year-end 2012, P12,000 of this merchandise remains at Toledo branch inventory.

The entry to adjust the branch income in the books of the home office will include
a. Debit to Allowance for overvaluation of branch inventory, P12,600
b. Credit to Toledo branch account, P2,400
c. Debit to Shipments to Toledo branch, P12,600
d. Credit to Toledo branch inventory, P2,400

22. May Corporation operate two stores: the Head Office store and Rose branch. On December 31, 2012, the Rose Branch account in the home office
books has a balance of P340,000. Both stores use a standard 120% markup on cost. However May’s home office ships merchandise to the
branches at cost. Rose’s ending inventory includes P20,000 of merchandise received from home office

Rose branch remitted P15,000 to home office on December 30, 2012. The Home office will not receive the remittance until January 4, 2013. The
Home office allocated P5,000 general expenses to each of the branches but Rose branch have not yet recorded the expenses at year-end)

Rose branch paid P2,000 for advertising “after Christmas” sales that were to be allocated equally between the two stores. The Home office has not
recorded its share in the expenses.
The unadjusted balance of the Home office account in the books of Rose branch is
a. P324,000 c. P323,000
b. P319,000 d. P318,000
Questions 23 & 24 are based on the following information.
On December 31, 2012, the home office account on the branch books shows a balance of P9,735. The following reconciling data are determined in
accounting for the difference.
a. Merchandise billed at P615 shipped by the home office to the branch on December 28 is still in transit.
b. The branch collected a home office accounts receivable of P2,500, but failed to notify the home office of this collection.
c. The home office recorded the branch net income for November at P1,125. This was in error, as the branch reported net income was P1,215.
d. The home office was charged P640 when the branch returned merchandise to the home office on December 31. The merchandise is in
transit.
23. The unadjusted balance of Branch account is
a. P9,735 c. P10,990
b. P10,350 d. P8,400

24. The adjusting entry to correct branch net income for November is
a. Debit, Branch profit and loss P90 and Credit, Branch account P90
b. Debit, Home office account P90 and Credit, Branch profit and loss P90
c. Debit, Branch account P90 and Credit, Branch profit and loss P90
d. Debit, Branch profit and loss P90 and Credit, Home office account P90

25. BONUS

---NOTHING FOLLOWS---
FINAL PREBOARD CPAR

1. A, B, and C are partners contributing services, P20,000 and P40,000


respectively. The distribution of the P80,000 net assets shall be:
a. A- P20,000 B-P20,000 C-P40,000
b. A- P5,000 B-P25,000 C-P50,000
c. A-P32,000 B-P16,000 C-P32,000
d. A-P26,667 B-P26,667 C-P26,667
b
2. This is not a characteristic of pledge
a. Bilateral b. Nominate c. Consensual d. Real

3. Statement 1- Shares of stock can be the object of pledge or mortgage


Statement 2- Real estate mortgage is an accessory contract
a. true, true b. false, false c. true, false d. false, true
a
4. One or more but less than all the partners have no authority to perform the
following except:
a. Renounce a partnership claim
b. Submit a partnership claim or liability to arbitration
c. Convey partnership property in the ordinary course of partnership
business
d. Do any act which would make it impossible to carry on the ordinary
business of the partnership
c
5. A limited partnership has A, as general partner, B, as limited partner, and C,
as an industrial partner contributing P50,000; P50,000 and services
respectively. The partnership failed and after disposing all its assets to pay
partnership debts there still remains a note payable in the sum of P30,000.
Against whom can the creditor demand payment?
a. A-P30,000 B-P0 C-P0
b. A-P15,000 B-P15,000 C-P0
c. A-P15,000 B-P0 C-P15,000
d. A-P10,000 B-P10,000 C-P10,000
c
6. Statement 1- Any stipulation authorizing the pledgee to appropriate the thing
pledged is void and without effect.
Statement 2- If after the auction sale, the thing pledged is not sold, the
pledgee can appropriate the thing pledged.
a. true, true b. false, false c. true, false d. false, true

7. A pledged his watch to B for P20,000. A failed to pay his obligation. B sold it
at public auction for P18,000. Can B recover the deficiency?
a. Yes, even without stipulation
b. Yes, if there is stipulation
c. No, even if there is stipulation
d. No, only if there is stipulation
c
8. Using the preceding number, if the sale is for P22,000, can A recover the
excess?
a. Yes, even without stipulation
b. Yes, if there is stipulation
c. No, even if there is stipulation
d. No, only if there is stipulation
b
9. A mortgaged his car to B for P200,000. A failed to pay his obligation. B sold it
at public auction for P180,000. Can B recover the deficiency?
a. Yes, even without stipulation
b. Yes, only if there is a stipulation
c. No, even if there is stipulation
d. No, unless there is stipulation
a
10. Using the preceding number, if the sale is for P220,000, can A recover the
excess?
a. Yes, even without stipulation
b. Yes, only if there is stipulation
c. No, even if there is stipulation
d. No, unless there is stipulation
a
11. A corporation is deemed dissolved
a. Ceased operation for at least five years
b. No commencement of business transactions within 2 years from issuance
of certificate of incorporation
c. A member of the board is convicted of an offense
d. Majority of the members of the board are dead
b
12. Which of the following corporate acts requires the approval of the majority of
the outstanding capital stock or of the members of the private corporation
a. To invest corporate funds in another corporation or business.
b. To amend the articles of incorporation.
c. To adopt by-laws
d. To dissolve the corporation
c
13. Except for one, every corporation whose character expires by its own
limitation, or annulled by forfeiture or otherwise, or whose corporate existence
for other purpose is terminated in any other manner shall nevertheless be
continued as a body corporate for three years after the time when it would
have been dissolved for the purpose of the following. The exception is
a. Prosecuting and defending suits by or against it and enabling it to settle
and close its affairs.

2
b. To dispose and convey its property
c. To distribute its assets
d. Continuing the business for which it was established
d
14. Which of the following is not correct?
a. A partnership begins from the moment of the execution of the contract,
unless it is otherwise stipulated.
b. Persons who are prohibited from giving each other any donation or
advantage cannot enter into universal partnership.
c. A particular partnership has for its objects determinate things, their use or
fruits, or a specific undertaking or the exercise of a profession or vocation.
d. Articles of universal partnership entered into without specification of its
nature, only constitute a universal partnership of all present property.
d
15. Objects of pledge, except:
a. Negotiable instruments
b. Shares of stocks
c. Pieces of jewelry
d. Piece of land
d
16. The following are the rules in case a managing partner collects a demandable
debt from a person who also owes the partnership a demand debt, except
a. The sum collected shall be applied to the partnership credit
b. It shall be applied to the two credits in proportion to their amounts
c. The sum shall be fully applied to the partnership credit, if the receipt given
is for the account of the partnership
d. The debtor has the right to have the payment applied in his debt to the
partner if it should be more onerous to him
a
17. The duty to make disclosure, where otherwise there would be a great and
unfair inequality of bargaining position by the use of inside position as regards
relation of directors to stockholders
a. Special circumstances rule
b. Trust fund theory
c. Doctrine of corporate opportunity
d. Incorporate theory
a
18. Chattel mortgage as distinguished from pacto de retro sale
a. A principal and independent contract
b. The title and possession are transferred to the vendee
c. Cannot appropriate to himself the property mortgaged, nor dispose of it
d. Seller may exercise the right to repurchase
c
19. Mutuum as distinguished from commodatum
a. Essentially gratuitous
b. Loan for consumption

3
c. Object is generally non-consummable
d. Bailor retains the ownership of the thing loaned
b
20. Not an essential requisite of pledge
a. The thing pledged to be placed in the possession of the debtor or of a third
person by common agreement
b. It is constituted to secure the fulfillment of a principal obligation
c. The pledgor is the absolute owner of the thing pledged
d. The person constituting the pledge has the free disposal of the property
a
21. A, B, C and D are partners. Their contributions are as follows: A-P50,000; B-
P30,000; C- P20,000; and D-services. The partnership incurred obligations to
third persons, which the firm was unable to pay. After exhausting all the
assets of the partnership, there is still an unpaid balance of P10,000. How
much should each partner pay to the creditors?
a. A-P5,000 B-P3,000 C-P2,000 D-P0
b. A-P2,500 B-P2,500 C-P2,500 D-P2,500
c. A-P4,000 B-P3,000 C-P2,000 D-P1,500
d. A-P4,000 B-P4,000 C-P2,000 D-P0
b
22. A, B and C, co-owners of a particular parcel of land, borrowed P75,000 from
X and Y. The three (3) debtors signed a promissory note on January 10, 2006
promising to pay the creditors on or before April 3, 2006. In addition, the
debtors constituted a mortgage on their property in favor of the creditors. On
maturity date, X demanded payment. How much can X collect from C upon
maturity of the obligations (creditors are solidary)?
a. P50,000 as it represents the actual share of X in the credit.
b. P25,000 as the obligation is presumed to be joint and the amount
represents C’s share in the obligation.
c. P75,000 based on the promissory note
d. P75,000 because the nature of the obligation is solidary
b
23. Using the preceding number, if C paid X his share in the obligations, can he,
as co-owner of the property, seek a partial release of the mortgage
constituted thereon?
a. Yes, because C is answerable only for P25,000
b. Yes, because C’s obligation is already extinguished
c. No, because mortgages are considered indivisible, payment in part shall
not extinguish the obligation
d. No, because the obligation is solidary, payment in part shall not extinguish
the obligation secured by mortgage
c
24. The following are considered elements of the contracts of pledge and
mortgage. Which is the exception?
a. The pledgor or mortgagor must have the free disposal of the thing pledged
or mortgaged

4
b. The pledgor or mortgagor must be the absolute owner of the thing pledged
or mortgaged
c. Both are accessory contracts
d. The thing maybe appropriated if the debtor cannot pay

d
25. P appointed A as his agent orally to sell his parcel of land for P20,000. ten
days later, A sold the same property for P300,000 through a public instrument
executed between him and B. What is the effect and status of sale between A
and B?
a. The sale is valid because A was authorized and it was executed in a
public instrument.
b. The sale is unenforceable because the agent went beyond the scope of
his authority for selling the land for a price higher than the price agreed
upon.
c. The sale is void because there was no valid appointment of A as agent of
P.
d. The sale can be ratified although the appointment of A was done orally
because the sale between A and B was in a public instrument.
c
26. Where a director by virtue of his office, acquired for himself opportunity which
belongs to the corporation, thereby obtaining profits to the prejudice of such
corporation, must account to the latter for all such profits by refunding the
same, unless his act is ratified by
a. A vote at a regular or special meeting by stockholders owning or
representing a majority of the outstanding capital stock.
b. A vote of the stockholders owning or representing at lease one-third of the
outstanding capital stock
c. A unanimous vote of all the stockholders of the corporation
d. A vote of the stockholders owning or representing at least two-thirds of
the outstanding capital stock
d
27. When a borrower uses the thing for a purpose different from the intended,
delays its return to the owner, receives the thing under appraisal, lends to
third person or saves his property instead of the thing in cases of emergency,
shall be liable even in cases of fortuitous events, because
a. The nature of the obligation requires an assumption of risk.
b. The parties have expressly stipulated on such liability.
c. The debtor in delay.
d. The law expressly so provides
d
28. A distribution by a corporation of shares of stock held by it in another
corporation is
a. Case of declaration of stock dividend
b. Situation equivalent to a sale of assets
c. Situation equivalent to a merger or consolidation

5
d. Property dividend or an actual distribution of corporate assets
d
29. A delivered to B 10,000 pieces of Davao fabricated shell craft jewelry for the
purpose of selling them at P1.00 each. Out of the proceeds of the expected
sale, B is to receive a 10% commission. After 3 days, however, B sold all
items at P1.50 each to C, but on 30-day credit. A compelled B to pay. Which
is correct?
a. B must pay P10,000 immediately
b. B cannot be compelled to pay at all
c. B must pay P15,000 immediately
d. B must pay P5,000 immediately and P10,000 after 30 days
a
30. In real estate mortgage, the mortgagor can sell the mortgaged property
a. With the consent of the mortgagee in writing
b. Even without the consent of the mortgagee
c. Only with the consent of the mortgagee in writing or orally
d. Only after paying his obligation to the mortgagee
b
31. P wrote a letter authorizing A to sell his parcel of land situated in Boracay. A
sold the land in writing to X but the agent (A) did not give the money to P. The
sale of the land by A to X is
a. Void, because the authority of A is not in public instrument.
b. Unenforceable, because the authority of A is not in public instrument.
c. Inexistent, because the authority of A is in private writing.
d. Valid and enforceable
d
32. The negotiable character of an instrument is affected by a provision
a. Authorizing the sale of collateral securities in case the instrument is not
paid at maturity.
b. Authorizing a confession of judgment if the instrument is not paid at
maturity.
c. Giving the marker an election to require something to be done in lieu of
payment of money.
d. Waiving the benefit of any law intended for the protection of the obligor.
c
33. This is a negotiable instrument
a. I promise to pay to bearer P10,000 Sgd. A
b. Pay to B or order P10,000 Sgd. A
c. I promise to pay B or order P10,000 in two installments. Sgd. A
d. Pay to B or order P10,000, 30 days after the death of Y. Sgd. A
a
34. This is not a negotiable instrument
a. I promise to pay to the order of bearer P10,000. Sgd. A
b. I promise to pay B or his agent P10,000. Sgd. A
c. I promise to pay B or order US$1,000. Sgd. A
d. I promise to pay to the order of B P10,000. Sgd. A

6
b
35. Rules on construction where instrument is ambiguous, except
a. If not dated, the date is considered to be the date of issuance.
b. Where the signature is placed and is not clear in what capacity the person
intended to sign, he is deemed an accommodation party
c. Between printed and written provision, the written provisions prevail
d. Instruments containing the words “I promise to pay” signed by two or more
persons, such persons are deemed jointly and severally liable
b
36. Which of the following is not negotiable?
a. Pay to B or order P10,000. Sgd. A to C
Accepted: For P5,000 only Sgd. C
b. Pay to Jose P10,000. Sgd. Juan to Pedro
Accepted: For P10,000 Sgd. Pedro
c. Pay to Maria or bearer P10,000. Sgd. Jose to Juan and Pedro
Accepted: If Maria passes the CPA Board Examination. Sgd. Juan
d. Pay to the order of B P10,000 at 123 CM Recto Avenue, Manila. Sgd. A to
C, J.P Rizal St. Makati
Accepted: Payable at 123, JP Rizal St., Makati City. Sgd. C
b
37. Where an indorsement is conditional, may the marker make payment
although the condition has not been fulfilled?
a. Yes, he may disregard the condition without incurring any liability.
b. Yes, he may disregard the condition but he becomes liable if the endorsee
fails to fulfill the condition
c. No, the qualified indorsement becomes part of the contract
d. No, the person who received payment will hold the proceeds subject to the
right of the conditional indorser
a
38. M issues a promissory note payable to P or bearer. Which of the following is
not correct?
a. If indorsed by P to A and A also indorsed it to B, B may negotiates the
note to C by mere delivery
b. If indorsed by P to A and A also indorses it to B, P is liable to A and B
c. If indorsed by P to A and A also indorses it to B and B negotiates the note
to C by delivery. A is liable to B and C
d. If P negotiates the note to A by delivery and A indorses it to B, B may
negotiate the note to C by delivery.
c
39. What indorsements are not necessary to the holder’s title?
Answer 1- Any indorsements thereon, where the instrument is originally
payable to bearer
Answer 2- All indorsements subsequent to a qualified indorsement, where the
instrument is originally payable to order
a. true, true b. false, false c. true, false d. false, true
c

7
40. Which of the following is not correct?
a. Where the instrument is negotiated back to a prior party, all intervening
indorsements are not necessary to the holder’s title
b. Where the instrument is originally payable to order, the holder may not
strike out the payee’s indorsement
c. The indorser whose indorsement is struck out and all indorsers are
thereby relieved from liability on the instrument
d. The holder may at anytime strike out an indorsement which is not
necessary to his title
c
41. Which of the following is not summary remedy of the Government to enforce
tax collection?
a. Distraint c. Levy
b. Civil Action d. Tax Lien
b
42. Which of the following cases may not be compromise by the Bureau of
Internal Revenue Commissioner?
a. Delinquents accounts
b. Cases under administrative protest
c. Fraud cases
d. Cases covered by pre-assessment notice
c
43. A fifty (50%) percent surcharge shall be impose on:
a. Failure to file any return and pay the tax due thereon.
b. Failure of return with an internal officer other than those designated by
law.
c. Willful neglect to file a tax return as prescribed by law.
d. Failure to pay deficiency tax within the prescribed period required by law.
c
44. The legal interest rate for tax delinquency is?
a. Six (6% percent.
b. Eight (8%) percent.
c. Twelve (12%) percent.
d. Twenty (20%) percent.
d
45. Which of the following is not a requisite of tax regulations (BIR revenue
regulations):
a. It must be reasonable and germane to the purpose of the law.
b. It must be within the authority conferred or under the color of valid title.
c. It must be given retroactive application.
d. It must be published.
c
46. It is said that taxes are what we pay for civilized society. Without taxes, the
government would be paralyzed for lack of the motive power to motivate
active and operate.
a. Lifeblood theory c. Benefit-protection theory

8
b. Necessity theory d. Reciprocal theory

47. The power to tax is exclusively lodged with the legislature, however, there are
exceptions provided by the Constitution, which of the following is not:
a. Authority of the President to fix tariff rates, import and export quotas,
tonnage and wharfage dues, and other duties and imposts
b. Power of local government units to levy taxes, fees and charges.
c. Delegation to administrative agencies for implementation and collection of
taxes.
d. Designation to private entities the enforcement and collection of taxes.
d
48. Which of the following is the least Constitutional limitations?
a. Due process clause c. Non-impairment clause
b. Equal protection d. Progressivism clause

49. The following taxes, fees and charges are deemed to be national revenue
taxes, except:
a. Documentary stamp taxes
b. Community taxes
c. Excise taxes
d. Income taxes
b
50. Which of the following is not an authority of the Bureau of Internal Revenue
Commissioner?
a. Authority to administer oaths and to take testimony
b. Authority to make arrests and seizures
c. Authority to deputized members of the Armed Forces of the Philippines
and members of the Philippine National Police to enforce tax laws
d. Authority to assign or reassign internal revenue officers and employees of
the BIR to other special duties connected with the enforcement or
administration of the tax laws
c
51. Which of the following is correct?
a. A protest should be filed by the taxpayer, otherwise the assessment
becomes final and no longer be questioned in court
b. A protest may be filed by the taxpayer anytime before the BIR collects the
tax
c. The assessment should be made by the BIR collects the tax
d. The assessment shall include only tax proper
a
52. Which of the following statements is correct?
a. A pre-assessment notice shall be required before an assessment may be
made.

9
b. The taxpayer shall be informed of the law and the facts on which the
assessment is made, otherwise the assessment shall be void.
c. The Commissioner may refund a tax even without a claim refund from the
taxpayer where on the face of the return upon which the payment was
made, such a payment clearly appears to have been erroneously made.
d. A suit may be brought even after the lapse of two years from the date of
payment, if any supervening cause arises after payment.
a
53. The authorities of the BIR commissioner include the following, except:
a. Compromise the payment of any internal revenue tax.
b. Abate a tax liability
c. Credit or refund tax that had been erroneously received
d. Inquire into bank deposit
d
54. The following are grounds to cancel a tax liability by the BIR commissioner,
except
a. The tax unjustly or excessively assessed
b. The administration and collection costs involved do not justify the
collection of the amount of tax
c. A reasonable doubt as to the validity of the claim against the taxpayer
d. Illegal collection of tax
d
55. Which of the following not a requisite to toll the collection of taxes to be made
by the Bureau of Internal Revenue, upon proof of the following to the Court of
Tax Appeals:
a. It will jeopardize the interest of the taxpayer;
b. It will jeopardize the interest of the Government;
c. Filing of a bond double the amount of tax assessed;
d. The case is not dilatory.
c
56. Transitional Input Tax Rate is:
a. 1.5% b. 8% c. 10% d. 2%
d
57. One of the following is not an ordinary deduction for purposes of Net Taxable
Estate.
a. Funeral expense b. Medical expense c. Judicial d. Taxes
expense
b
58. The following person’s signature must appear in the corporations income tax
return, except:
a. President c. Treasurer
b. Vice President d. External Auditor
d
59. Deduction for funeral expenses shall be allowed, except:
a. Only if paid out of the estate

10
b. For a non-resident alien limited to amount that was actually incurred in the
Philippines
c. Shall in no case to exceed 5% of the gross estate
d. Shall in no case to exceed P200,000
b
60. Which of the following is deductible from the gross estate?
a. Income tax paid on income received after death
b. Property taxes not accrued prior to death
c. Estate tax paid to a foreign country
d. Donor’s tax accrued prior to death
d
61. Which one is incorrect? Losses are deductible if:
a. Arising from natural calamity or fortuitous event
b. Not compensated by insurance
c. A property included in the Philippine gross estate
d. Incurred during the settlement of the estate
c
62. Deductions from gross estates for transfers for public purposes, except
a. Means legacy in a last will and testament to the government
b. Means device in the last will to the government
c. Include any kind of transfer in favor of the government
d. Do not include legacies to charitable institutions
c
63. Which of the following is not an attribute of characteristic of taxes?
a. A tax is a forced charge
b. It is payable in money
c. It is progressive
d. It is levied for public purposes
c
64. Importance of taxes is:
a. It is the lifeblood of the Government
b. It is personal to the taxpayer
c. It is a revenue measure
d. It is for public purpose
a
65. Which of the following is not a nature of taxing power?
a. It is an attribute of the sovereignty.
b. It is inherent in the state.
c. It is exclusively exercised by legislature.
d. It is plenary, comprehensive and unlimited.
c
66. Taxes proceed upon the theory that the existence of the government is a
necessity; it cannot continue without the means to pay its expenses; and for
those means, it has the right to compel all citizens and property within its
limits to contribute
a. Benefit- Protection Theory c. Lifeblood Theory

11
b. Necessity Theory d. Territoriality Theory
b

67. Which of the following will not interrupt the running of the prescriptive period
for assessment and collection of taxes:
a. When the Commissioner is prohibited from making the assessment or
beginning distrait and levy or a proceeding in court and for thirty (30) days
thereafter.
b. When the taxpayer requests for the reinvestigation which is granted by the
Commissioner.
c. When the taxpayer is out of the Philippines.
d. When the taxpayer cannot be located in the address given by him in the
return.
a
68. Which of the following are not deputized agents for the collection of national
internal revenue taxes?
a. The Commissioner of Customs and his subordinates with respect to the
collection of national internal revenue taxes on imported articles.
b. The head of the appropriate government office and his subordinates with
respect to the collection of energy tax.
c. Banks duly accredited by the Commissioner with respect to receipt of
payments of internal revenue taxes authorized to be made through banks.
d. Any officer or employee of an authorized agent bank assigned to transmit
tax returns.
d
69. Occupying the forefront of tax law enforcement is the:
a. Department of Finance c. Legislative Department
b. Bureau of Internal Revenue d. Executive Department
b
70. The powers and duties of the Bureau of Internal Revenue comprehend the
following, except?
a. Assessment and collection of all national and local revenue taxes.
b. Enforcement of all forfeitures, penalties and fines in connection connected
with the collection of national internal revenue taxes.
c. Execution of judgment in all cases decided in its favor by the Court of
Appeals, and the ordinary courts.
d. Effecting and administering the supervisory and police powers conferred
to it by the Tax Code or other laws.
a
71. The power to interpret the provisions of the Tax Code and other tax laws shall
be under the exclusive and original jurisdiction of the:
a. The Commissioner of the BIR
b. The Secretary of Finance
c. The Court of Tax Appeals
d. The Regular Courts

12
a
72. The power to decide disputed assessments, refunds of internal revenue
taxes, fees or other charges, penalties imposed in relation thereto, or other
matters arising under the Tax Code or other laws administered by the Bureau
of Internal Revenue is vested with:
a. The Commissioner of the BIR
b. The Secretary of Finance
c. The Court of Tax Appeals
d. The Regular Courts
a
73. It is the official action of an administrative officer in determining the amount of
tax due from a taxpayer, or it may be a notice to the effect that the amount
stated therein is due from the taxpayer with a demand fro payment of the tax
or deficiency stated therein.
a. Tax investigation
b. Tax audit
c. Tax assessments
d. Tax mapping
c
74. To escape a liability for a deficiency estate tax, after paying the estate tax,
must secure a written discharge of personal liability from:
a. The heirs
b. The commissioner
c. The probate court
d. The court where the estate is being settled
b
75. When an estate is settled extra-judicially, the estate tax return may be filed
and paid:
a. By any of the heirs, with the right of reimbursement from the other heirs
b. Only by the heir with written authority from the other heirs
c. By each of the heirs, the payment being for his distributive share in the
estate tax
d. None of the above

76. For a finding a fraudulent return by the BIR to prosper:


Statement No. 1: Fraud must be intentional, consisting of deception willfully
and deliberately done or resorted to in order to avoid payment of the tax.
Statement No. 2: Fraud cannot be presumed but must be proved. Fraudulent
intent can not be deducted from mistakes however frequent they may be,
especially if, such mistakes emanate from erroneous classification of items in
accounting methods utilized for determination of tax liabilities.
a. True; False
b. True; True
c. False; True

13
d. False; False

b
77. Net capital loss carryover will not be claimed by, except:
a. Individual taxpayers
b. Estate taxpayers
c. Trust taxpayers
d. Corporate taxpayers
d
78. Decedent who is married with a surviving spouse and one legitimate child and
two illegitimate children. Left the following properties.
Real properties P3,000,000
Family home 1,000,000
Other real property (exclusive) 2,000,000
Family lot (exclusive) 400,000
Funeral expenses 300,000
Taxes and losses 1,300,000
Medical expenses 1,000,000

What is the total net taxable estate?


a. P1,250,000
b. P2,250,000
c. P1,150,000
d. P2,450,000
a
79. The following are the reasons which necessitates the filing of an
administrative claim for refund with the BIR, except:
a. To afford the Commissioner an opportunity to consider the claim.
b. To give the Commissioner a chance to correct the errors of
subordinate officers.
c. To notify the Government that such taxes have been questioned
and the notice should be borne in mind in estimating the revenue available
for expenditures.
d. To give ample time to the Commissioner in preparing the cash to
be disbursed for the taxpayer’s refund.
d
80. 1st Statement: A resident corporation is allowed to deduct depreciation
expense regardless of the property’s location.
2nd Statement: A private educational institution may at its option elect either
to deduct capital expenditures during the taxable year or to deduct allowable
for depreciation thereof.
a. True; False
b. True; True
c. False; True
d. False; False
c

14
end

15
INTEGRATED REVIEW 2: Advanced Financial Accounting and Reporting (AFAR)
#1 | Partnership Accounting: Nature, Formation, Operations, Dissolutions, Liquidation

1. The JPB partnership reported net income of P160,000 for the year ended December 31,
20x4. According to the partnership agreement, partnership profits and losses are to be
distributed as follows:

J P B
Salaries P 50,000 P60,000 P30,000
Bonus on net income 10% 5% 10%
Remainder (if positive) 60% 30% 10%
Remainder (if negative) 30% 40% 30%

How should partnership net income for 20x4 be allocated to J, P, and B?


J P B
A. P96,000 P48,000 P16,000
B. P58,000 P64,000 P38,000
C. P60,000 P60,000 P40,000
D. P66,000 P68,000 P46,000

(RESA, May 2018)


2. DO is admitted into the partnership of RE and MI by investing cash equivalent to ¼ of their
capital. Which of the following is true after the admission of DO?
A. Assets of the partnership will increase
B. Total partners’ equity remain the same
C. RE and MI capital decreased by ¼
D. Assets of the partnership will remain the same

(RESA, May 2018)


For numbers 3-4, refer to the problem below:
The following condensed balance sheet is presented at February 18, 2018 for the
partnership of Dana and Janis, who share profits and losses in ratio of 60:40, respectfully.
Cash P150,000 Accounts payable P120,000
Non-cash assets 300,000 Dana, Capital 195,000
Dana, Loan 20,000 Janis, Capital 155,000

The non-cash assets realized P250,000 in actual liquidation


3. How much would Dana receive if cash is distributed to the partners just before the start of
actual liquidation?
A. P 5,000
B. P 18,000
C. P 30,000
D. P 0
4. How much cash would Janis receive upon final liquidation, assuming no prior cash
distribution had been made to the partners.
A. P 135,000
B. P 145,000
C. P 100,000
D. P 0
(PRTC, May 2018)

5. PP contributed P24,000 and CC contributed P48,000 to form a partnership, and they agreed
to share profits in the ratio of their original capital contributions. During the first year of
operations, they made a profit of P16,290; PP withdrew P5,050 and CC P8,000. At the start
of the following year, they agreed to admit GG into the partnership. He was to receive a
one-fourth interest in the capital and profits upon payment of P30,000 to PP and CC, whose
capital accounts were to be reduced by transfers to GG’s capital account of amounts
sufficient to bring them back to their original capital ratio.

How should the P30,000 paid by GG be divided between PP and CC?


A. PP, P 9,825; CC, P 20,175
B. PP, P 15,000; CC, P 15,000
C. PP, P 10,000; CC, P 20,000
D. PP, P 9,300; CC, P 20,700
(Dayag, 2015)

6. Scott, Joe, and Ed are liquidating their partnership. At the date the liquidation begins Scott,
Joe, and Ed have capital account balances of P162,000, P192,500, and P215,000,
respectively and the partners share profits and losses 40%, 35%, and 25%, respectively. In
addition, the partnership has a P36,000 Notes Payable to Scott and a P20,000 Notes
Receivable from Ed. When the liquidation begins, what is the loss absorption power with
respect to Joe?
A. P192, 500
B. P 67,375
C. P550,000
D. P770,000
(Dayag, 2015)
7. Which of the following is not considered a legitimate expense of a partnership?
A. Interest paid to partners based on the amount of invested capital
B. Depreciation on assets contributed to the partnership by partners
C. Salaries for management hired to run the business
D. Supplies used in the partners’ offices
(Punzalan, 2014)
8. In the AA-BB partnership, AA and BB had a capital ratio of 3:1 and a profit and loss
ratio of 2:1 respectively. The bonus method was used to record CC’s admittance as a new
partner. What ratio would be used to allocate, to AA and BB, the excess of CC’s
contribution over the amount credited to CC’s capital account?
A. AA and BB’s new relative ratio.
B. AA and BB’s new relative profit and loss ratio.
C. AA and BB’s old capital ratio.
D. AA and BB’s old profit and loss ratio.

(Dayag 2013)
9. The following is the priority sequence in which liquidation proceeds will be distributed
for a partnership:
A. Partnership drawings, partnership liabilities, partnership loans, partnership capital
balances
B. Partnership liabilities, partnership loans, partnership capital balances.
C. Partnership liabilities, partnership loans, partnership drawings, partnership capital
balances.
D. Partnership liabilities, partnership capital balances, partnership loans

(Punzalan, 2014)
10. Partnership capital and drawings accounts are similar to the corporate
A. Paid in capital, retained earnings, and dividends accounts.
B. Retained earnings accounts
C. Paid in capital and retained earnings accounts
D. Preferred and common stock accounts.
(Punzalan, 2014)

11. An advantage of the partnership as a form of business organization would be


A. Partners do not pay income taxes on their share in partnership income.
B. A partnership is bound by the act of the partners.
C. A partnership is created by mere agreements of the partners.
D. A partnership may be terminated by the death or withdrawal of a partner.
(Punzalan, 2016)

12. In the liquidation of a partnership it is necessary to (1) distribute cash to the partners; (2)
sell non-cash assets; (3) allocate any gain or loss on realization to the partners; and (4) pay
liabilities. These steps should be performed in the following order
A. 2,3,4,1
B. 2,3,1,4
C. 3,2,1,4
D. 3,2,4,1
(Punzalan, 2016)

13. It is the change in the relation of the partners caused by any partner ceasing to be associated
in the carrying on of the business.
A. Dissolution
B. Liquidation
C. Incorporation
D. Break-up
(Millan, 2017)
14. On January 1, 2016, Atta and Boy agreed to form a partnership contributing their
respective assets and equities subject to adjustment. On that date, the following were
provided:

Atta Boy
Cash 28,000 62,000
Accounts receivable 200,000 600,000
Inventories 120,000 200,000
Land 600,000
Building 500,000
Furniture and Fixtures 50,000 35,000
Intangible assets 2,000 3,000
Accounts Payable 180,000 250,000
Other liabilities 200,000 350,000
Capital 620,000 800,000

The ff adjustments were agreed upon:


A. Accounts receivable of P 20,000 and P 40,000 are uncollectible in A’s and B’s
respective books.
B. Inventories of P 6,000 and P 7,000 are worthless in A’s and B’s respective books
C. Intangible assets are to be written off in both books.
What will be the capital balances of the partners after adjustments?
Atta Boy
A. 592,000 750,00
B. 600,000 700,00
C. 592,000 756,300
D. 600,000 750,000
(Punzalan, 2016)

15. Partner Ae first contributed P50,000 of capital into existing partnership on March 1, 2016.
On June 1, 2016, said partner contributed another P20,000. On September 1, 2016, he
withdrew P15,000 from the partnership. Withdrawal in excess of P10,000 are charged to
partner’s capital accounts. What is the annual weighted average capital balance of Partner
Ae?
A. 32,500
B. 51,667
C. 60,000
D. 48,333
(Punzalan, 2016)

16. Maxwell is trying to decide whether to accept a salary of P 40,000 or salary of P 25,000
plus a bonus of 10% of net income after salaries and bonus as a means of allocating profit
among partners. Salaries traceable to the other partners are estimated to be P 100,000.
What amount of income would be necessary so that Maxwell would consider choices to
be equal
A. 165,000
B. 290,000
C. 265,000
D. 305,000
(Punzalan, 2016)

For numbers 17 and 18 refer to the problem below:


On June 30, 2016, the condensed balance sheet for the partnership of Eddy, Fox, and
Grimm together with their respective profit and loss sharing percentage, were as follows:

Assets, net of liabilities P 320,000

Eddy, Capital (50%) P 160,000


Fox, Capital (30%) 96,000
Grimm, Capital (20%) 64,000
Total Capital P 320,000
17. Eddy decided to retire from the partnership by mutual agreement is to be paid P 180,000
out of partnership funds for his interest. Total goodwill implicit in the agreement is to be
recorded. After Eddy’s retirement, what will be capital balances of the other partners?
Fox Grimm
A. 84,000 56,000
B. 102,000 68,000
C. 108,000 72,000
D. 120,000 80,000
(Punzalan, 2016)

18. Assume instead that Eddy remains in the partnership and that Hamm is admitted as a new
partner with 25% interest in the capital of the new partnership for a cash payment
P140,000. Total goodwill implicit in the transaction is to be recorded. Immediately after
admission of Hamm, Eddy’s capital account balance should be

A. 280,000
B. 210,000
C. 160,000
D. 140,000
(Punzalan, 2016)
For numbers 9 and 10 refer to the problem below:
The ABC Partnership has assets with book value of P240,000 and a market value of
P195,000, outside liabilities of P70,000, loans payable to Partner Able of P20,000, and
capital balances for Partners Able, Baker and Chapman of P70,000, P30,000 and P50,000,
respectively. The partners share profits and losses equally.

19. How would the first P100,000 of available assets be distributed?


A. P70,000 to outside liabilities, P20,000 to able and balances equally among partners
B. P70,000 to outside liabilities, and P30,000 to Able
C. P70,000 to outside liabilities, P25,000 to Able, and P5,000 to Chapman
D. P40,000 to Able, P20,000 to Chapman, and the balance equally among partners
(Punzalan, 2017)

20. If all outside creditors and loans to partners had been paid. How would the balance of the
assets be distributed assuming Chapman had already received assets with a value of
P30,000?
A. Each of the partners would receive P30,000
B. Each of the partners would receive P40,000
C. Able: P70,000; Baker: P30,000; Chapman: P20,000
D. Able: P55,000; Baker: P15,000; Chapman: P5,000
(Punzalan, 2017)

21. If a partner’s capital balance is credited for an amount greater than or less than the fair
value of his net contribution, the excess or deficiency is called a
A. Bonus
B. Goodwill
C. Discount
D. Premium
(Millan, 2016)

22. Before allocation of loss, which of the following items are allocated first?
A. Salaries
B. Bonuses to partners
C. Interest on the capital of an industrial partner
D. All of these
(Millan, 2016)

23. After the admission of a new partner, the total partnership capital increased by the fair
value of the new partner’s net contributions to the partnership. The admission was
accounted for
A. Under the goodwill method
B. Under the bonus method
C. As a purchase of interest
D. As an investment in the partnership
(Millan, 2016)

24. On May 1, 2016, Cobb and Mott formed a partnership and agreed to share profits and losses
in the ratio of 3:7, respectively. Cobb contributed a parcel of land that cost him P10,000.
Mott contributed P40,000 cash. The land was sold for P18,000 on May 1, 2016,
immediately after formation of the partnership. What amount should be recorded in
Cobbs’s capital account on formation of the partnership?
A. 18,000
B. 17,400
C. 15,000
D. 10,000
(Punzalan, 2018)

For numbers 5 to 6:
The Grey and Redd Partnership was formed on January 2, 2016. Under the partnership agreement,
each partner has an equal initial capital balance. Partnership net income or loss is allocated 60%
to Grey and 40% to Redd. To form the partnership, Grey initially contributed assets costing
P30,000 with a fair value of P60,000 on January 2, 2016, and Redd contributed P20,000 cash.
Drawings by the partners during 2016 totaled P3,000 by Grey and P9,000 by Redd. The partnership
net income in 2016 was P25,000.
(Punzalan 2018)

25. Under the goodwill method, what is Redd’s initial capital balance in the partnership?
A. 20,000
B. 25,000
C. 40,000
D. 60,000

26. Under the bonus method, what is the amount of bonus?


A. 20,000 bonus to Grey
B. 20,000 bonus to Redd
C. 40,000 bonus to Grey
D. 40,000 bonus to Redd

27. If a partnership has net income of P44,000 and Partner X is to be allocated bonus of 10%
of income after the bonus, what is the amount of bonus Partner X will receive?
A. 3,000
B. 3,300
C. 4,000
D. 4,400
(Punzalan, 2018)

28. A partnership has the following accounting amounts:


Sales P 700,000
Cost of goods sold 400,000
Operating expenses 100,000
Salary allocations to partners 130,000
Interest paid to banks 20,000
Partners' drawings 80,000
What is the partnership net income (loss)?
A. 200,000
B. 180,000
C. 50,000
D. (30,000)
(Punzalan, 2018)
Solution:

29. Ranken purchases 50% of Lark’s capital interest in the K and L partnership for P22,000. If
the capital balances of Kim and Lark are P40,000 and P30,000, respectively, Ranken’s
capital balance following the purchase is
A. 22,000
B. 35,000
C. 20,000
D. 15,000
(Punzalan, 2018)

30. The following condensed balance sheet is presented for the partnership of Smith and Jones,
who share profits and losses in the ratio of 60:40, respectively:
Other assets P 450,000
Smith, loan 20,000
P 470,000

Accounts payable P120,000


Smith, capital 195,000
Jones, capital 155,000
P 470,000

The partners decided to liquidate the partnership. If the other assets are sold for P385,000,
what amount of the available cash should be distributed to Smith?
A. 136,000
B. 156,000
C. 159,000
D. 195,000
(Punzalan, 2018)

31. Flat and Iron partnership agreement provides for Flat to receive 20% bonus on profits
before bonus. Remaining profits and losses are divided between Flat and Iron in the ratio
2:3, respectiviely. Ehich partner has greater advantage when the partnership has a profit or
when it has a loss
A. Profit: Flat; Loss:Iron
B. Profit:Flat; Loss: Flat
C. Profit: Iron; Loss: Flat
D. Profit: Iron; Loss: Iron
(Punzalan, 2018)
32. During 2016, Young and Zinc maintained average capital balances in their partnership of
160000 and 100000, respectively. The partners receive 10% interest on average capital
balances and residual profit or loss is divided equally. Partnership profit before interest was
4000. By what amount should Zinc’s capital account change for the year?
A. 11000 decrease
B. 2000 increase
C. 1000 decrease
D. 12000 increase
(Punzalan, 2018)

Mitz, Marc and Mart are partners sharing profits in the ratio of 5:3:2, respectively. As of
December 31, 2016, their capital balances were 95,000 for Mitz, 80000 for Marc & 60000 for
Mart. On Jan 1, 2017, the partners admitted Vince as a new partner and according to their
agreement, Vince will contribute 80000 in cash to the partnership and also pay 10000 for 15%
of Marc’s share. Vince will be given a 20% share in profits, while the original partners’ share
will be approximately the same as before. After the admission of Vince, the total capital will
be 330000 and Vince’s Capital will be 70000

33. The total amount of goodwill to the old partners, upon the admission of Vince would be:
A. 7000
B. 15000
C. 22000
D. 37000

34. . The balance of Marc’s Capital, after admission of Vince would be:
A. 72600
B. 74600
C. 79100
D. 81100
(Punzalan, 2018)

35. As of Dec 31, the books of AME Partnership showed capital balances of A - 40,000; M
25,000; E-5,000. The partners’ profit or loss ratio is 3:2:1, respectively. The partners
decided to dissolve and liquidate. They sold all the non-cash assets for 37,000 cash. After
settlement of all liabilities amounting to 12,000, they still have 28,000 cash left for
distribution. The loss on realization for distribution is
A. 40,000
B. 42,000
C. 44,000
D. 45,000
(Punzalan, 2018)
36. In installment liquidation, which of the following statements is correct regarding the
partial settlement of the partners’ claims?

A. The claims of the partners and outside creditors are partially settled in proportion
B. No distribution is made to the partners until after all non cash assets are realized
C. The carrying amount of unsold non cash assets is treated as loss
D. Estimates of future liquidation costs do not affect the distribution to the partners
(Milan, 2016)
37. Under the entity theory, a partnership is

A. Viewed as having its own existence apart from the partners


B. Viewed through the eyes of the partnera
C. A separate legal and tax entity
D. Unable to enter into contracts in its own name
(FT&C)

38. Which of the following statements is true concerning the treatment of salaries in
partnership accounting?

A. The salary of a partner is treated in the same manner as salaries of corporate


employees
B. Partner salaries are equal to the annual partner draw
C. Partner salaries may be used to allocate profits and losses; they are not
considered expenses of the partnership
D. Partners salaries are directly closed to the capital account
(FT&C)

39. Which of the following is true?

A. A stipulation that excludes one or more partners from any share in the profits or
losses is valid
B. The income summary account is credited in the entry to record the distribution of
profits
C. In the absence of any agreement, salary allowances to partners shall be provided
when the operations yield losses
D. Salary and interest allowances are reported in the statement of comprehensive
income as salaries and interest expense

40. Partners C & K share profits and losses equally after each has been credited in all
circumstances with annual salary allowances of 15,000 & 12,000, respectively. Under this
arrangement, C will benefit by 3,000 more than K in which of the following:

1. Only if the partnership has earnings of 27,000 or more for the year
2. Only If the partnership does not incur a loss for the year
3. In all earnings or loss situation
4. Only if the partnership has earnings of at least 3000 for the year
(FT&C 11e)
41. On June 30, 2015, the balance sheet of Western Marketing, a partnership, is summarized
as follows:
Sundry assets…………………………………………………………….P150,000
West, Capital…………………………………………………………….…90,000
Tern, Capital………………………………………………………………. 60,000
Wes and Tern share profit and losses at a 60:40 ratio, respectively. They agreed to take in
Cuba as a new partner, who purchases 1/8 interest of West and Tern for P25,000. What is
the amount of Cuba’s capital to be taken up in the partnership books if the book value
method is used?
A. P12,500
B. P18,750
C. P25,000
D. P31,250
(Dayag, 2015)

42. In the AD partnership, Allen’s capital is P140,000 and Daniel’s is P40,000 and they share
income in a 3:1 ratio, respectively. They decide to admit David to the partnership.

Allen and Daniel agree that some of the inventory is obsolete. The inventory account is
decreased before David is admitted. David invests P40,000 for a one-fifth interest. What is
the amount of inventory written down?
A. P4,000
B. P10,000
C. P15,000
D. P20,000
(Dayag, 2015)

43. In the AD partnership, Allen’s capital is P140,000 and Daniel’s is P40,000 and they share
income in a 3:1 ratio, respectively. They decide to admit David to the partnership. David
directly purchases one-fifth interest by paying Allen P34,000 and Daniel P10,000. The land
account is increased before David is admitted. By what amount is the land account
increased?

A. P40,000
B. P36,000
C. P20,000
D. P10,000
(Dayag, 2015)

44. RR and XX formed a partnership and agreed to divide initial capital equally, even though
RR contributed P25,000 and XX contributed P21,000 in identifiable assets. Under the
bonus method approach to adjust the capital accounts, XX’s unidentifiable assets should
be debited for:

A. P11,500
B. P4,000
C. P2,000
D. 0
(Dayag, 2015)

45. Partner A first contributed P50,000 of capital into an existing partnership on March 1, 2015.
On June 1, 2015, the partner contributed another P20,000. On September 1, 2015, the
partner withdrew P15,000 from the partnership. Withdrawals in excess of P10,000 are
charged to the partner’s capital account. The annual weighted-average capital balance is

A. P62,000
B. P51,667
C. P60,000
D. P48,333
(Dayag, 2015)
46. For financial accounting purposes, assets of an individual partner contributed to a
partnership are recorded by the partner at

A. Historical cost
B. Book value
C. Fair market value
D. Lower of cost or market
(Dayag, 2015)

47. Which of the following interest component calculation bases is the least susceptible to
manipulation when allocating profits and losses to partners?

A. Beginning capital account balance


B. Average of beginning and ending capital balance
C. Weighted average capital account balance
D. Ending capital balance
(Dayag, 2015)

48. In a partnership, interest on capital investment is accounted for as a(n)

A. Return on investment
B. Expense
C. Allocation of net income
D. Reduction of capital
(Dayag, 2015)

49. What is the underlying purpose of the interest on capital balances component of allocating
partnership profits and losses?

A. Compensate partners who contribute economic resources to the partnership


B. Reward labor and expertise contributions
C. Reward for special responsibilities taken
D. None of the above
(Dayag, 2015)

50. What is the underlying purpose of the salary component of allocating partnership profits
and losses?

A. Compensate partners who contribute economic resources to the partnership


B. Reward labor and expertise contributions
C. Reward for special responsibilities taken
D. None of the above
(Dayag, 2015)
51. A, B, and C are partners in an accounting firm. Their capital account balances at year-end
were A P90,000; B P110,000; C P50,000. They share profits and losses on a 4:4:2 ratio,
after the following special terms:
• Partner C is to receive a bonus of 10% of net income after the bonus.
• Interests of 10% shall be paid on that portion of a partner’s capital in excess of
P100,000
• Salaries of P10,000 and P12,000 shall be paid to partners A & C respectively.
Assuming a net income of P44,000 for the year, the total profit share of Partner C was:
A. P7,800
B. P16,800
C. P19,400
D. P19,800
(Dayag, 2015)

52. X and Y are in partnership, sharing profits equally and preparing their accounts to 31
December each year. On 1 July 2015, Z joined in the partnership, and from that date profits
are shared X 40%, Y 40%, and Z 20%.

In the year ended 31 December 2015, profits were:


6 months to 31 June 2015 P200,000
6 months to 31 December 2015 P300,000

It was agreed that X and Y only should bear equally the expense for a bad debt of P40,000
written-off in the six months to 31 December 2015 in arriving at the P300,000 profit.

Which of the following correctly states X’s profit share for the year?
A. P216,000
B. P200,000
C. P220,000
D. P224,000
(Dayag, 2015)

53. Pol and Loc are partners with capitals of P200,000 and P100,000 and sharing profits and
losses 3:1 respectively. They agree to admit Chic as partner. Chic invests P125,000 for a
25% interest in the firm. Parties agree that the total firm capital after Chic’s admission is
to be P425,000.
The capital balances of the partners after Chic’s admission are:
A. Pol, P214,062.50; Loc, P104,687.50; and Chic, P106,250
B. Pol, P200,000.00; Loc, P100,000.00; and Chic, P125,000
C. Pol, P239,062.50; Loc, P 79,687.50; and Chic, P125,000
D. Pol, P250,000.00; Loc, P125,000.00; and Chic, P100,000
(Guerrero, 2013)

54. Michelle, an active partner in the Michelle-Esme partnership receives an annual bonus of
25% of the partnership income after deducting the bonus. For the year ended, December
31, 2013, partnership income before the bonus amounted to P240,000. The bonus of
Michelle for the year 2013 is:
A. P45,000
B. P48,000
C. P80,000
D. P60,000
(Guerrero, 2013)

55. Rita, Sisa, and Tina are partners with the capital balances on June30, 2013 of P60,000,
P60,000 and P40,000, respectively. Profits and losses are shared equally. Tina withdraws
from the partnership. The partners agree that Tina is to take certain furniture at their
secondhand value of P2,400 and cash for the balance of her interest. The furniture is carried
on the books as fully depreciated.

The amount of cash to be paid to Tina and the capital balances of the remaining partners
after the retirement of Tina are:
Cash Rita, capital Sisa, capital
A. P40,000 P60,000 P60,000
B. P37,000 P61,200 P61,200
C. P38,400 P60,800 P60,800
D. P42,800 P58,800 P58,800
(Guerrero, 2013)

For question 55-56 refer to the problem below


The AA, BB, CC Partnership was formed on January 2. 2019. The original cash
investments were as follows:
AA P 48,000
BB P 72,000
CC P108,000
According to the general partnership contract, the partners were to be remunerated as
follows:
a. Salaries of P72,000 for AA, P6,000 for BB, and P6,800 for CC.
b. Interest at 12% on the average capital account balances during the year.
c. Remainder divided 40% to AA, 30% to BB, and 30% for CC.
Income before partners’ salaries for the year ended December 31, 2019, was P46,040. AA
invested an additional P12,000 in the partnership on July 1; CC withdrew P18,000 from
the partnership on October 1, and, as authorized by the partnership contract, AA, BB, and
CC each withdrew P375 monthly against their shares of net income for the year.

56. Determine the share of partner AA in the net income:


A. P18,416
B. P17,616
C. P13,080
D. P5,880
(ReSA, 2018)

57. Determine the capital balance of partner CC on December 31, 2019:


A. P108,770
B. P104,270
C. P100,112
D. P99,312
(ReSA, 2018)

58. If the salaries to partners are to be recognized as operating expenses by the partnership,
what amount is the share of partner BB in the net income?
A. P18,416
B. P14,190
C. P8,190
D. P7,812
(ReSA, 2018)

(For question 59 – 60)


DD and EE was organized and began operations of March 1, 2019. On that date, DD
invested P75,000 and EE invested land and building with current fair value of P40,000 and
P50,000, respectively. EE also invested P30,000 in the partnership on November 1, 2019
because of its shortage of cash. The partnership contract includes the following
remuneration plan:
DD EE
Annual Salary P9,000 P12,000
Annual interest on average capital account balance 10% 10%
Remainder 60% 40%

The annual salary was to be withdrawn by each partner in 12 monthly installments. During
fiscal year ended, February 28, 2020, DD and EE had net sales of P25,000, cost of goods
sold of P140,000 and total operating expenses of P50,000 (excluding partners’ salaries and
interest on average capital account balances). Each partner made monthly cash drawings
in accordance with partnership contract.

59. Determine the share of partner DD in the net income:


A. P29,400
B. P33,000
C. P36,000
D. P23,400
(ReSA, 2018)

60. The capital balance of each partner on March 1, 2020 should be:
A. DD P95,400; EE P138,600
B. DD P66,000; EE P82,000
C. DD P108,000; EE P147,000
D. DD P99,000; EE P135,000
(ReSA, 2018)

61. Which of the following is not considered a legitimate expense of a partnership?


a. Supplies used in the partners’ office
b. Depreciation on assets contributed to the partnership by partners
c. Salaries for management hired to run the business
d. Interest paid to partners based on the amount of invested capital

Millan Textbook (2016)

62. If the partnership agreement does not specify how income is to be allocated, profits and
loss should be allocated
A. Equally.
B. In proportion to the weighted average of capital invested during the period.
C. Equitably so that partners are compensated for the time and effort expended on
behalf of the partnership.
D. In accordance with their capital contributions.
(Millan, 2016)
63. When Mill retired from the partnership of Mill, Yale, and Lear, the final settlement of
Mill’s interest exceeded Mill’s capital balance. Under the bonus method, the excess
A. Was recorded as goodwill.
B. Was recorded as an expense.
C. Reduced the capital balances of Yale and Lear.
D. Had no effect on the capital balances of Yale and Lear.
(Millan, 2016)
64. State the correct order of the claims on the personal assets of a partner,
I. The partner’s separate creditors
II. To the other partner’s by way of contribution
III. The partnership creditors
A. I, III, II
B. I, II, III
C. III, II, I
D. II, I, III
(Millan, 2016)
65. It is the change in the relation of the partners caused by any partner ceasing to be associated
in the carrying on of the business.
A. Dissolution
B. Liquidation
C. Incorporation
D. Break-up
(Millan, 2016)
66. MM, NN, and OO are partners with capital balances on December 31, 2012 of P300,000,
P300,000 and P200,000, respectively. Profits are shared equally. OO wishes to withdraw
and it is agreed that OO is to take certain equipment with second-hand value of P50,000
and a note for the balance of OO’s interest. The equipment are carried on the books at
P65,000. Brand new equipment may cost P80,000. Compute for: (1) OO’s acquisition of
the second-hand equipment will result to reduction in capital; (2) the value of the note that
will OO get from the partnership’s liquidation.
A. (1) P 15,000 each for MM and NN, (2) P150,000
B. (1) P5,000 each for MM, NN and OO, (2) P145,000
C. (1) P5,000 each for MM, NN and OO, (2) P195,000
D. (1) P7,500 each for MM and NN, (2) P145,000
(Dayag, 2013)
67. The partnership agreement of XX, YY and ZZ provides for the year-end allocation of net
income in the following order:
• First, XX is to receive 10% of net income up to P200,000 and 20% over
P200,000.
• Second, YY and ZZ each are to receive 5% of the remaining income over
P300,000.
• The balance of income is to be allocated equally among the three partners.
The partnership’s 2011 net income was P500,000 before any allocations to partners. What
amount should be allocated to XX?
A. P202,000
B. P216,000
C. P206,000
D. P220,000
(Dayag, 2013)
68. RR and XX formed a partnership and agreed to divide initial capital equally, even though
RR contributed P25,000 and XX contributed P21,000 in identifiable assets. Under the
bonus approach to adjust the capital accounts. XX’s unidentifiable assets should be debited
for:
A. P11,500
B. P 4,000
C. P 2,000
D. P 0
(Dayag, 2013)
69. As of December 31, 2012, the books of Ton Partnership showed capital balances of: T,
P40,000; O, P25,000; N, P5,000. The partner’s profit and loss ratio was 3:2:1, respectively.
The partners decided to liquidate and they sold all non-cash assets for P37,000. After
settlement of all liabilities amounting P12,000, they still have cash of P28,000 left for
distribution. Assuming that any capital debit balance is uncollectible, the share of T in the
distribution of the P28,000 cash would be:
A. P17,800
B. P18,000
C. P19,000
D. P17,000
(Dayag, 2013)
70. CC, PP and AA, accountants, agree to form a partnership and to share profits in the ratio
of 5:3:2. They also agreed that AA is to be allowed a salary of P28,000, and that PP is to
be guaranteed P21,000 as his share of the profits. During the first year of operation, income
from fees are P180,000, while expenses total P96,000. What amount of net income should
be credited to each partner’s capital account?
A. CC, P28,000, PP, P16,800, AA, P11,200
B. CC, P25,000, PP, P21,000, AA, P38,000
C. CC, P24,000, PP, P22,000, AA, P38,000
D. CC, P25,000, PP, P21,000, AA, P39,000
(Dayag, 2013)
71. Allen retired from the partnership of Allen, Beck and Chale. Allen’s cash settlement from
the partnership was based on new goodwill determined at the date of retirement plus the
carrying amount of the other net assets. As a consequence of the settlement, the capital
accounts of Beck and Chale were decreased. In accounting for Allen’s withdrawal, the
partnership could have used the:

BONUS METHOD GOODWILL METHOD


A. No Yes
B. No No
C. Yes Yes
D. Yes No
(Milan, 2016)

72. Which of the following has the least priority of payment in case of partnership
liquidation?

A. Priority claims such as artisans. Government, liquidation expenses


B. Secured creditors to the extent of covered by the proceeds from the sale of pledged
assets.
C. Unsecured credit to the extent covered by proceeds from sale of unpledged (or free)
assets.
D. The partners’ capital balances.
(Milan 2016)

73. State the proper order of liquidation


I Outside creditors
II Owners’ interests
III Inside creditors
A. I, III, II
B. I, II, III
C. III, II, I
D. II, I, III

(Milan, 2016)

74. According to the Philippine Civil Code, if only the shares of each partner in the profits
has been agreed upon, the share of each in the losses shall be
A. equally
B. equally, but the industrial partner shall not share in the loss
C. the same as the sharing in profits
D. the same as the sharing in profits. However, the industrial partner shall not
share in the loss.
(Milan, 2016)

75. Which of the following is not considered a legitimate expense of a partnership?


A. Supplies used in the partners’ offices.
B. Depreciation on assets contributed to the partnership by partners.
C. Salaries for management hired to run the business.
D. Interest paid to partners based on the amount of invested capital.
(Milan, 2016)

76. CC, PP and AA, accountants, agree to form a partnership and to share profits in the ratio
of 5:3:2. They also agreed that AA is to be allowed a salary P28,000 and that PP is to be
guaranteed P21,000 as his share of the profits. During the first year of operation, income
from fees are P180, 000, while expenses total P96,000. What amount of net income should
be credited to each partner’s capital account?
A. CC, P28,000 PP, P16,800 AA, P11,200
B. CC, P25,000 PP, P21,000 AA, P38,000
C. CC, P24,000 PP, P22,000 AA, P38,000
D. CC, P25,000 PP, 21, 000 AA, P39, 000
(Dayag, 2015)
77. The same information in Number 32, except the partnership had a loss of P125, 624 after
the interest and salaries to partners, by what amount should BB’s capital account change
increase(decrease)?
A. P (115,443) C. P (41,875)
B. 23,865 D. (18,010)
(Dayag, 2015)

78. PP, QQ and RR, partners to a firm, have capital balances of P11, 200, P13, 000 and P5,
800, respectively, and share profits in the ratio of 4:2:1. Prepare a schedule showing how
available cash will be given to the partners as it becomes available. Who among the
partners shall be paid first with an available cash of P1, 400?
A. QQ B. No One C. RR D. PP

79. The August, Albert and Gerry partnership became insolvent on January 1, 2015, and the
partnership is being liquidate as as practicable. In this respect the following information
for the partners has been marshaled.

Capital Balances Personal Assets Personal Liabilities


August P 70,000 P 80,000 P 40,000
Albert - 60,000 30,000 50,000
Gerry - 30,000 70,000 30,000

Assume that residual profits and losses are shared equally among the three partners.
Based on this information, calculate the maximum amount that August can expect to
receive from the partnership liquidation is:

A. P20, 000 C. P70,000


B. 40,000 D. 110,000
(Dayag, 2015)

80. RR and XX formed a partnership and agreed to divide initial capital equally, even though
RR contributed P25,000 and XX contributed P21,000 in identifiable assets. Under the
bonus approach to adjust the capital accounts. XX’s unidentifiable assets should be debited
for:

A. 11,500 C. 2,000
B. 4,000 D. 0

(Dayag, 2015)

81. Partnership capital and drawing accounts are similar to the corporate

A. Paid-in capital, retained earnings, and dividend accounts.


B. Retained earnings account.
C. Paid-in capital and retained earnings accounts.
D. Preferred and common stock accounts.
(Gleim)

82. The partnership agreement is an express contract among the partners (the owners of the
business). Such an agreement generally does not include

A. A limitation on a partner’s liability to creditors.


B. The rights and duties of the partners.
C. The allocation of income between the partners.
D. The rights and duties of the partners in the event of partnership dissolution.
(Gleim)

83. A partnership records a partner’s investment of assets in the business at

A. The market value of the assets invested.


B. A special value set by the partners.
C. The partner’s book value of the assets invested.
D. Any of the above, depending upon the partnership agreement.
(RPCPA 0598)

84. Assume that C has a P50,000 equity in the partnership of “A, B, and C.” Partner C arranges
to sell his entire interest to D for P80,000 Cash. Partners A and B agree to the admission
of D. At what amount will the equity of the incoming partner, D, be shown in the balance
sheet?

A. at P50,000.
B. at P50,000 and the P30,000 will be divided equally among the original partners.
C. at P80,000
D. at P80,000 and the P30,000 will represent Goodwill which will be apportioned between
E. the existing equities of A and B.
(RPCPA 107)

Albion and Blaze share profits and losses equally. Albion and Blaze receive salary allowances of
$20,000 and $30,000, respectively, and both partners receive 10% interest on their average capital
balances. Average capital balances are calculated at the beginning of each month balance
regardless of when additional capital contributions or permanent withdrawals are made
subsequently within the month. Partners’ drawings are not used in determining the average capital
balances. Total net income for 2006 is $120,000.

Albion Blaze
January 1 capital balances $ 100,000 $ 120,000
Yearly drawings ($1,500 a month) 18,000 18,000
Permanent withdrawals of capital:
June 3 ( 12,000 )
May 2 ( 15,000 )
Additional investments of capital:
July 3 40,000
October 2 50,000

85. What is the weighted-average capital for Albion and Blaze in 2006?

A. $100,000 and $120,000.


B. $105,333 and $126,667.
C. $110,667 and $119,583.
D. $126,667 and $105,333.
(Beams, 2009)
86. If the average capital for Albion and Blaze from the above information is $112,000 and
$119,000, respectively, what will be the total amount of profit allocated after the salary
and interest distributions are completed?

A. $70,000.
B. $73,100.
C. $75,000.
D. $80,000.
(Beams, 2009)

87. If the average capital balances for Albion and Blaze are $100,000 and $120,000,
what will the final profit allocations for Albion and Blaze in 2006?

A. $50,000 and $70,000.


B. $54,000 and $66,000.
C. $70,000 and $50,000.
D. $75,000 and $45,000.
(Beams, 2009)
For number 88 to 89 refer to the problem below:
Bloom and Carnes share profits and losses in a ratio of 2:3, respectively. Bloom and Carnes receive
salary allowances of $10,000 and $20,000, also respectively, and both partners receive 10%
interest based upon the balance in their capital accounts on January 1. Partners’ drawings are not
used in determining the average capital balances. Total net income for 2006 is $60,000. If net
income after deducting the interest and salary allocations is greater than $20,000, Carnes receives
a bonus of 5% of the original amount of net income.

Bloom Carnes
January 1 capital balances $ 200,000 $ 300,000
Yearly drawings ($1,500 a month) 18,000 18,000

88. What are the total amounts for the allocation of interest, salary, and bonus, and, how
much over-allocation is present?

A. $60,000 and $0.


B. $80,000 and $20,000.
C. $83,000 and $0.
D. $83,000 and $23,000.
(Beams, 2009)
89. The XYZ partnership provides a 10% bonus to Partner Y that is based upon
partnership income, after deduction of the bonus. If the partnership's income is
$121,000, how much is Partner Y's bonus allocation?

A. $11,000.
B. $11,450.
C. $11,650.
D. $12,100.
(Beams, 2009)

Lara, Ives, and Jack are in the process of liquidating their partnership. Since it may take several
months to convert the other assets into cash, the partners agree to distribute all available cash
immediately, except for $10,000 that is set aside for contingent expenses. The balance sheet and
residual profit and loss sharing percentages are as follows:

Cash $ 400,000 Accounts payable $ 200,000


Other assets 200,000 Hara, capital (40%) 135,000
Ives, capital (30%) 216,000
Jack, capital (30%) 49,000

Total assets $ 600,000 Total liab./equity $ 600,000

90. How much cash should Ives receive in the first distribution?

A. $146,000.
B. $147,000.
C. $153,000.
D. $156,000
(Beams, 2009)

91. Which of the following transactions shall not affect the capital balance of a partner?
A. Share of a partner in the partnership’s net loss
B. Receipt of bonus by a partner from another partner based on the agreement
C. Advances made by the partnership to a partner
D. Additional investment by a partner to the partnership

CPAR Testbank

92. Which of the following will not result to the dissolution of a partnership?
E. Insolvency of the partnership
F. Admission of a new partner in an existing partnership
G. Assignment of an existing partner’s interest to a third person
H. Retirement of a partner

CPAR Testbank

93. He refers to a partner who contributed not only money and property but also industry
to the newly formed partnership.
A. Industrial partner
B. Nominal partner
C. Capitalist-industrial partner
D. Capitalist partner
CPAR 2017 Pre-Board

94. It refers to a type of partnership wherein all partners are liable to the creditors pro-rata
up to the extent of personal or separate assets after the partnership’s assets are
exhausted.
A. General partnership
B. Partnership by estoppel
C. Limited partnership
D. Particular partnership
CPAR 2017 Pre-Board

95. Which of the following statements concerning the formation of partnership business is
correct?
A. Philippine Financial Reporting Standards (PFRS) allows recognition of goodwill
arising from the formation of partnership.
B. The juridical personality of the partnership arises from the issuance of certification of
registration.
C. The parties may become partners only upon contribution of money or property but not
of industry or service.
D. The capital to be credited to each partner upon formation may not be the amount
actually contributed by each partner.
CPAR 2017 Pre-Board
96. The partners, C and D, share profits 3:2. However, C is to receive a yearly bonus of
20% of the profits, in addition to his profit share. The partnership made a net income
for the year of P960,000 before the bonus. Assuming C’s bonus is computed on profit
after deducting said bonus, how much profit share will D receive?
A. P307,200
B. P320,000
C. P640,000
D. P160,000
CPAR 2017 Pre-Board

97. A, B, and C are partners and share profits and losses as follows: Salaries of P40,000 to
A; P30,000 to B; and none to C. If net income exceeds salaries, then a bonus is allocated
to A. The bonus is 5 percent of net income after deducting salaries and the bonus.
Residual profits or residual losses are allocated 10 percent to A, 20 percent to B, and
70 percent to C. If net income before salaries and bonus is P140,000, how much is the
share of A?
A. P50,150
B. P43,333
C. P46,667
D. P50,000
CPAR 2017 Pre-Board

98. Carson and Lamb establish a partnership to operate and used-furniture business under
the name of C & L Furniture. Carson contributes furniture that cost P60,000 and has a
fair value of P90,000. Lamb contributes P30,000 cash and delivery equipment that cost
P40,000 and has a fair value of P30,000. The partners agree to share profits and losses
60% to Carson and 40% to Lamb. Calculate the peso amount of inequality that will
result if the initial noncash contributions of the partners are recorded at cost rather than
fair market value.
A. P30,000
B. P10,000
C. P20,000
D. P18,000
CPAR 2017 Pre-Board
Items 99 and 100 are based on the following:
Lucy and Annie were partner sharing profits and losses equally. Ochie was admitted as a partner
by contributing cash of P60,000 for one-third interest in the firm. They agreed to set the total
capital at P210,000 after Ochie’s admission. Prior to Ochie’s admission, the old partner’s capital
accounts were Lucy, P48,000, and Annie, P96,000.
99. The capital balance of Annie after Ochie’s admission was
A. P92,667
B. P94,000
C. P91,000
D. P96,000
100. Assuming that Ochie will share one-fourth interest on the partnership assets the
capital balance of Annie after Ochie’s admission is
A. P96,000
B. 99,750
C. P99,000
D. P102,750
CPAR 2017 Pre-Board

101. A partnership agreement calls for allocation of profits and losses by salary
allocations, a bonus allocation, interest on capital, with any remainder to be allocated by
preset ratios. If a partnership has a loss to allocate, generally which of the following
procedures would be applied?

A. Any loss would be allocated equally to all partners.


B. Any salary allocation criteria would not be used.
C. The bonus criteria would not be used.
D. The loss would be allocated using the profit and loss ratios, only.
(Fisher, 2008)

102. Della Reise was admitted to a partnership. She contributed $25,000 cash plus
equipment she purchased for $50,000 and which had accumulated depreciation for tax
purposes of $20,000. The fair value of the equipment was $35,000. She also assumed 1/3
of partnership debt of $15,000. Her beginning capital balance was $48,000. For tax
purposesher partnership interest should be initially valued at

A. $60,000
B. $48,000
C. $55,000
D. $65,000
(Fisher, 2008)
103. Under the bonus method, when a new partner is admitted to the partnership, the
total capital of the new partnership is equal to:

A. the book value of the previous partnership + the fair market value of the
consideration paid to the existing partnership by the incoming partner
B. the book value of the previous partnership + any necessary asset write ups from
book value to market value + the fair market value of the consideration paid to the
existing partnership by the incoming partner
C. the book value of the previous partnership - any asset write downs from book to
market value + the fair market value of the consideration paid to the existing
partnership by the incoming partner
D. the fair market value of the new partnership as implied by the value of the
incoming partner's consideration in exchange for an ownership percentage in the
new partnership
(Fisher, 2008)

104. Assume that a partnership had assets with a book value of $240,000 and a market
value of $195,000, outside liabilities of $70,000, loans payable to partner Able of $20,000,
and capital balances for partners Able, Baker, and Chapman of $70,000, $30,000, and
$50,000. How would the first $100,000 of available assets be distributed assuming profits
and losses are allocated equally?

A. $70,000 to outside liabilities, $20,000 to Able, and the balance equally among the
partners
B. $70,000 to outside liabilities and $30,000 to Able
C. $70,000 to outside liabilities, $25,000 to Able, and $5,000 to Chapman
D. $40,000 to Able, $20,000 to Chapman, and the balance equally among the partners
(Fisher, 2008)

105. Partners Dalton, Edwards, and Finley have capital balances of $40,000, 90,000 and
$30,000, respectively, immediately prior to liquidation. Total remaining assets have a book
value of $160,000, the liabilities having been paid. Among these remaining assets is a
machine with a fair value of $35,000. The partners split profits and losses equally. Edwards
covets the machine and is willing to accept it for $35,000 in lieu of cash. The other partners
have no designs on specific assets, only cash in liquidation. How much cash, in addition to
the machine, would be first distributed to Edwards, before any of the other partners
received anything?

A. $15,000
B. $50,000
C. $166,667
D. $300,000
(Fisher, 2008)

106. Partners Thomas, Adams and Jones have capital balances of $24,000, $45,000, and
$90,000 respectively. They split profits in the ratio of 3:3:4, respectively. Under a
predistribution plan, one of the partners will get the following total amount in liquidation
before any other partners get anything:

A. $22,500
B. $30,000
C. $40,000
D. $75,000
(Fisher, 2008)

107. Which of the following statements are true when comparing corporations and
partnerships?

A. Partnership entities provide for taxes at the same rates used by corporations.
B. In theory, partnerships are more able to attract capital.
C. Like corporations, partnerships have an infinite life.
D. Unlike shareholders, general partners may have liability beyond their capital
balances.

108. Which of the following characteristics of a partnership most likely explains why a
public accounting firm is organized as a partnership from a public policy viewpoint?

A. A partnership is not a taxable entity.


B. A partnership is characterized by unlimited liability.
C. A partnership is characterized by a fiduciary relationship among the partners.
D. Salaries to the partners are not considered a component of net income.
(Fisher, 2008)

109. The partnership agreement is an express contract among the partners (the owners
of the business). Such an agreement generally does not include:

A. A limitation on a partner’s liability to creditors.


B. The rights and duties of the partners.
C. The allocation of income between the partners.
D. The rights and duties of the partners in the event of partnership dissolution.

(Punzalan, 2016)

110. A partnership records a partner’s investment of assets in the business at

A. The market value of the assets invested.


B. A special value set by the partners.
C. The partner’s book value of the assets invested.
D. Any of the above, depending upon the partnership agreement.
(Punzalan, 2016)

111. When property other than cash is invested in a partnership, at what amount should
the noncash property be credited to the contributing partner’s capital account?

A. Fair value at the date of recognition.


B. Contributing partner’s original cost.
C. Assessed valuation for property tax purposes.
D. Contributing partner’s tax basis.

(Punzalan, 2016)

112. X, Y, Z are capitalist partners and D an industrialist partner. The partnership


reported a net loss of P200,000. How much is the share of D?

A. 0
B. 10,000
C. 25,000
D. 100,000
(Punzalan, 2016)

113. Assume that C has a P715,000 equity in the partnership of “A, B, and C.” Partner
C arranges to sell his entire interest to D for P80,000 Cash. Partners A and B agree to the
admission of D.At what amount will the equity of the incoming partner, D, be shown in
the balance sheet?

A. at P715,000.
B. at P50,000 and the P30,000 will be divided equally among the original partners.
C. at P80,000
D. at P80,000 and the P30,000 will represent Goodwill which will be apportioned
between
E. the existing equities of A and B.
(Punzalan, 2016)

114. Partner Morgan is personally insolvent, owing P600,000. Personal assets will only
bring P200,000 when liquidated. At the same time, Morgan has a credit balance in the
partnership of P120,000. The capital amounts of the other partners total a credit balance
of P250,000. Under the doctrine of marshaling of assets, how much the personal creditors
of Morgan can collect?

A. 120,000
B. 200,000
C. 320,000
D. 570,000
(Punzalan, 2016)

For Numbers 115 to 116 refer to the problem below


As of December 31, the books of AME Partnership showed capital balances of A-P40,000; M-
P25,000; And E-P5,000. The partners’ profit and loss ratio were 3:2:1, respectively. The partners
decided to dissolve and liquidate. They sold all the non-cash assets for P37,000 cash. After
settlement of all liabilities amounting to P12,000, they still have P28,000 cash left for distribution.

115. The loss on the realization of the non-cash assets was

A. P40,000
B. P42,000
C. P44,000
D. P45,000

(Punzalan, 2016)

116. Assuming that any partner’s capital debit balance is uncollectible, the share of A in
the P28,000 cash for distribution would be

A. P19,000
B. P18,000
C. P17,800
D. P40,000
(Punzalan, 2016)

117. The following balance sheet is presented for the partnership A, B and C, who share
profits and losses in the respectively ratio of 5:3:2

Assets Liabilities and Capital


Cash Php 120,000 Liabilities Php 280,000
Other Assets 1,080,000 A, Capital 560,000
B, Capital 320,000
C, Capital 40,000
Total Php 1,200,000 Total Php 1,200,000

Assume the three partners decided to liquidate the partnership. If the other assets are sold
for P800,000, how should the available cash be distributed to each partner?
A B C
A. 280,000 320,000 40,000
B. 324,000 236,000 16,000
C. 410,000 230,000 0
D. 412,000 228,000 0

(Punzalan, 2016)

118. Partners Almond, Barney and Colors have capital balances of P20,000, P50,000,
and P90,000, respectively. They split profits in the ratio of 2:4:4, respectively. Under a
safe cash distribution plan, one of the partners will get the following total amount in
liquidation before any other partners get anything
A. 0
B. 15,000
C. 40,000
D. 180,000

(Punzalan, 2016)
119. Methods exist for the division of partnership profits and losses

A. Equally
B. Arbitrary ratio
C. Capital contribution ratio
D. All of the above
(Dayag, 2015)
120. For a partner to withdraw or retire from the partnership, the total interest of a partner
should be properly determined which includes:

A. Share in the profit or loss of the partnership


B. Adjustments in assets and liabilities to reflect fair market values
C. Loans to and from partnership
D. All of the above
(Dayag, 2015)
121. On December 1, 2015, EE and FF formed a partnership, agreeing to share for profits and
losses in the ratio of 2:3, respectively. EE invested a parcel of land that cost him P25,000.
FF invested P30,000 cash. The land was sold for P50,000 on the same date, thress hours
after formation of the partnership. How much should be the capital balance of EE right
after formation?

A. P25,000
B. P30,000
C. P60,000
D. P50,000
(Dayag, 2015)

122. MM, NN, and OO are partners with capital balances on December 31, 2015 of P300,000,
P300,000 and P200,000, respectively. Profits are shared equally. OO wishes to withdraw
and it is agreed that OO is to take certain equipment with second-hand value of P50,000
and a note for the balance of OO’s interest. The equipment are carried on the books at
P65,000. Brand new equipment may cost P80,000. Compute for: (1) OO’s acquisition of
the second-hand equipment will result to reduction in capital; (2) the value of the note that
will OO get from the partnership’s liquidation.

A. (1) P15,000 each for MM and NN, (2) P150,000


B. (1) P5,000 each for MM, NN and OO, (2) P145,000
C. (1) P5,000 each for MM, NN and OO, (2) P195,000
D. (1) P7,500 each for MM and NN, (2) P145,000
(Dayag, 2015)

123. JJ and KK are partners who share profits and losses in the ratio of 60%: 40%, respectively.
JJ’s salary is P60,000 and P30,000 for KK. The partners are also paid interest on their
average capital balances. In 2015, JJ received P30,000 of interest and KK, P12,000. The
profit and loss allocation is determined after deductions for the salary and interest
payments. If KK’s share in the residual income (income after deducting salaries and
interest) was P60,000 in 2015, what was the total partnership income?
A. P192,000
B. P345,000
C. P282,000
D. P387,000
(Dayag, 2015)

124. CC, PP, and AA, accountants, agree to form a partnership and to share profits in the ratio
of 5:3:2. They also agreed that AA is to be allowed a salary of P28,000, and that PP is to
be guaranteed P21,000 as his share of the profits. During the first year of operation, income
from fees are P180,000, while expenses total P96,000. What amount of net income should
be credited to each partner’s capital account?

A. CC, P28,000, PP, P16,800, AA, P11,200


B. CC, P25,000, PP, P21,000, AA, P38,000
C. CC, P24,000, PP, P22,000, AA, P38,000
D. CC, P25,000, PP, P21,000, AA, P39,000
(Dayag, 2015)

125.The following condensed balance sheet is presented for the partnership of AA, BB, and
CC, who share profits and losses in the ratio of 4:3:3, respectively:

Cash P160,000
Other Assets 320,000
Total P480,000

Liabilities P180,000
AA, capital 48,000
BB, capital 216,000
CC, capital 36,000
Total P480,000

The partners agreed to dissolve the partnership after selling the other assets for P200,000.
Upon dissolution of the partnership, AA should have received

A. P0
B. P48,000
C. P72,000
D. P84,000
(Dayag, 2015)

126. Larry, Marsha, and Natalie are partners in accompany that is being liquidated. They share
profits and losses 55 percent, 20 percent, and 25 percent, respectively. When the liquidation
begins, they have capital account balances of P108,000, P62,000 and P56,000,
respectively. The partnership just sold equipment with a historical cost and accumulated
depreciation of P25,000 and P18,000, respectively for P10,000. What is the balance in
Marsha’s capital account after the transaction is completed?

A. P62,000
B. P61,400
C. P62,600
D. P65,000
(Dayag, 2015)

127.Partner A first contributed P50,000 of capital into an existing partnership on March 1,


2015. On June 1, 2015, the partner contributed another P20,000. Withdrawals in excess of
P10,000 are charged to the partner’s capital account. The annual weighted-average capital
balance is

A. P62,000
B. P51,667
C. P60,000
D. P48,333

(Dayag, 2015)
128. (1) All assets contributed to the partnership are recorded by the partner at their agreed
values.
(2) All liabilities that the partnership assumes are recorded at their net present values.

A. Only the first statement is correct


B. Only the second statement is correct
C. Both statements are correct
D. Both statements are incorrect
(Dayag, 2015)

129. If a partnership has only non-cash assets, all liabilities have been properly
disbursed, and no additional liquidation expenses are expected, the maximum potential
loss to the partnership in the liquidation process is:
A. The fair market value of the non-cash assets
B. The book value of the non-cash assets
C. The estimated proceeds from the sale of the assets less the book value of the
non-cash assets.
D. None of the above.
(RESA Pre-Board July 2017)

130. In partnership,

A. Management consists of the board of directors


B. Profits are always divided equally among partners
C. Dissolution results when a partner leaves the partnership
D. No partner is liable for more than a proportion of the company’s debt
(RESA Pre-Board April 2016)

131. Mr. MAC is admitted into the partnership of Do and Nald by investing cash
equivalent to ¼ of their capital. Which of the following is true after the admission of
Mr. MAC?

A. Assets of the partnership will increase


B. Total partner’s equity remain the same
C. Do and Nald capital decreased by ¼.
D. Assets of the partnership will remain the same
(RESA Pre-Board July 2017)

132. Under the bonus method, when a new partner is admitted to the partnership, the
total capital of the new partnership is equal to:

A. The book value of the previous partnership plus the fair market value of the
consideration paid to the existing partnership by the incoming partner.
B. The book value of the previous partnership plus any necessary asset write-ups from
book value to market value plus the fair market value of the consideration paid to the
existing partnership by the incoming partner.
C. The book value of the previous partnership minus any asset write downs from book to
market value plus the fair market value of the consideration paid to the existing
partnership by the incoming partner.
D. The fair market value of the new partnership as implied by the value of the incoming
partner’s consideration in exchange for an ownership percentage in the new
partnership.
(RESA Pre-Board July 2017)
133. Which of the following statements is correct regarding a partner’s debit capital
balances?

A. The partner should make contributions to reduce the debit balance to whatever extent
possible.
B. If contributions are not possible, the other partners with credit capital balances will be
allocated a portions of the debit balance based on their proportionate profit-and-loss-
sharing percentages.
C. Partners who absorb another’s debit capital balance have a legal claim against the
deficient partner.
D. All of these statements are correct.
(RESA Pre-Board July 2017)

134. Following is the balance sheet of the ABCD Partnership at March 31, 2018, when
the partnership is to be liquidated:

Cash P 6, 000 Liabilities P 12, 400


Other Assets 126, 000 A, Loan 12, 000
B, Loan 14, 400
D, Loan 9, 600
A, Capital – 25% 16, 200
B, Capital – 25% 12, 000
C, Capital – 25% 37, 700
D, Capital – 25% 17, 700

During the month of April 2018, assets having a book value of P 18, 000 are sold at a loss of
P 2, 400. Liquidation expenses of P 600 are paid as well as P 7, 200 of the liabilities. Of the
liabilities shown in the balance sheet, P 240 represents salary payable to D and P 160 represents
salary payable to C.

On April 30, 2018 cash to be distributed to A, B, C and D as follows:

A B C D
A. P 0 P 0 P 0 P 9, 000
B. P 1, 950 P 1, 950 P 1, 950 P 1, 950
C. P 0 P 0 P 0 P 1, 950
D. P 0 P 0 P 9, 000 P 0
(RESA Pre-Board July 2017)
135. Cheryl is the manager of a local store. She is also a partner in the company and she
receives a bonus as part of the profit and loss allocation. Cheryl’s bonus is based on the
increase in revenues recorded during the period. The bonus arrangement is that Cheryl
receives 1 percent of net income for every full percentage point growth for revenues in
excess of a 5 percent revenue growth. During the most recent period, revenues grew
from P500, 000 to P540, 000 and net income grew from P 98, 000 to P 120, 000. How
much bonus does Cheryl receive for this period?

A. P 1, 100
B. P 3, 600
C. P 2, 000
D. P 6, 000
(RESA Pre-Board July 2017)

Use the following information for questions 8 and 9:

Cleary, Wasser, and Nolan formed a partnership on January 1, 20x4, with investments of
P 100, 000, P 150, 000, and P 200, 000, respectively. For division of income, they agreed
to (1) interest of 10% of the beginning capital balance each year, (2) annual compensation
of P 10, 000 to Wasser and (3) sharing the remainder of the income or loss in a ratio of
20% for Cleary and 40% each for Wasser and Nolan. Net income was P 150, 000 in 20x4
and P 180, 000 in 20x5. Each partner withdrew P 1, 000 for personal use every month
during 20x4 and 20x5.

136. What was Wasser’s share of income for 20x4?

A. P 63, 000
B. P 53, 000
C. P 58, 000
D. P 29, 000
E. P 51, 000
(RESA Pre-Board April 2016)

137. What was Wasser’s capital balance at the end of 20x5?


A. P 201, 000
B. P 263, 520
C. P 264, 540
D. P 304, 040
E. P 313, 780
(RESA Pre-Board April 2016)
138. XX, YY, and ZZ, a partnership formed on January 1, 2018 had the following initial
investment:
XX ………………………………………P 170, 000
YY ………………………………………. 255, 000
ZZ ………………………………………. 382, 500

The partnership agreement states that the profits and losses are to be shared equally by the
partners after consideration is made for the following:
- Salaries allowed to partners: P102, 000 for XX, P81, 600 for YY, and P61, 200 for ZZ.
- Average partners’ capital balances during the year shall be allowed 10%.

Additional information:
- On June 30, 2018, XX invested an additional P102, 000.
- ZZ withdrew P119, 000 from the partnership on September 30, 2018.
- Share the remaining partnership profit was P 8, 500 for each partner.

The total partnership capital on December 31, 2018 was:


A. P 688, 500
B. P 1, 141, 550
C. P 816, 000
D. d. P 1, 143, 675
(RESA Pre-Board July 2017)

139. At the time of partnership liquidation, which credits shall be settled first?

A. Those amount owing to third persons.


B. Those amount owing to partners other than capital contribution and share in profit.
C. Those amount owing to partners with respect to capital contribution.
D. Those amount owing to partners with respect to share in profit.
(CPAR Reviewer, 2017)
140. How should the net profit or net loss of the partnership be divided among the
partners, whether capitalist or industrial?

A. In accordance with their capital contribution ratio.


B. In accordance with just and equitable sharing taking into account the circumstances of
the partnership.
C. Equally
D. In accordance with the partnership agreement.
(CPAR Reviewer, 2017)
141. At the date of partnership formation of a partnership, the amount credited to A’s
capital is less than the fair value of the property contributed. Which is the most valid
reason?

A. The property contributed by A is impaired.


B. The property contributed by A has been subjected to positive asset revaluation.
C. Bonus has been given by partner A to the other partners.
D. Goodwill arising from partnership formation has been recognized.
(CPAR Reviewer, 2017)

142. When a new partner is admitted to an existing partnership through the purchase of
a portion of existing interest of an incumbent partner, which statement is correct?

A. The total capital of the old and new partnership will be the same.
B. The partnership will recognize gain or loss on the difference between the amount paid
and capital transferred.
C. Goodwill may be recognized by virtue of the admission.
D. There will be increase in the total assets of the partnership equivalent to the amount
paid by the newly admitted partner.
(CPAR Reviewer, 2017)

143. At the time of retirement, a retiring partner receives more than the amount of his
capital contribution while the remaining partners capital increase after the retirement.
Which of the following is most valid reason?

A. Goodwill during retirement is recognized.


B. Asset revaluation is recognized.
C. Bonus is given by retiring partner to remaining partners.
D. Bonus is given by the remaining partners to retiring partner.
(CPAR Reviewer, 2017)

144. On January 1, 2017, Toni, Abbie and JM entered into articles of co-partnership for
the operation of TAJ computer shop. Toni contributed investment property with assessed
value of P1,700,000 subject to mortgage payable of P500,000 to be assumed by the
partnership. Abbie contributed computer equipment with cost of P600,000 with
accumulated depreciation of P200,000. The fair market value of the computer equipment
is P300,000.
On January 2, 2017, the partnership was able to sell the investment property for
P2,000,000. How much cash shall be contributed by JM if the articles of co-partnership
provide that Toni will have 60% interest in the partnership?

A. 500,000
B. 700,000
C. 800,000
D. 600,000

(CPAR Reviewer, 2017)

145. On January 1, 2017, Yazzi, Angel and Nadine organized YAN partnership by
investing P5M, 2M and P3M for capital interest ratio of 4:5:1 respectively. Nadine has
been appointed as managing partner. During year 2017, YAN partnership reported net
income of P3,000,000. Their profit/loss distribution and drawing agreement are presented
below:

• 20% interest on beginning capital


• P10,000, P20,000 and P50,000 monthly salary, respectively
• 25% bonus of net income after interest and salary to managing partner
• The remainder will be divided equally among the partners.
• The partners must withdraw at the end of the year 50% of their share in net income for
the period.

What is the capital balance of Nadine on December 31, 2017?

A. 1,410,000
B. 3,410,000
C. 1,610,000
D. 3,610,000
(CPAR Reviewer, 2017)
146. On December 31, 2017, the capital balance of partners Cristy, Paula and Ara of
CPA Partnership are P1M, P3M and P6M, respectively with profit or loss agreement ratio
of 4:1:5. On January 1, 2018, Cristy decided to retire and received P400,000 from the
partnership.

If the assets of the partnership are not properly valued at the time of retirement, how much
is the capital balance of Paula after the retirement of Cristy?

A. 2,900,000
B. 2,850,000
C. 3,100,000
D. 3,150,000
(CPAR Reviewer, 2017)
Numbers 147 and 148
On December 31, 2017, the Statement of Financial Position of DEF with profit or loss ratio
of 4:1:5 is presented below:

Cash 2M Liability to third person 4M


Noncash asset 8M D, capital 3.5M
E, capital 1.5M
F, capital 1M

On January 31, 2018, DEF partnership has been subjected to installment liquidation. As of
January 31, 2018, the following data concerning liquidation are provided:

➢ Noncash asset with book value of P6M has been sold at a loss of P2M.
➢ Liquidation expense amounting to P400,000 has been incurred for the month of
January.
➢ P600,000 cash has been withheld for future liquidation expense.
➢ P3M liability has been paid.

147. What is F’s share in the maximum possible loss on January 31, 2018?

A. 1,300,000
B. 1,000,000
C. 1,500,000
D. 500,000

148. What is the amount received by E on January 31, 2018?

A. 300,000
B. 700,000
C. 1,000,000
D. 0
149. On December 1, 2011, EE and FF formed a partnership, agreeing to share for profits and
losses in the ratio of 2:3, respectively. EE invested a parcel of
land that cost him P25,000. FF invested P30,000 cash. The land was sold for
P50,000 on the same date, three hours after formation of the partnership.
How much should be the capital balance of EE right after formation?

A. P25,000
B. 30,000
C. 20,000
D. 50,000

150. MM, NN, and OO are partners with capital balances on December 31
2011 of P300,000, P300,000 and P200,000, respectively. Profits are shared
equally. OO wishes to withdraw and it is agreed that OO is to take certain
equipment with second-hand value of P50,000 and a note for the balance
of OO's interest. The equipment are carried on the books at P65,000. Brand
new equipment may cost P80,000. Compute for: (1) OO's acquisition of
the second-hand equipment will result to reduction in capital; (2) the value
of the note that will OO get from the partnership's liquidation.

A. (1) P15.000 each for MM and NN,(2) P150,000.


B. (1) P5,000 each for MM, NN and OO,(2) P145,000
C. (1) P5,000 each for MM. NN and OO,(2) P195,000
D. (1) P7,500 each for MM and NN,(2) P145,000.

151. JJ and KK are partners who share profits and losses in the ratio of 60%: 40%
respectively. JJ's salary is P60,000 and P30,000 for KK. The partners are also
paid interest on their averdge capital balances. In 2011, JJ received P30.000
of interest and KK, P12,000. The profit and loss allocation is determined
after deductions for the salary and interest payments. If KK's share in the
residual income (income after deducting salaries and interest) was P60,000
in 2011,

What was the total partnership income?


A. P192,000
B. 345,000
C. P282,000
D. 387,000

152. Lancelot is trying to decide whether to accept a salary of P40.000 or a


salary of P25.000 plus a bonus of 10% of net income after salary and bonus
as a means of allocating profit among the partners. Salaries traceable to
the other partners are estimated to be P100,000. What amount of income
would be necessary so that Lancelot would consider the choices to be
equal

A. P165,000
B 290,000
C. P265,000
D. 305,000

153. Merlin, a partner in the Camelot Partnership, has a 30% participation in


partnership profits and losses. Merlin's capital account has a net decrease
of P.200.000 during the calendar year 2011. During 2011, Merlin withdrew
P2.600,000 (charged against his capital account) and contributed property
valued at P500,000 to the partnership. What was the net income of the
Camelot Partnership for year 2011?

A. P3,000,000
B. 4,666,667
C. P 7,000,000
D. 11,000,000

154. The partnership agreement of XX, YY & ZZ provides for the


allocation of net income in the following order:

First, XX is to receive 10% of net income up to P200.000 and 20% over P200,000.
Second, YY and ZZ each are to receive 5% of the remain income over P300,000
The balance of income is to be allocated equally among the three partners.
The partnership's 2011 net income was P500,000 before any allocations to
partners. What amount should be allocated to XX?

A. P202,000
B. 216,000
C. P206,000
D. 220,000

155. On April 30, 2011, XX, YY and ZZ formed a partnership by combining their
separate business proprietorships. XX contributed cash of P75,000. YY
contributed property with a P54,000 carrying amount, a P60,000 original
cost, and P120,000 fair value. The partnership accepted responsibility for
the P52,500 mortgage attached to the property. ZZ contributed equipment
with a P45,000 carrying amount, a P112,500 original cost, and P82,500 fair
value. The partnership agreement specifies that profits and losses are to
be shared equally but is silent regarding capital contributions. Which
partner has the largest April 30, 2011, capital balance?

A. XX
B. YY
C. ZZ
D. All capital account balances are equal

156. The Partnership has the following accounting amounts:

(1) Sales P70,000


(2) Cost of Goods Sold P40,000
(3) Operating Expenses P10,000
(4) Salary allocations to partners P13,000
(5) Interest paid to banks P2,000
(6) Partners' withdrawals P8,000
The partnership net income (loss) is:

A. P20,000
B 18,000
C. P 5,000
157. The capital accounts of the partnership of NN, vv, and JJ on lune
are presented below with their respective profif and loss ratios: (P139,200 1/2 , 208,800 1/3 ,
96,000 1/6) On June 1, 2011, LL is admitted to the partnership when LL purchased, for
P132,000, a proportionate interest from NN and JJ in the net assets and
profits of the partnership. As a result of a transaction LL acquired a
fifth interest in the net assets and profits of the firm. What is the combined gain realized by
NN and JJ upon.the sale of a portion of their interest in
the partnership to LL?

A. P 0
B. 43,200
C P62,400
D. 82,000

158. On January 31, 2011, partners of Lon, Mac & Nan, LLP, had the following
loan and capital account balances (after closing entries for January):

Loan receivable from Lon 20,000 dr


Loan payable to Nan 60,000 cr
Lon, capital 30,000 dr
Mac, capital 120,000 dr
Nan, capital 70.000cr

The partnership's income sharing ratio was Lon, 50%; Mac, 20%, and Nan,
30%. On January 31, 2011, Ole was admitted to the partnership for a 20%
interest in total capital of the partnership in exchange for an investment
of P40,000 cash. Prior to Ole's admission, the existing partners agreed to
increase the carrying amount of the partnership's inventories to current
fair value, a P60,000 increase. The capital account to be credited to Ole:

A. P60,000
B. P40,000
C. P52,000
D. P46,000

159. Which of the following transactions will not affect the total equity of the partnership?

A. Recognition of impairment loss in case of admission of a new partner


B. Withdrawal of a partner
C. Admission of a new partner by purchase of existing partner’s interest below its
book value
D. Retirement of an existing partner with payment of above the book value of such
interest
(CPAR Final Pre-board Examination May 2017)

160. A, B, and C are partners with average capital balances during 2017 of P472,500, P238,650
and P162,350; respectively. The partners receive 10% interest on their average capital
balances; after deducting salaries of P122,325 to A and P82,625 to C, the residual profit or loss
is divided equally.

In 2017, the partnership had net loss of P125,624 before interest and salaries to partners.
What amount should A and C capital change respectively?

A. P40,844 decrease and P31,237 decrease


B. P30,267 increase and P40,448 decrease
C. P29,476 increase and P17,536 increase
D. P28,358 increase and P32,458 increase
(CPAR Final Pre-board Examination May 2017)

161. A partner was admitted in an existing partnership through investment of cash equivalent to
¼ of the new capitalization. If the capital balance of the old partners increases, what is the most
valid reason under Philippine GAAP?

A. Asset revaluation of existing partnership’s assets


B. Impairment loss of existing partnership’s assets
C. Recognition of goodwill of existing partnership
D. Receipt of bonus from the new partner
(CPAR Final Pre-board Examination May 2017)

162. Which of the following statements concerning the formation of partnership business is
correct?

A. PFRS allows recognition of goodwill arising from the formation of partnership


B. The juridical personality of the partnership arises from the issuance of
certification of registration
C. The parties may become partners only upon contribution of money or property but
not of industry or service
D. The capital to be credited to each partner upon formation may not be the amount
actually contributed by each partner
(CPAR Final Pre-board Examination May 2017)

163. Regina, Jessica and Nataly entered into a contract of partnership with a total capital
contribution of P5,000. The parties failed to register its articles of co-partnership with the
Securities and Exchange Commission. Which of the following statements is correct?

A. The contract of partnership is void because the law provides that when the capital
contribution is at least P3,000 it must be registered with Securities and Exchange
Commission
B. The contract of partnership will bind third persons
C. The contract of partnership remains to be valid
D. The partnership does not obtain juridical personality for failure to register with
Securities and Exchange Commission.
(CPAR Final Pre-board Examination May 2017)

164. In the absence of agreement as to distribution of profit, how shall the partnership profit be
distributed to the partners?

A. The industrial partner shall receive a share equivalent to the least share of a
capitalist partner while the capitalist partners shall share based on capital
contribution ratio.
B. The industrial partner shall receive a just and equitable share and the remainder
shall be distributed to the capitalist partners on the basis of capital contribution
ratio
C. The profit shall be distributed on the basis of loss contribution ratio which may
have been agreed upon by the partners
D. The profit shall be distributed equally to all partners including the industrial
partner

(CPAR Final Pre-board Examination May 2017)

165. On July 1, 2016, Anne, Bianca and Carla formed a business partnership to be operated as
an advertising agency. Anne contributed P10M cash while Bianca shall have a capital credit
of P6M upon receipt of bonus of P1M from Anne based on the provision in Articles of Co-
Partnership. The terms of the agreement provide that Anne and Bianca shall have a combined
40% capital interest in the newly formed partnership. What is the capital contribution made by
Carla to the partnership?

A. P24,000,000
B. P22,500,000
C. P25,000,000
D. P32,000,000
(CPAR Final Pre-board Examination May 2017)

166. On January 1, 2017, Angel, Bea and Colleen formed ABC & Co., a general professional
partnership for the exercise of their common profession. Angel contributed a building with a
cost of P5M and accumulated depreciation of P4M. Based on the city assessor’s records, the
building has an assessed value of P2M. The building has an annotated mortgage payable
amount to P500,000 to be assumed by the partnership.
On the other hand, Bea contributed 10,000 shares of stocks with par value of P200/share
and prevailing quoted price of P300/share. On January 2, 2017, the building contributed by
Angel was sold for P5.5M. If Colleen wants to have 20% capital interest in the newly
formed partnership, how much cash shall be contributed by her?

A. P875,000
B. P1,125,000
C. P2,125,000
D. P2,000,000
(CPAR Final Pre-board Examination May 2017)

167. On January 1, 2014, AB and QR agreed to form a partnership. The following are their
assets and liabilities:
Accounts AB QR
Cash 136,000 76,000
Accounts Receivable 88,000 48,000
Inventories 304,000 364,000
Machinery 480,000 440,000
Accounts Payable 216,000 144,000
Notes Payable 140,000 60,000

AB decided to pay off his notes payable from his personal assets. It was also agreed that
QR inventories were overstated by P24,000 and AB machinery was over depreciated by
P20,000. QR is to invest/withdraw cash in order to receive a capital credit that is 20% more
than AB’s total net investment in the partnership.

How much cash will be presented in the partnership’s statement of financial position?

A. 486,400
B. 546,300
C. 250,400
D. 640,300
(CPAR Final Pre-board Examination May 2017)

168. On December 1, 2014, MG and AN are combining their separate businesses to form a
partnership. Cash and noncash assets are to be contributed. The noncash assets to be
contributed and the liabilities to be assumed are as follows:

MG AN
Book value Fair value Book value Fair value
Accounts Receivable 250,000 262,500 200,000 195,000
Inventory 400,000 450,000 200,000 207,500
PPE 1,000,000 912,500 862,500 822,500
Accounts Payable 150,000 150,000 112,500 112,500

MG and AN are to invest equal amount of cash such that the contribution of MG would be 10%
more than the investment of AN. What is the amount of cash presented on the partnership’s
statement of Financial Position on December 1, 2014?

A. 5,025,000
B. 5,500,000
C. 5,750,000
D. 4,950,000
(CPAR Final Pre-board Examination May 2017)

169. After the admission of a new partner, the total partnership capital increased by the fair
value of the new partner’s net contributions to the partnership. The admission was accounted
for
A. Under the goodwill method
B. Under the bonus method
C. As a purchase of interest
D. As an investment in the partnership
(Millan, 2016)

170. If a new partner acquires a partnership interest directly from the partners rather than from
the partnership itself,
A. No entry is required.
B. The partnership should be revalued.
C. The existing partners’ capital accounts should be reduced and the new partner’s
account increased.
D. The partnership has undergone a quasi-reorganization.
(Punzalan, 2015)

171. Which of the following is not a characteristic of a partnership?


A. Limited liability
B. Limited life
C. Mutual agency
D. Ease of formation
(Punzalan, 2015)

172. Abel and Carr formed a partnership and agreed to divide initial capital equally, even though
Abel contributed P100,000 and Carr contributed P84,000 in identifiable assets. Under the
bonus approach to adjust the capital accounts, Carr’s unidentifiable assets should be debited
for
A. 46,000
B. 16,000
C. 8,000
D. 0
(Punzalan, 2015)

173. Alder, Benson and Carl are capitalist partners and Denver, an industrial partner. The
partnership reported a net loss of P100, 000. How much is the share of Denver in the reported
net loss?
A. 0
B. 10,000
C. 25,000
D. 100,000
(Punzalan, 2015)

174. XYZ Partnership provided for the following in their distribution of profits and losses:
First: X to receive 10 % of net income up to P100,000 and 20% of the amount in excess
thereof.
Then: Y and Z are each to receive 5% of the remaining income in excess of P150,000
after X’s share.
Finally: The balance is to be distributed equally to the three partners.

If the partnership earned a net income of P250,000, what is the total share of Partner X?

A. 100,000
B. 108,000
C. 110,000
D. 130,000
(Punzalan, 2015)

175. After incurring losses resulting from very unprofitable operations, the Goh Kong Wei
Partnership decided to liquidate when the partners’ capital balances were:

Goh, Capital (40%) P80,000


Kong, Capital (40%) 130,000
Wei, Capital (20%) 96,000

The noncash assets were sold in installment. Available cash were distributed to partners in
every sale of noncash assets. After the second sale of noncash assets, the partners received
the same amount of cash in the distribution. And from the third sale of noncash assets, cash
available for distribution amounts to P28,000, and unsold noncash assets has a book value
of P12,500. Using cash priority program, what amount did Wei receive in the third
installment of cash?

A. 11,600
B. 8,000
C. 5,600
D. 0
(Punzalan, 2015)

176. The condensed balance sheet of Adams & Gray, a partnership, at December 31, 2014,
follows:

Current assets P250,000


Equipment (net) 30,000
Total assets P280,000

Liabilities P20,000
Adams, Capital 160,000
Gray, Capital 100,000
Total liabilities and capital P280,000

On December 31, 2014, the fair values of the assets and liabilities were appraised at
P240,000 and P20,000, respectively, by an independent appraiser. On January 2, 2015, the
partnership was incorporated and 1,000 shares of P5 par value common stock were issued.
Immediately after the incorporation, what amount should the new corporation report as
additional paid in capital?
A. 270,000
B. 260,000
C. 215,000
D. 0
(Punzalan, 2015)

177. Partner Morgan is personally insolvent, owing P600,000. Personal assets will only bring
P200,000 when liquidated. At the same time, Morgan has a credit balance of P120,000. The
capital amounts of the other partners total a balance of P250,000. Under the doctrine of
marshalling of assets, how much the personal creditors of Morgan can collect?
A. 120,000
B. 200,000
C. 320,000
D. 570,000
(Punzalan, 2015)

178. The partnership agreement of Reid and Simm provides that 10% per year is to be credited
to each partner on the basis of weighted-average capital balances. A summary of Simm’s
capital account for the year-ended December 31, 2014, is as follows:

Balance, January 1 P140, 000


Additional Investment, July 1 40, 000
Withdrawal, August 1 (15, 000)
Balance, December 31 165, 000

What amount of interest should be credited on Simm’s capital account for 2014?
A. 15,250
B. 15,375
C. 16,500
D. 17,250
(Punzalan, 2015)

179. The fact that salaries paid to partners are not a component of partnership income is
indicative of
A. A departure from generally accepted accounting principles
B. Being characteristic of the entity theory
C. Being characteristic of the proprietary theory
D. Why partnerships are characterized by unlimited liability
(Punzalan, 2016)
180. The doctrine of marshalling of assets

A. Is applicable only if the partnership is insolvent


B. Allows partners to first contribute personal assets to unsatisfied partnership creditors
C. Is applicable if either the partnership is insolvent or individual partners are insolvent
D. Amount owed to personal creditors and to partnership for debit capital balances are
shared proportionately from the personal assets of the partners
(Punzalan, 2016)

181. If goodwill is traceable to the incoming partner, the new partner's capital balance equals

A. the fair market value of consideration paid by the incoming partner


B. the book value of the older partnership divided by the existing partners' ownership
percentage in the new partnership minus the book value of the old partnership.
C. incoming partner's ownership percentage multiplied by the capital of the new
partnership
D. none of the above.

(Guerrero, 2014)

AY and AN are partners who have the agreement to share profit and loss in the following manner:

AY AN
Annual salaries 261,000 259,000
Interest on average balances 5% 10%
Bonus (based on net income after salaries and interest) 10%
Remainder 50% 50%

During the year ended December 31, 2014, the partnership generated a profit of P575,000 before
any deductions. AY’s and AN’s average capital balances for the year are P600,000 and P300,000,
respectively. Income is distributed to the partners only as far as it is available.

182. How much is the total share of AN in the net income for the year ended 2014?

A. P286,500 C. P288,500
B. P287,500 D. P295,665
(RESA, 2014)
On January 1, 2014, L, M, and N formed a partnership with capital contributions of P625,000;
P750,000; and P937,500, respectively. The partners agreed that profit and loss would be allocated
as follows: P75,000 salary to each partner, 3% interest on initial capital contributions, the
remainder divided in the ratio 2:4:4, respectively to L, M, and N. The partnership generated income
amounting to P375,000 for the year 2014. During 2014, the following partnership errors were
discovered before the distribution of profit:

• In 2014, a purchase of piece of equipment costing P50,000 was expensed. The equipment has
an estimated life of ten years with equal service potential each year.
• On December 31, 2014, ending inventory was understated by P50,000.
On January 1, 2015, N decided to retire from the partnership.
183. If the balance of the capital of L after retirement amounts to P770,000, how much is the
settlement to N for his retirement?

A. P1,120,000 C. P1,085,000
B. P1,062,500 D. P1,110,875
(RESA, 2014)

184. If the balance of the capital of M after retirement amounts to P890,000, how much is the
settlement to N for his retirement?

A. P1,127,500
B. P1,090,500
C. P1,231,500
D. P1,152,500
(RESA, 2014)

On December 1, 2014, MG and AN are combining their separate businesses to form a partnership.
Cash and noncash assets are to be contributed. The noncash assets to be contributed and the
liabilities to be assumed are as follows:
MG AN
Book value Fair value Book value Fair value
Accounts Receivable 250,000 262,500 200,000 195,000
Inventory 400,000 450,000 200,000 207,500
PPE 1,000,000 912,500 862,500 822,500
Accounts Payable 150,000 150,000 112,500 112,500

MG and AN are to invest equal amount of cash such that the contribution of MG would be 10%
more than the investment of AN.
185. What is the amount of cash presented on the partnership’s statement of Financial Position
on December 1, 2014?
(RESA,2014)
A. P2,762,500
B. P2,512,500
C. P5,525,000
D. P5,025,000

On December 1, 2014, MV and CD agreed to invest equal amounts and share profits equally to
form a partnership. MV invested P3,120,000 cash and a piece of equipment. CD invested some
assets which are shown on the next page:

Book value

Accounts Receivable 400,000

Inventory 1,120,000

Machineries, net 2,240,000

Intangibles, net 920,000

The assets invested by CD are not properly valued, P32,000 of the accounts receivable are proven
uncollectible. Inventories are to be written down to P1,040,000. Included in the machineries is an
obsolete apparatus acquired for P384,000 with an accumulated depreciation balance of P336,000.
Part of the intangibles is a patent with a carrying value of P56,000 which was sued upon by a
competitor. CD unsuccessfully defended the case and the final decision of the court was released
on November 29, 2014.

186. What is the fair value of the equipment invested by MV?

A. P1,400,000 C. P1,344,000
B. P968,000 D. P1,560,000
(RESA, 2014)
The partnership of CD, AY, and GP decided to liquidate their partnership on May 31, 2013. Before
liquidating and sharing of net income, their capital balances are as follows: CD (30%) P875,000,
AY (30%) P630,000, and GP (40%) P770,000. Net income from January 1 to May 31 is P420,000.
Liabilities of the partnership amounted to P735,000 and its total assets include cash amounting to
P245,000. Unsettled liabilities are P385,000. CD invested additional cash enough to settle their
partnership’s indebtedness. AY is personally solvent, GP is personally insolvent, and CD becomes
insolvent after investing the cash needed by the partnership.

187. How much were the partnership’s non-cash sold for?


A. P157,500 C. P105,000
B. P3,080,000 D. P525,000

188. How much will CD receive as a result of their liquidation?


A. P385,000
B. 0
C. P315,000
D. P462,000
(RESA, 2014)
189. On April 30, 2016, Al, Ben, and Ces formed a partnership by combining their separate
business proprietorships. Al contributed cash of P50,000. Ben contributed property with a
P36,000 carrying amount, a P40,000 original cost, and P80,000 fair value. The partnership
accepted responsibility for the P35,000 mortgage attached to the property. Ces contributed
equipment with a P30,000 carrying amount, a P75,000 original cost, and P55,000 fair value.
The partnership agreement specifies that profits and losses are to be shared equally but is
silent regarding capital contributions. Which partner has the largest capital account balance
at April 30, 2016?
A. Al
B. Ben
C. Ces
D. All capital balances are equal
(Punzalan, 2015)

190. A partnership records a partner’s investment of assets in the business at


A. The market value of the assets invested.
B. A special value set by the partners.
C. The partner’s book value of the assets invested.
D. Any of the above, depending upon the partnership agreement.
(RPCPA 0598)
191. In the Adel-Brick partnership, Adel and Brick had a capital ratio of 3:1 and a profit and
loss ratio of 2:1, respectively. The bonus method was used to record Colter’s admittance as a
new partner. What ratio would be used to allocate, to Adel and Brick, the excess of Colter’s
contribution over the amount credited to Colter’s capital account?

A. Adel and Brick’s new relative capital ratio.


B. Adel and Brick’s new relative profit and loss ratio.
C. Adel and Brick’s old capital ratio.
D. Adel and Brick’s old profit and loss ratio.
(AICPA 0r92 T-35)
192. The final cash distribution to the partners in a partnership in liquidation should be made in
accordance with

A. Balances of the partners’ capital accounts.


B. Partners’ profit and loss sharing ratio.
C. Ratio of capital contributions made by the partners.
D. Ratio of capital contributions less withdrawals made by the partners.
(RPCPA 1081,0586)
193. K, L, and M are partners with average capital balance during 2011 of P472,500, P238,650,
and P162,350, respectively. The partners receive 10% interest on their average capital
balances; after deducting salaries of P122,325 to K and P82,625 to M, the residual profits or
loss is divided equally.

In 2011, the partnership had a net loss of P125,624 before the interest and salaries to
partners.

By what amount should K’s and M’s capital account change?


K’s Capital Account M’s Capital Account
A. P40,844 decrease P31,235 decrease
B. P28,358 increase P32,458 increase
C. P29,476 increase P17,536 increase
D. P30,267 increase P40,448 decrease
(Guerrero, 2013)
194. Prior to partnership liquidation, a schedule of possible losses is frequently prepared to
determine the amount of cash that may be safely distributed to the partners. The schedule of
possible losses

A. Consists of each partner’s capital account plus loan balance, divided by that
partner’s profit-and-loss sharing ratio.
B. Shows the successive losses necessary to eliminate the capital accounts of partners
(assuming no contribution of personal assets by partners).
C. Indicates the distribution of successive amounts of available cash to each partner.
D. Assumes contribution of personal assets by partners unless there is a substantial
presumption of personal insolvency by the partners.
(Gleim)
195. The following condensed balance sheet is presented for the partnership of Axel, Barr, and
Cain, who share profits and losses in the ratio of 4:3:3, respectively:
Cash P100,000
Other assets 300,000
Total 400,000

Liabilities P150,000
Axel, Capital 40,000
Barr, Capital 180,000
Cain, Capital 30,000
Total 400,000

The partners agreed to dissolve the partnership after selling the other asset for P200,000.
Upon dissolution of the partnership, Axel should have received

A. 0
B. 40,000
C. 60,000
D. 70,000
(Punzalan, 2015)
196. The following blance sheet is presented for the partnership of A, B, and C, who share
profits and losses in the respectively ratio of 5:3:2.
Assets Liabilities and Capital
Cash P 120,000 Liabilities P280,000
Other Assets 1,080,000 A, Capital 560,000
B, Capital 320,000
C, Capital 40,000
Total P1,200,000 Total P1,200,000

Assume that the three partners decided to liquidate the partnership. If the other assets are
sold for P800,000, how should the available cash be distributed to each partner?
A B C
A. 280,000 320,000 40,000
B. 324,000 236,000 16,000
C. 410,000 230,000 0
D. 412,000 228,000 0
(Punzalan, 2015)

197. Red, White, and Blue form a partnership on May 1, 2011. They agree that Red will
contribute office equipment with a toal fair value of P40,000; White will contribute delivery
equipment with a fair value of P80,000; and Blue will contribute cash. If Blue wants a on third
interest in the capital and profits, he should contribute cash of:
A. P40,000
B. P120,000
C. P60,000
D. P180,000
(Guerrero, 2013)

198. AK and BK decided to form a partnership on October 1, 2014. Their Statement of


Financial Position on this date were:
AK Bk
Cash 65,625.00 164,062.50
Accounts Receivable 1,487,500.00 896,875.00
Merchandise Inventory 875,000.00 885,937.50
Equipment 656,250.00 1,268,750.00
Total 3,084,375.00 3,215,625.00

Accounts Payable 459,375.00 1,159,375.00


AK, Capital 2,625,000.00
BK, Capital 2,056,250.00
Total 3,084,375.00 3,215,625.00

They agreed the following adjustments shall be made:


• Equipment of AK is underdepreciated by P87,500 and that BK is overdepreciated by
P131,250.
• Allowance for doubtful accounts is to be set up amounting to P297,500 for AK and P196,875
for BK.
• Inventories of P21,875 and P15,312.50 are worthless in the books of AK and BK
respectively.
• The partnership agreement provides for a profit and loss ratio of 70% to AK and 30% to BK.
Assuming the use of transfer of capital method, how much is the agreed capital of AK to bring
the capital balances proportionate to their profit and loss ratio.
On January 1, 2014, AB and QR agreed to form a partnership. The following are their assets and
liabilities:
Accounts AB QR
Cash 136,000 76,000
Accounts Receivable 88,000 48,000
Inventories 304,000 364,000
Machinery 480,000 440,000
Accounts Payable 216,000 144,000
Notes Payable 140,000 60,000

AB decided to pay off his notes payable from his personal assets. It was also agreed that QR
inventories were overstated by P24,000 and AB machinery was over depreciated by P20,000. QR
is to invest/withdraw cash in order to receive a capital credit that is 20% more than AB’s total net
investment in the partnership.
How much cash will be presented in the partnership’s statement of financial position?
A. 2,935,406.25
B. 2,218,125.00
C. 1,975,312.50
D. 1,258,031.25
CPAR Pre-Boards October 2017

199. On December 1, 2014, MV and CD agreed to invest equal amounts and share profits
equally to form a partnership. MV invested P3,120,000 cash and a piece of equipment. CD
invested some assets which are shown on the next page:
Book value
Accounts Receivable 400,000
Inventory 1,120,000
Machineries, net 2,240,000
Intangibles, net 920,000

The assets invested by CD are not properly valued, P32,000 of the accounts receivable are
proven uncollectible. Inventories are to be written down to P1,040,000. Included in the
machineries is an obsolete apparatus acquired for P384,000 with an accumulated depreciation
balance of P336,000. Part of the intangibles is a patent with a carrying value of P56,000 which
was sued upon by a competitor. CD unsuccessfully defended the case and the final decision of
the court was released on November 29, 2014.
A. 274,000
B. 212,000
C. 486,000
D. 374,000
CPAR Pre-Boards October 2017

200. On December 1, 2014, MG and AN are combining their separate businesses to form a
partnership. Cash and noncash assets are to be contributed. The noncash assets to be
contributed and the liabilities to be assumed are as follows:
MG AN
Book value Fair value Book value Fair value
Accounts Receivable 250,000 262,500 200,000 195,000
Inventory 400,000 450,000 200,000 207,500
PPE 1,000,000 912,500 862,500 822,500
Accounts Payable 150,000 150,000 112,500 112,500

MG and AN are to invest equal amount of cash such that the contribution of MG would be 10%
more than the investment of AN.
What is the amount of cash presented on the partnership’s statement of Financial Position on
December 1, 2014?

A. 1,344,000
B. 1,244,000
C. 3,120,000
D. 2,180,000

CPAR Pre-Boards October 2017


201. CC Partnership began operations on June 1, 2014. On that date, CY and CR have capital
credits of P175,000 and P240,000, respectively. The partnership has the following profit-
sharing plan:
a.) 10% interest on partners’ capital balances at the end of the year
b.) P60,000 and P75,000 annual salaries for CY and CR, respectively.
c.) Remaining profit will be divided to CY and CR on a 3:2 ratio, respectively.

During the year, CY invested P150,000 worth of merchandise and withdrew P40,000 cash,
while CR invested P120,000 cash. The partnership earned a profit of P266,375 during the year.
How much is CY’s capital balance at the end of 2014?
A. 5,025,000
B. 2,512,000
C. 3,215,000
D. 1,223,750
CPAR Pre-Boards October 2017
202. CC Partnership began operations on June 1, 2014. On that date, CY and CR have capital
credits of P175,000 and P240,000, respectively. The partnership has the following profit-
sharing plan:
a) 10% interest on partners’ capital balances at the end of the year
b) P60,000 and P75,000 annual salaries for CY and CR, respectively.
c) Remaining profit will be divided to CY and CR on a 3:2 ratio,
respectively.

During the year, CY invested P150,000 worth of merchandise and withdrew P40,000 cash, while
CR invested P120,000 cash. The partnership earned a profit of P266,375 during the year.
How much is CY’s capital balance at the end of 2014?
A. 266,375
B. 426,625
C. 285,000
D. 150,000
CPAR Pre-Boards October 2017

203. Cherryhill and Hace had been partners for several years, and they decided to admit Quincy
to the partnership. The accountant for the partnership believed that the dissolved partnership
and the newly formed partnership were two separate entities. What method would the
accountant have used for recording the admission of Quincy to the partnership?

A) the bonus method.


B) the equity method.
C) the goodwill method.
D) the proportionate method.
E) the cost method.
CPAR Pre-Boards October 2017

204. When the hybrid method is used to record the withdrawal of a partner, the partnership

A) revalues assets and liabilities and records goodwill to the continuing partner but not to the
withdrawing partner.
B) revalues liabilities but not assets, and no goodwill is recorded.
C) can recognize goodwill but does not revalue assets and liabilities.
D) revalues assets but not liabilities, and records goodwill to the continuing partner but not
to the withdrawing partner.
E) revalues assets and liabilities but does not record goodwill.
CPAR Pre-Boards October 2017
205. The disadvantages of the partnership form of business organization, compared to
corporations, include
A) the legal requirements for formation.
B) unlimited liability for the partners.
C) the requirement for the partnership to pay income taxes.
D) the extent of governmental regulation.
E) the complexity of operations.
CPAR Pre-Boards October 2017
206. The advantages of the partnership form of business organization, compared to
corporations, include

A) single taxation.
B) ease of raising capital.
C) mutual agency.
D) Limited liability.
E) difficulty of formation.
CPAR Pre-Boards October 2017
207. The dissolution of a partnership occurs

A) only when the partnership sells its assets and permanently closes its books.
B) only when a partner leaves the partnership.
C) at the end of each year, when income is allocated to the partners.
D) only when a new partner is admitted to the partnership.
E) when there is any change in the individuals who make up the partnership.
CPAR Pre-Boards October 2017

208. If a partner’s capital balance is credited for an amount greater than or less than the fair
value of his net contribution, the excess or deficiency is called a
A. Bonus
B. Goodwill
C. Discount
D. Premium
(Millan, 2016)
209. If the partnership agreement does not specify how income is to be allocated, profits and
loss should be allocated
A. Equally
B. In proportion to the weighted average of capital invested during the period
C. Equitably so that partners are compensated for the time and effort expended
on behalf of the partnership
D. In accordance with their capital contributions
(Millan, 2016)
210. When property other than cash is invested in a partnership, at what amount should the
noncash property be credited to the contributing partner’s capital account?

A. Fair value at the date of contribution


B. Contributing partner’s original cost
C. Assessed valuation for property tax purposes
D. Contributing partner’s tax basis
(Millan, 2016)
211. The admission of a new partner effected through purchase of interest in the partnership is

A. Recorded in the partnership books as a debit to cash or other asset and credit
to the incoming partner’s capital account
B. Recorded in the partnership books as a transfer within equity
C. Recorded in the partnership books as a transfer from equity to liability
D. Not recorded in its entirety
(Millan, 2016)

212. State the proper order of liquidation


I. Outside creditors
II. Owners’ interests
III. Inside creditors
A. I, III, II
B. I, II, III
C. III, II, I
D. II, I, III

(Millan, 2016)

213. Lancelot is trying to decide whether to accept a salary of P40,000 or a salary of P25,000
plus a bonus of 10% of net income after salary and bonus as a means of allocating profit among
the partners. Salaries traceable to the other partners are estimated to be P100,000. What amount
of income would be necessary so that Lancelot would consider the choices to be equal?
A. P 165,000
B. P 290,000
C. P 265,000
D. P 305,000
(Dayag, 2015)
214. MM, NN, OO are partners with capital balances on December 31, 2015 of P 300,000, P
300,000 and P 200,000, respectively. Profits are shared equally. OO wishes to withdraw and it
is agreed that OO is to take certain equipment with second-hand value of P 50,000 and a note
for the balance of OO’s interest. The equipment are carried on the books at P65,000. Brand
new equipment may cost P 80,000. Compute for: (1) OO’s acquisition of the second-hand
equipment that will result to reduction in capital; (2) the value of the note that will OO get
from the partnership’s liquidation.
A. (1) P15,000 each for MM and NN, (2) P150,000
B. (1) P5,000 each for MM, NN and OO, (2) P145,000
C. (1) 5,000 each for MM, NN and OO, (2) P195,000
D. (1) P7,500 each for MM and NN, (2) P145,000
(Dayag, 2015)
215. RR and XX formed a partnership and agreed to divide initial capital equally, even though
RR contributed P25,000 and XX contributed P21,000 in identifiable assets. Under the bonus
approach to adjust the capital accounts. XX’s unidentifiable assets should be debited for:
A. P 11,500
B. P 4,000
C. P 2,000
D. P 0

(Dayag, 2015)

216. A. Smith, a partner in an accounting firm, decided to withdraw from the partnership,
Smith’s share of the partnership profits and losses was 20%. Upon withdrawing from the
partnership he was paid P88,800 in final settlement for his interest. The total of the partner’s
capital accounts before recognition of partnership goodwill prior to Smith’s withdrawal was
P252,000. After his withdrawal the remaining partners’ capital accounts, excluding their share
of goodwill, totaled P192,000. The total goodwill of the firm was:
A. P 144,000
B. P 168,000
C. P 192,000
D. P 300,000

(Dayag, 2015)

217. The following condensed balance sheet is presented for the partnership of AA, BB, and
CC, who share profits and losses in the ratio of 4:3:3, respectively:

Cash P 160,000
Other assets 320,000
Total P 480,000

Liabilities P 180,000
AA, Capital 48,000
BB, Capital 216,000
CC, Capital 36,000
Total P 480,000
The partners agreed to dissolve the partnership after selling the other assets for P200,000.
Upon dissolution of the partnership, AA should have received

A. P 0
B. P 48,000
C. P 72,000
D. P 84,000

(Dayag, 2015)

218. When property other than cash is invested in a partnership, at what amount should the
noncash property be credited to the contributing partner’s capital account?
A. Fair Value at the date of recognition
B. Contributing partner’s original cost
C. Assessed valuation for property tax purposes
D. Contributing partner’s tax basis
(AICPA 0594 F-35)

219. A partnership records a partner’s investment of assets in the business at

A. The market value of the assets invested


B. A special value set by the partners
C. The partner’s book value of the assets invested
D. Any of the above, depending upon the partnership agreement
(RPCPA 0598)

220. In a partnership liquidation, the final cash distribution to the partners should be made in
accordance with the

A. Partners’ profit and loss sharing ratio


B. Balances of the partners’ capital accounts
C. Ratio of capital contributions made by the partners
D. Ratio of capital contributions less withdrawals made by the partners
(RPCPA 1079)

221. As a result of the retirement of a partner in an existing partnership, the capital balance of
the remaining partners increases. If the assets of the partnership before retirement are
properly valued, which of the following statements is true?

A. The retiring partner received less than his capital balance before retirement
B. There is partnership net loss prior to the retirement of the said partner
C. The remaining partner gives bonus to the retiring partner
D. There is impairment of existing assets recognized prior to retirement
(CPAR PREBOARD WEEK, 2017)
222. If the partnership agreement does not specify how income is to be allocated, profits
should be allocated

A. Equally
B. In proportion to the weighted-average of capital invested during the period
C. Equitably so that partners are compensated for the time and effort expended
on behalf of the partnership
D. In accordance with an established ratio
(Gleim)

223. On June 30, 2016, a partnership was formed by Mendoza and Lopez. Mendoza
contributed cash. Lopez, previously a sole proprietor contributed non-cash assets
including a realty subject to a mortgage which was assumed by the partnership. Lopez’s
capital account at June 30,2016 should be recorded at

A. The fair value of the property on June 30, 2016


B. Lopez’s carrying amount of the property on June 30, 2016
C. The fair value of the property on June 30, 2016 less the mortgage payable
D. Lopez’s carrying amount of the property on June 30, 2016 less the mortgage
payable
(BAYSA & LUPISAN, 2016)

For numbers 224 to 225 refer to the problem below:


Diaz and Esteban entered into a partnership on February 1, 2016 by investing the following
assets:

Diaz Esteban
Cash P 15,000
Merchandise Inventory P 45,000
Land 15,000
Building 65,000
Furniture and Fixtures 100,000

The agreement between Diaz and Esteban provides that profits and losses are to be divided into
40% and 60% to Diaz and Esteban respectively. The partnership is to assume the P30,000
mortgage loan on the building.
224. If Esteban is to receive a capital credit equal to his profit and loss ratio, how much cash
must he invest?

A. 77,500
B. 97,500
C. 127500
D. 172,500
(BAYSA & LUPISAN, 2016)

225. Assuming Esteban invests P50,000 cash and each partner is to be credited for the full
amount of the net assets invested, the total capital of the partnership is

A. 210,000
B. 250,000
C. 260,000
D. 290,000
(BAYSA & LUPISAN, 2016)

226. Assuming the partnership agreement provides that the partners should initially have an
equal interest in the partnership capital, what is Esteban’s capital upon partnership
formation?
A. 95,000
B. 105,000
C. 115,000
D. 125,000
(BAYSA & LUPISAN, 2016)

227. Canlas, a partner in the 3C Partnership, has a 30% participation in partnership profits and
lossess. Canlas’ capital account had a net decrease of P120,000 during the calendar year
2016. During 2016, Canlas withdrew P260,000 (charged against his capital account) and
contributed property valued at P50,000 to the partnership. What was the profit of 3C
Partnership?

A. 300,000
B. 466,667
C. 700,000
D. 1,100,000
(BAYSA & LUPISAN, 2016)

228. Profit is the difference between


A. assets and liabilities
B. the incoming cash and outgoing cash
C. the assets purchased with cash contributed by the owner and the cash spent to operate
the business
D. the assets received for goods and services and the amounts used to provide the goods
and services

(Warren 9th ed.)

229. Which of the items below is not a business organization form?

A. Entrepreneurship C. Partnership
B. Proprietorship D. Corporation
(Warren 9th ed.)

230. An entity that is organized in which ownership is divided into shares of stock is a

A. Proprietorship C. Partnership
B. Corporation D. Governmental Unit

(Warren 9th ed.)

231. Financial reports are used by

A. Management C. Inventors
B. Creditors D. All are correct
(Warren 9th ed.)

232. Which of the following is not a characteristic of a corporation?


A. Corporations are organized as a separate legal taxable entity
B. Ownership is divided into shares of stock.
C. Corporations experience an ease in obtaining large amounts of resources by issuing
stock.
D. A corporation’s resources are limited to their individual owners’ resources.

(Warren 9th ed.)

233. He refers to a partner who contributed not only money and property but also
industry to the newly formed partnership.

A. industrial partner
B. nominal partner
C. capitalist-industrial partner
D. capitalist partner
(CPAR handout, 2018)

234. It refers to a type of partnership wherein all partners are liable to the creditors pro-
rata up to the extent of personal or separate assets after the partnership’s asset are
exhausted.

A. General partnership
B. Partnership by estoppel
C. Limited partnership
D. Particular partnership
(CPAR handout, 2018)
235. Which of the following will decrease the capital balance of a partner?
A. Share in partnership profit
B. Receipt of share in revaluation surplus from a partnership property, plant and
equipment
C. Drawing made by partner
D. Advances made by a partner to the partnership
(CPAR handout, 2018)
A, B and C decided to form ABC Partnership. It was agreed that A will contribute an equipment
with assessed value of P100,000 with historical cost of P800,000 and accumulated depreciation of
P600,000. A day after the partnership formation, the equipment was sold for P300,000.

B will contribute a land and building with carrying amount of P1,200,000 and fair value of
P1,500,000. The land and building are subject to a mortgage payable amounting to P300,000 to be
assumed by the partnership. The partners agreed that B will have 60% capital interest in the
partnership. The partners also agreed that C will contribute sufficient cash to the partnership.

236. What is the total agreed capitalization of ABC Partnership?


A. 1,500,000
B. 2,000,000
C. 2,500,000
D. 3,000,000

237. What is the cash to be contributed by C in the ABC Partnership?


A. 500,000
B. 600,000
C. 700,000
D. 800,000
(CPAR handout, 2018)

238. When property other than cash is invested in a partnership, at what amount should
the noncash property be credited to the contributing partner’s capital account?

A. Contributing partner’s tax basis


B. Contributing partner’s original cost
C. Fair value at the date of contribution
D. Assessed valuation for property tax purposes
(Wiley 2014)
239. In a limited partnership, a general partner
A. Is excluded from management
B. Is not entitled to a bonus at the end of the year
C. Has limited liability for partnership debt
D. Has unlimited liability for partnership debt
(BCA & L 10e)
240. Partnership drawings are
A. Usually maintained in a separate account from the partner’s capital account
B. Equal to partner’s salaries
C. Similar to advances made to partners and are included as assets on the balance sheet
D. Not discussed in the specific contract provisions of the partnership
(FT & C 11e)
241. Which of the following is an advantage of a partnership?
A. Mutual agency
B. Limited life
C. Unlimited liability
D. None of these
(J & C 3e)
242. The profit and loss sharing ratio should be
A. In the same ratio as the percentage interest owned by each partner
B. Based on relative effort contributed to the firm by the partners
C. A weighted average of capital and effort contributions
D. Based on any formula that the partners choose
(J & C 3e)
243. Maxwell is trying to decide whether to accept a salary of $60,000 or a salary of
$25,000 plus a bonus 0f 20% of net income after the bonus as a means of allocating profit
among the partners. What amount of income would be necessary so that Maxwell would
consider the choices to be equal?
A. $35,000
B. $85,000
C. $140,000
D. $210,000
(FT & C 11e)
244. Joan a senior partner in a fashion designing firm has a share of 30% in earnings. In
19x8, she transferred to the firm, property with current fair value of P25,000 but made
capital withdrawal of P130,000. If her closing capital balance was P60,000 less than her
beginning capital balance, how much was the partnership’s net income loss in 19x8?

A. P(45,000)
B. P135,000
C. P150,000
D. P180,000
(RRCPA 0598)
245. JJ and KK are partners who share profits and losses in the ratio of 60% and 40%
respectively. JJ’s salary is P60,000 and P30,000 for KK, the partners are also paid interest
on their average capital balances. In 2012, JJ received P30,000 of interest and KK, P12,000.
The profit and loss allocation is determined after deductions for the salary and interest
payments. If KK’s share in the residual income (income after deducting salaries and
interest) was P60,000 in 2012, what was the total partnership income?

A. P192,000
B. P345,000
C. P282,000
D. P387,000
(Dayag 11)

FINISH!
A year ago, you did not know today.
You did not know how you’d make it here.
But you made it here!

By grace, you made it here! 😊


- (M.H.N)

You might also like